You are on page 1of 246

Út a sikeres emelt matematikaérettségihez

György Dániel

2017
Tartalomjegyzék
1. Számelmélet 7
1.1. Négyzetszámok . . . . . . . . . . . . . . . . . . . . . . . . . . . . . . . . . . . . 7
1.2. Nem négyzetszám bizonyı́tása . . . . . . . . . . . . . . . . . . . . . . . . . . . . 7
1.2.1. A prı́mtényezős felbontásban szereplő prı́mek négyzetével is oszthatónak
kell lennie. . . . . . . . . . . . . . . . . . . . . . . . . . . . . . . . . . . . 7
1.2.2. Nem végződhet 2, 3, 7, 8-ra . . . . . . . . . . . . . . . . . . . . . . . . . 8
1.3. Nagyobb számmal oszthatóság . . . . . . . . . . . . . . . . . . . . . . . . . . . . 9
1.4. Diofantoszi egyenletek . . . . . . . . . . . . . . . . . . . . . . . . . . . . . . . . 10
1.5. Algebrai törtek egész számok . . . . . . . . . . . . . . . . . . . . . . . . . . . . 11
1.6. Oszthatósági feltételek #1 . . . . . . . . . . . . . . . . . . . . . . . . . . . . . . 13
1.7. Oszthatósági feltételek #2 . . . . . . . . . . . . . . . . . . . . . . . . . . . . . . 14
1.8. Nevezetes azonosság, mint bizonyı́tási módszer . . . . . . . . . . . . . . . . . . . 16

2. Függvények 17
2.1. Racionális törtfüggvény ábrázolása . . . . . . . . . . . . . . . . . . . . . . . . . 17
2.2. |f (x)| tı́pusú feladatok . . . . . . . . . . . . . . . . . . . . . . . . . . . . . . . . 18
2.3. Függvények paritása . . . . . . . . . . . . . . . . . . . . . . . . . . . . . . . . . 19
2.4. Egyenletek és egyenlőtlenségek grafikus megoldása . . . . . . . . . . . . . . . . . 22
2.5. A teljes négyzetté alakı́tás . . . . . . . . . . . . . . . . . . . . . . . . . . . . . . 26
2.6. |x + 1| + |x − 3| tı́pusú függvények . . . . . . . . . . . . . . . . . . . . . . . . . 27
2.7. Függvények kompozı́ciója . . . . . . . . . . . . . . . . . . . . . . . . . . . . . . 28
2.8. A paraméteres egyenletek grafikus megoldása . . . . . . . . . . . . . . . . . . . . 29
2.9. Racionális törtfüggvény egyszerűsı́tése és a folytonosság . . . . . . . . . . . . . . 34
2.10. Függvények határértéke . . . . . . . . . . . . . . . . . . . . . . . . . . . . . . . 36
2.11. Függvény monotonitásának következményei . . . . . . . . . . . . . . . . . . . . 37

3. Algebra 42
3.1. Az értelmezési tartomány és az értékkészlet fogalma . . . . . . . . . . . . . . . . 42
3.2. Polinomok szorzattá alakı́tása . . . . . . . . . . . . . . . . . . . . . . . . . . . . 44
3.3. Ekvivalens és nem ekvivalens átalakı́tások . . . . . . . . . . . . . . . . . . . . . 45

4. Egyenletek 48
4.1. Egyenlettı́pusok . . . . . . . . . . . . . . . . . . . . . . . . . . . . . . . . . . . . 48
4.1.1. Az abszolút értékes egyenlet . . . . . . . . . . . . . . . . . . . . . . . . . 48
4.1.2. A hatványegyenlet . . . . . . . . . . . . . . . . . . . . . . . . . . . . . . 49
4.1.3. A gyökös egyenlet . . . . . . . . . . . . . . . . . . . . . . . . . . . . . . . 49
4.1.4. Az exponenciális egyenlet . . . . . . . . . . . . . . . . . . . . . . . . . . 50
4.1.5. A logaritmusos egyenlet . . . . . . . . . . . . . . . . . . . . . . . . . . . 51
4.1.6. A trigonometrikus egyenlet . . . . . . . . . . . . . . . . . . . . . . . . . . 52
4.2. A |x + 1| + |x − 3| = 8 tı́pusú egyenletek . . . . . . . . . . . . . . . . . . . . . . 52
4.3. A gyökös egyenletek . . . . . . . . . . . . . . . . . . . . . . . . . . . . . . . . . 53
4.4. Egyenletrendszer megoldása új ismeretlen bevezetésével . . . . . . . . . . . . . . 55
4.5. Tömegszázalékos szöveges feladatok . . . . . . . . . . . . . . . . . . . . . . . . . 56
4.6. Sebességes szöveges feladatok . . . . . . . . . . . . . . . . . . . . . . . . . . . . 58
4.7. Munkavégzéssel kapcsolatos szöveges feladatok . . . . . . . . . . . . . . . . . . . 60
4.8. Számjegyes szöveges feladatok . . . . . . . . . . . . . . . . . . . . . . . . . . . . 62
4.9. Egyenletre vagy egyenletrendszerre vezető szöveges feladatok . . . . . . . . . . . 63
4.10. Gyökös egyenlet visszavezetése abszolút értékes egyenletre . . . . . . . . . . . . 66
4.11. Szélsőérték feladatok . . . . . . . . . . . . . . . . . . . . . . . . . . . . . . . . . 67
4.11.1. Elemi úton, másodfokú függvényábrázolással (diszkrimináns > 0) . . . . 67
4.11.2. Elemi úton, teljes négyzetté alakı́tással (diszkrimináns < 0) . . . . . . . . 68
4.11.3. Elemi úton, számtani és mértani középpel . . . . . . . . . . . . . . . . . 69
4.11.4. Differenciálással . . . . . . . . . . . . . . . . . . . . . . . . . . . . . . . . 69
4.12. Exponenciális egyenletek . . . . . . . . . . . . . . . . . . . . . . . . . . . . . . . 72
4.12.1. I. tı́pusú . . . . . . . . . . . . . . . . . . . . . . . . . . . . . . . . . . . . 72
4.12.2. II. tı́pusú . . . . . . . . . . . . . . . . . . . . . . . . . . . . . . . . . . . 74
4.12.3. III. tı́pusú . . . . . . . . . . . . . . . . . . . . . . . . . . . . . . . . . . . 75
4.13. Logaritmusos egyenletek – új alapra áttérés . . . . . . . . . . . . . . . . . . . . . 77
4.14. Egyenletrendszerek trigonometrikus egyenlettel . . . . . . . . . . . . . . . . . . . 79
4.15. Értékkészlet-vizsgálat egyenleteknél . . . . . . . . . . . . . . . . . . . . . . . . . 81

5. Egyenlőtlenségek 83
5.1. Negatı́v számmal szorzás, osztás . . . . . . . . . . . . . . . . . . . . . . . . . . . 83
5.2. Ismeretlent tartalmazó taggal szorzás, osztás . . . . . . . . . . . . . . . . . . . . 83
5.3. A másodfokú egyenlőtlenség . . . . . . . . . . . . . . . . . . . . . . . . . . . . . 85
5.4. A gyökös egyenlőtlenség . . . . . . . . . . . . . . . . . . . . . . . . . . . . . . . 89
5.5. Az abszolút értékes egyenlőtlenség . . . . . . . . . . . . . . . . . . . . . . . . . . 91
5.5.1. Az |x| ≤ a tı́pusú . . . . . . . . . . . . . . . . . . . . . . . . . . . . . . . 91
5.5.2. Az |x| ≥ a tı́pusú . . . . . . . . . . . . . . . . . . . . . . . . . . . . . . . 92
5.6. Az exponenciális egyenlőtlenség . . . . . . . . . . . . . . . . . . . . . . . . . . . 92
5.7. A logaritmusos egyenlőtlenség . . . . . . . . . . . . . . . . . . . . . . . . . . . . 94
5.8. A trigonometrikus egyenlőtlenség . . . . . . . . . . . . . . . . . . . . . . . . . . 95

6. Sı́kgeometria, térgeometria 97
6.1. Számolás a háromszög területével . . . . . . . . . . . . . . . . . . . . . . . . . . 97
6.2. A Pitagorasz-tétel . . . . . . . . . . . . . . . . . . . . . . . . . . . . . . . . . . . 100
6.3. A körszelet területe . . . . . . . . . . . . . . . . . . . . . . . . . . . . . . . . . . 105
6.4. Háromszögek hasonlósága . . . . . . . . . . . . . . . . . . . . . . . . . . . . . . 107
6.5. Testek összeillesztése . . . . . . . . . . . . . . . . . . . . . . . . . . . . . . . . . 110

7. Trigonometria 114
7.1. Másodfokúra vezető trigonometrikus egyenletek . . . . . . . . . . . . . . . . . . 114
7.2. Addı́ciós tételes feladatok . . . . . . . . . . . . . . . . . . . . . . . . . . . . . . 116
7.3. A cos x − sin x = 0 és cos x + sin x = 0 tı́pusú egyenletek . . . . . . . . . . . . . 120
7.4. A sin x = sin y, a cos x = cos y és a tg x = tg y tı́pusú tı́pusú egyenletek . . . . . 121
7.4.1. A sin x = sin y tı́pusú egyenletek . . . . . . . . . . . . . . . . . . . . . . . 121
7.4.2. A cos x = cos y tı́pusú egyenletek . . . . . . . . . . . . . . . . . . . . . . 122
7.4.3. A tg x = tg y tı́pusú egyenletek . . . . . . . . . . . . . . . . . . . . . . . 122
7.5. A sin2 x + cos2 x = 1 használata . . . . . . . . . . . . . . . . . . . . . . . . . . . 122
7.6. A koszinusz-tétel . . . . . . . . . . . . . . . . . . . . . . . . . . . . . . . . . . . 123
7.7. A dupla koszinusz-tétel . . . . . . . . . . . . . . . . . . . . . . . . . . . . . . . . 125
7.8. A tg és a ctg . . . . . . . . . . . . . . . . . . . . . . . . . . . . . . . . . . . . . . 129

8. Koordináta-geometria 132
8.1. A kör zárt egyenlete . . . . . . . . . . . . . . . . . . . . . . . . . . . . . . . . . 132
8.2. A távolságképlet . . . . . . . . . . . . . . . . . . . . . . . . . . . . . . . . . . . 134
8.3. Egy belső szög kiszámolása . . . . . . . . . . . . . . . . . . . . . . . . . . . . . . 135
8.4. Vektorok párhuzamosságának bizonyı́tása . . . . . . . . . . . . . . . . . . . . . . 137
8.5. Vektorok merőlegességének bizonyı́tása . . . . . . . . . . . . . . . . . . . . . . . 138
8.6. Egyenesek párhuzamosságának bizonyı́tása . . . . . . . . . . . . . . . . . . . . . 139
8.6.1. Normálvektoruk egymás skalár szorosa . . . . . . . . . . . . . . . . . . . 139
8.6.2. Meredekségük egyenlő . . . . . . . . . . . . . . . . . . . . . . . . . . . . 140
8.7. Egyenesek merőlegességének bizonyı́tása . . . . . . . . . . . . . . . . . . . . . . 141
8.7.1. A két egyenes normálvektorának skaláris szorzata 0 . . . . . . . . . . . . 141
8.7.2. A két egyenes meredekségének szorzata -1 . . . . . . . . . . . . . . . . . 141
8.8. Derékszög bizonyı́tása . . . . . . . . . . . . . . . . . . . . . . . . . . . . . . . . 141
8.9. Pont és egyenes távolsága . . . . . . . . . . . . . . . . . . . . . . . . . . . . . . 143
8.10. Paraméterezés (sugársor) . . . . . . . . . . . . . . . . . . . . . . . . . . . . . . . 147
8.10.1. Sugársor egyenletrendszerrel . . . . . . . . . . . . . . . . . . . . . . . . . 147
8.10.2. Sugársor a pont és egyenes távolsága képlettel (csak körnél) . . . . . . . 151
8.11. Paraméterezés (egyenessereg) . . . . . . . . . . . . . . . . . . . . . . . . . . . . 152
8.11.1. Egyenessereg egyenletrendszerrel . . . . . . . . . . . . . . . . . . . . . . 153
8.11.2. Egyenessereg a pont és egyenes távolsága képlettel (csak körnél) . . . . . 154
8.12. Húrnégyszög bizonyı́tása . . . . . . . . . . . . . . . . . . . . . . . . . . . . . . . 155
8.13. Lineáris programozás . . . . . . . . . . . . . . . . . . . . . . . . . . . . . . . . . 159

9. Kombinatorika 163
9.1. A ,,csomagos” feladatok . . . . . . . . . . . . . . . . . . . . . . . . . . . . . . . 163
9.2. Esetszétválasztás a nulla miatt . . . . . . . . . . . . . . . . . . . . . . . . . . . . 164
9.3. A szétbontásos feladatok #1 . . . . . . . . . . . . . . . . . . . . . . . . . . . . . 166
9.4. A szétbontásos feladatok #2 . . . . . . . . . . . . . . . . . . . . . . . . . . . . . 168
9.5. Az ,,összes-rossz” módszer . . . . . . . . . . . . . . . . . . . . . . . . . . . . . . 169
9.6. Összegre vagy szorzatra bontós feladatok . . . . . . . . . . . . . . . . . . . . . . 171

10.Valószı́nűségszámı́tás 173
10.1. A ,,csomagos” feladatok . . . . . . . . . . . . . . . . . . . . . . . . . . . . . . . 173
10.2. A két kockadobásos feladatok . . . . . . . . . . . . . . . . . . . . . . . . . . . . 173
10.3. A hipergeometrikus eloszlás . . . . . . . . . . . . . . . . . . . . . . . . . . . . . 175
10.4. A binomiális eloszlás . . . . . . . . . . . . . . . . . . . . . . . . . . . . . . . . . 176
10.5. A teljes valószı́nűség tétele . . . . . . . . . . . . . . . . . . . . . . . . . . . . . . 179
10.6. A Bayes-tétel . . . . . . . . . . . . . . . . . . . . . . . . . . . . . . . . . . . . . 180
10.7. Az ,,összes-rossz” módszer . . . . . . . . . . . . . . . . . . . . . . . . . . . . . . 182
10.8. A várható érték . . . . . . . . . . . . . . . . . . . . . . . . . . . . . . . . . . . . 186

11.Sorozatok 189
11.1. A páratlan tagú számtani sorozatok . . . . . . . . . . . . . . . . . . . . . . . . . 189
11.2. A páratlan tagú mértani sorozatok . . . . . . . . . . . . . . . . . . . . . . . . . 190
11.3. Számtani sorozat → mértani sorozat . . . . . . . . . . . . . . . . . . . . . . . . 191
11.4. Mértani sorozat → számtani sorozat . . . . . . . . . . . . . . . . . . . . . . . . 193
11.5. Sorozatok egymásba fűzése . . . . . . . . . . . . . . . . . . . . . . . . . . . . . . 194
11.6. Határérték számolás leosztással . . . . . . . . . . . . . . . . . . . . . . . . . . . 196
11.7. Határérték számolás e-vel . . . . . . . . . . . . . . . . . . . . . . . . . . . . . . 198
11.8. Határérték számolás rendőrelvvel . . . . . . . . . . . . . . . . . . . . . . . . . . 200
11.9. Határérték számolás konjugálttal . . . . . . . . . . . . . . . . . . . . . . . . . . 202
11.10.A mértani sor . . . . . . . . . . . . . . . . . . . . . . . . . . . . . . . . . . . . . 203
11.11.A mértani sor geometria feladatokban . . . . . . . . . . . . . . . . . . . . . . . . 205
11.12.Sorozatok monotonitása . . . . . . . . . . . . . . . . . . . . . . . . . . . . . . . 212
11.13.Sorozatok korlátossága . . . . . . . . . . . . . . . . . . . . . . . . . . . . . . . . 215
12.Analı́zis 218
12.1. A lánc-szabály . . . . . . . . . . . . . . . . . . . . . . . . . . . . . . . . . . . . . 218
12.2. Függvények szélsőértéke . . . . . . . . . . . . . . . . . . . . . . . . . . . . . . . 219
12.3. Függvények görbülete . . . . . . . . . . . . . . . . . . . . . . . . . . . . . . . . . 221
12.4. A határozott integrál és a terület kapcsolata . . . . . . . . . . . . . . . . . . . . 223
12.5. Két függvény közötti terület . . . . . . . . . . . . . . . . . . . . . . . . . . . . . 229
12.6. Határozott integrál feladatok . . . . . . . . . . . . . . . . . . . . . . . . . . . . . 233

13.Gondolkodási műveletek, halmazok 236


13.1. A skatulya-elv . . . . . . . . . . . . . . . . . . . . . . . . . . . . . . . . . . . . . 236
13.2. A teljes indukciós bizonyı́tás . . . . . . . . . . . . . . . . . . . . . . . . . . . . . 238
13.3. Az indirekt bizonyı́tás . . . . . . . . . . . . . . . . . . . . . . . . . . . . . . . . 240
13.4. Bizonyı́tás elégséges feltételek láncolatával . . . . . . . . . . . . . . . . . . . . . 241
13.5. Bizonyı́tás nevezetes közepekkel . . . . . . . . . . . . . . . . . . . . . . . . . . . 243
13.6. Folytonos halmazok megadása . . . . . . . . . . . . . . . . . . . . . . . . . . . . 244
Előszó
Kedves Érettségiző!

Több mint egy évtizedes munkám során sokféle tehetségű, felkészültségű és szorgalmú
diákkal találkoztam és készı́tettem fel őket az emelt szintű matematikaérettségire. Azt tapasz-
taltam, hogy hiába a sok feladat, hiába a sok gyakorlás, az emelt szinten érettségizőknek nem
mindig sikerül olyan jól a dolgozat, mint ahogyan azt remélték. Ekkor született az ötlet, hogy
ı́rnom kellene egy olyan trükkgyűjteményt, ami ahelyett, hogy feladatokat gyakoroltatna velük,
arról szól, hogy milyen trükkökkel, gondolatokkal, módszerekkel kell magabiztosan rendelkeznie
egy emelt szinten érettségiző diáknak ahhoz, hogy nagyobb sikere legyen.
Ez a könyv tehát nem tankönyv és nem is feladatgyűjtemény. Nincsenek benne sem de-
finı́ciók, sem tételek. Ez egy ,,hasznos tippek tárháza” könyv. Azt a célt szolgálja, hogy ha
találkoztok egy vadidegen feladattal, akkor legyen egy első gondolatmenet, amivel megpróbál-
játok azt megoldani. Ha nem sikerül, akkor legyen egy második stb. Másképp fogalmazva ,,ha ezt
látom a feladat szövegében, vagy ezek a kiindulási adatok vannak megadva, akkor megpróbálom
először ezt a gondolatmenetet, aztán azt, stb”. A kidolgozott példák alapos magyarázatán kı́vül
igyekeztem felhı́vni a figyelmet az esetleges buktatókra, tı́pushibákra is. A kidolgozott példáknak
a végén a ♦ szimbólum áll.
Szintén tapasztaltam munkám során, hogy sok diákot összezavar, ha egy feladatra több
megoldást adnak, vagy ha rengeteg a kivétel és az ezekhez tartozó feltétel. Ennek következtében
a könyvben néhány helyen nem a teljes igazság szerepel. Több módszernél lehet másképp is
csinálni bizonyos lépéseket, de szerintem egyszerűbb megjegyezni, ha mindenre ugyanazt a
sémát ráhúzzuk, mintha azt jegyeznénk meg, hogy ,,ezt ı́gy kell csinálni, de ha ez meg az van,
akkor kivételesen lehet úgy is”.
A gondolatmenetek magyarázata időnként szájbarágós, illetve néha egyszerű nyelvezetet
is használtam azért, hogy érthetőbb legyen azok számára is, akik az emelt szintű tananyaggal
megküzdenek. A szabatos szaknyelv emiatt hiányzik, de remélem ettől csak ,,diákközelibb” lett.
Ez a könyv leginkább az emelt szinten érettségiző diákoknak szól, de a középszinten érettsé-
gizők is találhatnak benne hasznos gondolatokat. Igyekeztem minden fő témakörhöz összegyűj-
teni olyan alapvető módszereket és gondolatokat, amelyek megkönnyı́thetik az elindulást egy
ismeretlen feladatnál is. A feladatok egy része korábbi emelt szintű érettségi feladat, a többi
pedig a ,,sárga csı́kos” és ,,zöld csı́kos” feladatgyűjteményekből, a régi ,,zöld könyvből” van,
illetve saját szerzemények is találhatók benne.
Bı́zom benne, hogy hasznosnak fogjátok találni ezt a könyvet. Mindenkinek sikeres felkészü-
lést kı́vánok az érettségihez!

Budapest, 2017.

György Dániel
1. fejezet

1. Számelmélet
1.1. Négyzetszámok
Egy négyzetszám prı́mtényezős felbontásában minden kitevőnek párosnak kell lennie.

Példa 1 :
Hány négyzetszám osztója van 190 512 000-nek?

190 512 000 = 27 · 35 · 53 · 72


Egy négyzetszámban minden kitevő páros. A 2 kitevője 0, 2, 4 vagy 6 lehet. A 3 kitevője 0,
2 vagy 4 lehet. Az 5 kitevője 0 vagy 2 lehet. A 7 kitevője pedig szintén 0 vagy 2 lehet. A
lehetőségek számát összeszorozva kapjuk, hogy 4 · 3 · 2 · 2 = 48 lehetőség van négyzetszám
előállı́tására.♦

Példa 2 :
Igaz-e, hogy ha egy szám osztója a2 -nek, akkor osztója a-nak is?

a2 = p1 2α1 ·p2 2α2 ·. . .·pn 2αn alakú, mivel négyzetszám (ahol pi -k különböző prı́mszámok, αi ∈ N).
Vegyük osztójának bármelyik pi 2αi -t. Ekkor a = p1 α1 · p2 α2 · . . . · pn αn alakú, és ı́gy nem osztható
már egyik prı́m négyzetével sem. Számoknál pl. a 9 osztója a 225-nek, de a 15-nek már nem.♦

Példa 3 : √
Bizonyı́tsuk be, hogy 5 irracionális.
√ √ p
Induljunk ki abból, hogy 5 racionális, vagyis 5= (ahol p, q ∈ N+ ).
q
5q 2 = p2

p2 és q 2 is négyzetszám, vagyis ha szerepel p és q felbontásában az 5-ös prı́m, akkor páros
kitevővel van benne. De 5q 2 -ben ekkor már az 5-ös páratlan
√ kitevővel szerepel. Emiatt nem
lehetnek egyenlők soha. Így nem lehet racionális szám 5, vagyis az eredeti állı́tás igaz.♦

1.2. Nem négyzetszám bizonyı́tása


Ha egy számról azt kell megmutatni, hogy nem lehet négyzetszám, akkor

1.2.1. A prı́mtényezős felbontásban szereplő prı́mek négyzetével is oszthatónak kell


lennie.
Példa 1 :
Az 52 941 számjegyeit leı́rjuk az összes lehetséges sorrendben. Bizonyı́tsuk be, hogy egyik szám
sem négyzetszám.

Csak a hárommal és a kilenccel való oszthatóságnak a szabálya az, hogy a számjegyek összegét
kell figyelni. Ez az egyetlen oszthatósági tulajdonság állandó most, mivel ha a számjegyeket
permutáljuk, sokféle végződés előfordulhat. 5 + 2 + 9 + 4 + 1 = 21. A 21 osztható hárommal,
de kilenccel már nem, ı́gy nem lehet négyzetszám egyik ötjegyű szám sem.♦

7
1. fejezet

Példa 2 :
Hány olyan természetes szám van, aminek a négyzete 2001 darab egyesből és tetszőleges számú
nullából áll?

Megint a hárommal való oszthatóságot kell vizsgálni. A számjegyek összege 2001, ami osztható
hárommal, de kilenccel már nem, tehát nincs ilyen négyzetszám.♦

Példa 3 :
Bizonyı́tsuk be, hogy kilenc egymást követő egész szám négyzetének összege nem lehet négyzet-
szám.

Hogy egyszerűbb legyen a felı́rás és a szimmetria miatt kiessenek az elsőfokú tagok, célszerű a
középső tagból kiindulni és onnan felı́rni a kilenc számot.

(n − 4)2 + (n − 3)2 + (n − 2)2 + (n − 1)2 + n2 + (n + 1)2 + (n + 2)2 + (n + 3)2 + (n + 4)2 =


= 9n2 + 60 = 3 3n2 + 20


Ez a szám hárommal ugyan osztható, de kilenccel már nem, mivel a második szorzótényező
nem osztható hárommal. Ezt onnan tudjuk, hogy 3n2 osztható, de 20 nem, ı́gy az összeg sem.
Vagyis nem lehet négyzetszám.♦

Példa 4 :
Bizonyı́tsuk be, hogy öt egymást követő egész szám négyzetének összege nem lehet négyzetszám.

Ismét kihasználjuk a szimmetriát.

(n − 2)2 + (n − 1)2 + n2 + (n + 1)2 + (n + 2)2 = 5n2 + 10 = 5 n2 + 2




Ez a szám biztosan osztható öttel, de 25-tel már nem, mert a második szorzótényező nem oszt-
ható öttel. Ezt onnan tudjuk, hogy minden öttel osztható szám nullára vagy ötre végződik,
vagyis n2 -nek nyolcra vagy háromra kell végződnie, ami lehetetlen.♦

1.2.2. Nem végződhet 2, 3, 7, 8-ra


Egy négyzetszám nem végződhet 2, 3, 7, 8 egyikére sem. Ez azért van ı́gy, mert ha végignézzük
a számjegyek négyzeteinek végződéseit, nem jön ki 2, 3, 7, 8 soha.

02 = 0; 12 = 1; 22 = 4; 32 = 9; 42 = 16; 52 = 25; 62 = 36; 72 = 49; 82 = 64; 92 = 81

Példa 1 :
Bizonyı́tsuk be, hogy az 19931994 + 19941993 nem lehet négyzetszám.

Amikor szeretnénk megtudni, hogy milyen számjegyre végződik egy nagy szám bizonyos hat-
ványa, elegendő az utolsó számjegy hatványának utolsó számjegyét megtudni. 19931 3-ra végző-
dik, 19932 9-re, 19933 7-re, 19934 1-re, majd 19935 megint 3-ra. Lehet látni, hogy a végződések
négyesével ismétlődnek: (3; 9; 7; 1) , (3; 9; 7; 1) , . . .
Ahhoz hogy megtudjuk 19931994 utolsó számjegyét, az 1994-et el kell osztani maradékosan 4-
gyel. 1994 = 498 · 4 + 2. Vagyis 498 teljes ,,négyes blokk” készı́thető, és a 499. blokk második

8
1. fejezet

helyét foglalja el az 19931994 . Így biztosan 9-re végződik.


Ugyanı́gy számoljuk ki 19941993 utolsó számjegyét is. 19941 4-re, 19942 6-ra, majd 19943 megint
4-re végződik. Most még egyszerűbb dolgunk van. Minden páratlan kitevős hatvány 4-re, páros
kitevős 6-ra végződik, tehát 19941993 4-re végződik.
Egy 9-re és egy 4-re végződő szám összege 3-ra végződik, vagyis nem lehet négyzetszám.♦

Példa 2 :
Bizonyı́tsuk be, hogy n! + 2017 egyetlen n pozitı́v egész szám esetén sem lehet négyzetszám.

n! = 1 · 2 · 3 · . . . · n

Ha n ≥ 5, akkor n! biztosan nullára végződik a 2 · 5 miatt, vagyis n! + 2017 hétre végződik,


tehát nem lehet négyzetszám.

Ha n=1 ⇒ n! + 2017 = 2018


Ha n=2 ⇒ n! + 2017 = 2019
Ha n=3 ⇒ n! + 2017 = 2023
Ha n=4 ⇒ n! + 2017 = 2041
Ezek sem négyzetszámok.♦

1.3. Nagyobb számmal oszthatóság


Ha egy nagy számmal való oszthatóságot vizsgálunk, akkor olyan számok szorzatára kell bon-
tani, melyek relatı́v prı́mek, és azokkal egyszerre kell oszthatónak lennie. Ha valamelyik szorzó-
tényező a 3 vagy a 9, akkor azt kell legkésőbb megvizsgálni, amikor már a másik / többi
szorzótényező az ismeretleneket leszűkı́tette. Ellenkező esetben rengeteg esetet kellene végig-
néznünk.

Példa 1 :
Milyen számjegyeket ı́rjunk az ismeretlenek helyére, hogy 12 | 32X45Y igaz legyen?

A 12 felbontható 2 · 6, vagy 3 · 4 alakban is, de csak a 3 és 4 relatı́v prı́mek. Ha a hárommal való
oszthatósággal kezdenénk, akkor az X bármilyen számjegy lehetne, és mindegyikhez tartozna
sok Y lehetőség is. Így fordı́tva csináljuk.
Mivel 4 | 32X45Y , ezért Y = 2 vagy Y = 6.
Ha Y = 2, akkor a számjegyek összege miatt X = 2; 5; 8 lehet.
Ha pedig Y = 6, akkor X = 1; 4; 7.
A végeredmények (2; 2) , (5; 2) , (8; 2) , (1; 6) , (4; 6) , (7; 6).♦

Példa 2 :
Milyen számjegyeket ı́rjunk az ismeretlenek helyére, hogy 72 | 5X12Y igaz legyen?

A 72 több alakban is felbontható szorzatra, de csak a 8 és 9 relatı́v prı́mek.


Mivel 8 | 5X12Y , ezért Y = 0 vagy Y = 8.
Ha Y = 0, akkor a számjegyek összege miatt X = 1 lehet.
Ha Y = 8, akkor X = 2.
A végeredmények (1; 0) , (2; 8).♦

9
1. fejezet

1.4. Diofantoszi egyenletek


Diofantoszi-egyenletnek nevezzük a legalább kétismeretlenes egyenletet, melynek megoldásai
csak egész számok lehetnek. Néha egyszerű következtetéssel oldjuk meg. Néha érdemes az
egyik ismeretlent a lehető legnagyobbnak megválasztani, és azt folyamatosan csökkenteni, végig
kiszámolva hozzá a többi ismeretlent, ı́gy nem marad ki egy lehetőség sem. Ha összevissza
csináljuk, vagy ránézésre, akkor kimaradhat néhány megoldás.

Példa 1 :
Oldjuk meg a nemnegatı́v egész számok halmazán: 2a + 5b = 30

Ha a ≥ 16, nincs megoldás.


Ha a = 15, akkor b = 0.
Ha a = 14; 13; 12; 11, akkor nem létezik hozzá egész b.
Ha a = 10, akkor b = 2.
Ha a = 9; 8; 7; 6, akkor nem létezik hozzá egész b.
Ha a = 5, akkor b = 4.
Ha a = 4; 3; 2; 1, akkor nem létezik hozzá egész b.
Ha a = 0, akkor b = 6.

A végeredmények (15; 0) , (10; 2) , (5; 4) , (0; 6).♦

Példa 2 :
Határozzuk meg azt az abcd négyjegyű számot, amelyre abcd − abc − ab − a = 1994.

abcd − abc − ab − a = 1994


1000a + 100b + 10c + d − 100a − 10b − c − 10a − b − a = 1994
889a + 89b + 9c + d = 1994

Mivel a szerepel a legnagyobb együtthatóval, ezért az ő értéke a legdominánsabb.


Ha a ≥ 3, akkor a bal oldal értéke legalább 2667, azaz túl sok.
Ha a = 1, és emellett b = c = d = 9, akkor is a bal oldal legfeljebb 1780, tehát kevés.
Ebből következik, hogy a = 2, vagyis az egyenlet átrendezhető 89b + 9c + d = 216 alakba.
Hasonlóan folytatjuk a gondolatmenetet.
Ha b ≥ 3, akkor a bal oldal értéke legalább 267, vagyis túl sok.
Ha b = 1, és emellett c = d = 9, akkor is a bal oldal legfeljebb 179, tehát kevés.
Vagyis b = 2 adódik, az egyenlet ismét átrendezhető 9c + d = 38 alakba.
Erről már le is olvasható, hogy c = 4, d = 2 az egyetlen megoldás.
A végeredmény a 2242.♦

Példa 3 :
Oldjuk meg a pozitı́v egész számok halmazán: 3xy − 4y 2 = 18

y (3x − 4y) = 18

A következő lehetőségek állnak elő: 1 · 18; 2 · 9; 3 · 6; 6 · 3; 9 · 2; 18 · 1

10
1. fejezet

Ha y = 1 ⇒ 3x − 4 = 18, nincs egész megoldás.


Ha y = 2 ⇒ 3x − 8 = 9, nincs egész megoldás.
Ha y = 3 ⇒ 3x − 12 = 6, vagyis x = 6.
Ha y = 6 ⇒ 3x − 24 = 3, vagyis x = 9.
Ha y = 9 ⇒ 3x − 36 = 2, nincs egész megoldás.
Ha y = 18 ⇒ 3x − 72 = 1, nincs egész megoldás.
A végeredmény x = 6; y = 3 vagy x = 9; y = 6.♦

Példa 4 :
Melyek azok a háromjegyű számok, melyek egyenlők a számjegyeik összegének 18-szorosával?

abc = 18 (a + b + c)
100a + 10b + c = 18a + 18b + 18c
82a = 8b + 17c

82a és 8b biztosan páros, ı́gy c helyére páros számjegyet kell illeszteni, hogy 17c is páros legyen.
Ha a ≥ 3, akkor a bal oldal legalább 246, a jobb oldal pedig a maximális b = 9; c = 8 helyet-
tesı́tés mellett sem lehet ennyi.
Ha a = 2, akkor 164 = 8b + 17c. Ha c helyére behelyettesı́tjük a lehetséges értékeit, csak c = 4
esetben kapunk b-re egész számot, de b = 12 nem számjegy.
Ha a = 1, akkor 82 = 8b + 17c. Behelyettesı́tjük c helyére ugyanúgy a {0; 2; 4; 6; 8} halmaz
elemeit, és az egyetlen megoldás a c = 2; b = 6 esetén lesz.
Ez a szám tehát a 162.♦

Példa 5 :
Egy vizsgán 20 kérdésre kellett válaszolni. Az értékeléskor minden jó válasz öt-öt pontot ért,
viszont minden rossz válasz esetén két-két pontot levontak. Ha egy kérdésre a vizsgázó nem
válaszolt, akkor arra nulla pontot kapott. Egy vizsgázó 48 pontot gyűjtött. Hány jó választ
adott?

Legyen j a jó válaszok száma, r a rossz válaszok száma. Ekkor 20 − j − r a meg nem válaszolt
kérdések száma, de erre nulla pont jár, ı́gy nem szerepel az egyenletben.
5j − 2r = 48, amiből megállapı́tható azonnal, hogy j ≥ 10
2r biztosan páros szám, ezért 5j-nek, tehát j-nek is biztosan párosnak kell lennie.

Ha j = 10 ⇒ r = 1
Ha j = 12 ⇒ r = 6
Ha j = 14 ⇒ r = 11
Az utolsó sor már sok kérdést jelentene.
Két lehetőség van akkor: 10 jó, 1 rossz és 9 meg nem válaszolt kérdés, vagy 12 jó, 6 rossz és 2
meg nem válaszolt.♦

1.5. Algebrai törtek egész számok


Amikor egy algebrai törtről kell megmondani, hogy milyen n esetén lesz a tört értéke egész
szám, akkor a számlálóban előállı́tjuk a nevezőnek egy többszörösét, és utána korrigáljuk, hogy
az átalakı́tás stimmeljen. Ezután a számlálóból ,,leszedjük” a legtöbbször benne lévő nevezőt,

11
1. fejezet

ı́gy a számlálóban már csak egy egész szám marad. Ennek természetesen oszthatónak kell lennie
a nevezővel, ı́gy ezeket az eseteket kell megvizsgálnunk.

Példa 1 :
2n − 1
Milyen n egészre lesz a tört értéke egész szám?
n−1

2n − 1 2 (n − 1) + 1 2 (n − 1) 1 1
= = + =2+
n−1 n−1 n−1 n−1 n−1

Ebből tudjuk, hogy n − 1 = 1 vagy −1 lehet csak, mivel a nevezőnek az 1 osztójának kell lennie.
A végeredmény n ∈ {2; 0}.♦

Példa 2 :
4 − 5n
Milyen n egészre lesz a tört értéke egész szám?
n+3

4 − 5n −5 (n + 3) + 19 −5 (n + 3) 19 19
= = + = −5 +
n+3 n+3 n+3 n+3 n+3

Ebből következik, hogy n + 3 = 1; −1; 19; −19 lehet csak, mivel a nevezőnek a 19 osztójának
kell lennie.
A végeredmény n ∈ {−2; −4; 16; −22}.♦

Ez a gondolatmenet másodfokú vagy akár magasabb fokú algebrai törtekre is eljátszható, csak
akkor több lépésben.

Példa 3 :
n2 + 2n + 5
Milyen n egészre lesz a tört értéke egész szám?
n−2

n2 + 2n + 5 n (n − 2) + 4n + 5 n (n − 2) + 4 (n − 2) + 13
= = =
n−2 n−2 n−2
n (n − 2) 4 (n − 2) 13 13
= + + =n+4+
n−2 n−2 n−2 n−2

Azaz n − 2 = 1; −1; 13; −13 lehet csak, mivel a nevezőnek a 13 osztójának kell lennie.
A végeredmény n ∈ {3; 1; 15; −11}.♦

Példa 4 :
Melyek azok a téglalapok, amelyeknek cm-ben kifejezett oldalai pozitı́v egész számok, és a cm2 -
ben kifejezett területük, illetve cm-ben kifejezett kerületük mérőszáma ugyanaz a pozitı́v egész
szám?

ab = 2a + 2b
ab − 2a = 2b
a (b − 2) = 2b
2b 2 (b − 2) + 4 4
a = = =2+
b−2 b−2 b−2
A feladatot az átrendezéssel visszavezettük oda, hogy b − 2 osztója 4-nek.

12
1. fejezet

Így b − 2 = 1; 2; 4, vagyis b = 3; 4; 6. Hozzá kiszámolva a = 6; 4; 3.


Vagyis két ilyen téglalap létezik. Az egyiknek 3 cm és 6 cm, a másiknak 4 cm és 4 cm a két
oldala.♦

Példa 5 :
Három prı́mszám szorzata összegük ötszörösével egyenlő. Melyik ez a három szám?

pqr = 5 (p + q + r)

A jobb oldal osztható öttel, ı́gy a bal oldal is, vagyis az egyik prı́m 5.

5qr = 5 (5 + q + r)
qr = 5+q+r
qr − q = 5+r
q (r − 1) = 5+r
5+r 1 · (r − 1) + 6 6
q = = =1+
r−1 r−1 r−1
r − 1 lehetséges értékei 1; 2; 3; 6, vagyis r lehetséges értékei 2; 3; 4; 7. A 4 nem prı́m, ı́gy
nem megoldás. Marad az r = 2; 3; 7. Hozzájuk kiszámolva q = 7; 4; 2, de a 4 megint nem
prı́m. Így az egyetlen megoldás a 2; 5; 7 számhármas.♦

1.6. Oszthatósági feltételek #1


Amikor oszthatóságokat vizsgálunk úgy, hogy nincsenek számjegyek, csak ,,n-es alakban felı́rt
számok”, akkor az osztót prı́mtényezőkre kell bontani és megmutatni, hogy az általánosan felı́rt
kifejezés ezek mindegyikével osztható. Előfordulhat az a gondolat is, hogy szorzatra bontjuk,
vagy a végződését vizsgáljuk.

Példa 1 :
Bizonyı́tsuk be, hogy tetszőleges természetes számra 2 | n2 − n.

n2 − n = n (n − 1)

Vagyis két szomszédos természetes szám szorzata. Az egyik páros, a másik páratlan, vagyis a
szorzat biztosan páros.♦

Példa 2 :
Bizonyı́tsuk be, hogy tetszőleges természetes számra 6 | n3 − n.

Akkor osztható hattal, ha van benne egy kettes és egy hármas osztó.

n3 − n = n n2 − 1 = (n − 1) n (n + 1)


Vagyis három szomszédos természetes szám szorzata. Három szomszédos természetes szám közül
az egyik biztosan osztható hárommal (a másik kettő pedig egy illetve kettő maradékot ad
hárommal osztva), és biztosan van legalább egy páros is. Tehát a szorzat osztható hattal. ♦

13
1. fejezet

Példa 3 :
Bizonyı́tsuk be, hogy tetszőleges természetes számra 30 | n5 − n.

Akkor osztható 30-cal, ha van benne egy kettes, egy hármas és egy ötös osztó.

n5 − n = n n4 − 1 = n n2 − 1 n2 + 1 = (n − 1) n (n + 1) n2 + 1
   

Három szomszédos természetes szám szorzata látható újra. A három szomszédos természetes
szám közül az egyik biztosan osztható hárommal, illetve biztosan van legalább egy páros is.
Ha a három szomszédos természetes szám bármelyike osztható öttel, akkor készen vagyunk. Ha
egyik sem, akkor ez azt jelenti, hogy n öttel osztva kettő vagy három maradékot ad, azaz 5k + 2
vagy 5k + 3 alakú.
Ha n = 5k + 2 áll fenn, akkor:

n2 + 1 = (5k + 2)2 + 1 = 25k 2 + 20k + 5 = 5 5k 2 + 4k + 1




Ha n = 5k + 3 teljesül, akkor:

n2 + 1 = (5k + 3)2 + 1 = 25k 2 + 30k + 10 = 5 5k 2 + 6k + 2




Mindkét esetben az n2 + 1 szorzótényező lesz osztható öttel.♦

Példa 4 :
104n−1 + 2 103n+3 + 23
Bizonyı́tsuk be, hogy tetszőleges pozitı́v egész számra + egész szám.
3 9
Egy tı́zhatvány egy darab egyesből és nullákból áll. Ha hozzáadunk kettőt, a számjegyek összege
három lesz, tehát osztható hárommal a számláló. A másik törtnél a tı́zhatványhoz nyolcat adunk
hozzá, vagyis itt a számjegyek összege kilenc lesz, tehát ez a tört pedig osztható kilenccel.♦

1.7. Oszthatósági feltételek #2


Olyan oszthatósági feladatok is vannak, ahol betűkkel műveleteket végzünk, és ezek után be
kell bizonyı́tani, hogy oszthatók egy számmal. Ebben az esetben a helyi érték szerint kibontjuk
őket, és általában a megoldás kulcsa az, hogy kiemelhető egy szám, amiről már tudjuk, hogy
osztható a megadott számmal.

Példa 1 :
Bizonyı́tsuk be, hogy ha egy szám abcabc alakú, akkor osztható 91-gyel.

Kibontjuk a számot.

abcabc = 100000a + 10000b + 1000c + 100a + 10b + c = 100100a + 10010b + 1001c

Az összegből 1001 kiemelhető.

100100a + 10010b + 1001c = 1001 (100a + 10b + c)

Mivel 1001 = 11 · 91, ezért biztosan osztható 91-gyel.♦

14
1. fejezet

Példa 2 :
Bizonyı́tsuk be, hogy ha kétjegyű számot megszorzunk kettővel, majd az eredmény után leı́rjuk
az eredeti kétjegyű számot, akkor olyan négy- vagy ötjegyű számot kapunk, amely osztható
67-tel.

Az eredeti szám 10a + b alakú. A kétszerese 20a + 2b. Ha mögé ı́rjuk az eredeti kétjegyű számot,
akkor ezt megszorozzuk százzal, mert arrébb toljuk két helyi értékkel.
Az új számunk:

100 (20a + 2b) + 10a + b = 2010a + 201b

Ebből 201 kiemelhető.

2010a + 201b = 201 (10a + b)

Mivel 201 = 67 · 3, ezért biztosan osztható 67-tel.♦

Példa 3 :
Bizonyı́tsuk be, ha egy négyjegyű számhoz hozzáadjuk a számjegyei helyi érték szerinti sor-
rendjének megfordı́tásával kapott számot, akkor az eredmény osztható 11-gyel.

abcd + dcba = 1000a + 100b + 10c + d + 1000d + 100c + 10b + a = 1001a + 110b + 110c + 1001d

Kiemelhető 11.

1001a + 110b + 110c + 1001d = 11 (91a + 10b + 10c + 91d)

Az összeg biztosan osztható 11-gyel.♦

Példa 4 :
Bizonyı́tsuk be, hogy két páratlan szám négyzetének különbsége mindig osztható nyolccal.

Egy páratlan szám 2k + 1 alakú.

(2k + 1)2 − (2l + 1)2 = 4k 2 + 4k + 1 − 4l2 + 4l + 1 = 4k 2 + 4k − 4l2 − 4l




Kiemelhető 4.

4k 2 + 4k − 4l2 − 4l = 4 k 2 + k − l2 − l


Most még nem vagyunk kész, mert k 2 + k − l2 − l-ről be kell látnunk, hogy páros. Ez eléggé
nyilvánvaló, hiszen k 2 + k − l2 − l = k (k + 1) − l (l + 1), vagyis két szomszédos szám szorzatából
kivonjuk két szomszédos szám szorzatát. Ez mindig páros, mert két szomszédos szám szorzata
páros, és páros számok különbsége mindig páros.♦

15
1. fejezet

1.8. Nevezetes azonosság, mint bizonyı́tási módszer


Időnként nevezetes azonosságok felhasználásával is lehet bizonyı́tani oszthatóságot.

an − bn = (a − b) (an−1 + an−2 b + an−3 b2 + . . . + abn−2 + bn−1 )


2n+1
a + b2n+1 = (a + b) (a2n − a2n−1 b + a2n−2 b2 − . . . ± . . . + b2n )

A bizonyı́tásokhoz csak az fontos, amit kiemelünk az elején, a hosszú maradék szorzótényező


nem számı́t.

Példa 1 :
Bizonyı́tsuk be, hogy tetszőleges természetes számra 11 | 100n − 1.

100n − 1 = 100n − 1n = (100 − 1) 100n−1 + . . . + 1 = 99 100n−1 + . . . + 1


 

Osztható 11-gyel, mert a 99 osztható 11-gyel.♦

Példa 2 :
Bizonyı́tsuk be, hogy tetszőleges természetes számra 15 | 24n − 1.

24n − 1 = 16n − 1n = (16 − 1) 16n−1 + . . . + 1 = 15 16n−1 + . . . + 1


 

Osztható 15-tel.♦

Példa 3 :
Bizonyı́tsuk be, hogy tetszőleges természetes számra 6 | 52n − 1.

52n − 1 = 25n − 1n = (25 − 1) 25n−1 + . . . + 1 = 24 100n−1 + . . . + 1


 

Osztható 6-tal, mert a 24 osztható 6-tal.♦

Példa 4 :
Bizonyı́tsuk be, hogy tetszőleges természetes számra 181 | 3105 + 4105 .

3105 +4105 = 24321 +102421 = (243 + 1024) 24320 − . . . + 102420 = 1267 24320 − . . . + 102420
 

Osztható 181-gyel, mert 1267 = 181 · 7.♦

16
2. fejezet

2. Függvények
2.1. Racionális törtfüggvény ábrázolása
Ha ábrázolni kell egy racionális törtfüggvényt, akkor általában hiperbola lesz belőle. A gondo-
latmenet pedig ugyanaz, mint az 1.5. fejezetben.

Példa 1 :
2x − 1
Ábrázoljuk az f (x) = függvényt.
x−1

2x − 1 2(x − 1) + 1 1
f (x) = = =↑ 2 + ♦
x−1 x−1 x−1

Példa 2 :
1−x
Ábrázoljuk az f (x) = függvényt.
x+1

1−x −(x + 1) + 2 2
f (x) = = =↓ −1 + ♦
x+1 x+1 x+1

17
2. fejezet

2.2. |f (x)| tı́pusú feladatok


Ha egy függvényt abszolút értékbe teszünk, akkor az azt jelenti, hogy az x-tengely felett lévő
rész marad, az alatta lévő rész pedig tengelyesen feltükröződik.

Példa 1 :
Ábrázoljuk az f (x) = |3x + 5| függvényt.

Az eredeti lineáris függvényt a szokásos módon ábrázoljuk. Az x-tengely feletti része megmarad,
az alatta lévő részét pedig a zérushelytől feltükrözzük. Ez egy egyenes, tehát elég egy pontját
feltükrözni.♦

Példa 2 :
Ábrázoljuk az f (x) = |−x2 + 4| függvényt.

Az eredeti parabolát ábrázoljuk. Az x-tengely feletti része megmarad, az alatta lévő két ágát
pedig a zérushelytől feltükrözzük. Érdemes a pontosabb ábrázolás kedvéért megkeresni alul a
parabolán még egy-két pontot.♦

18
2. fejezet

Példa 3 :
Ábrázoljuk az f (x) = |sin x| függvényt.

Minden lenti ,,völgy” feltükröződik ı́gy a végeredményben csak ,,hegyek” lesznek.♦

Példa 4 :
Ábrázoljuk az f (x) = lg (x − 1) függvényt.

A lenti rész feltükrözésénél figyeljünk arra, hogy az a rész is ,,simuljon” az aszimptotához.♦

2.3. Függvények paritása


Ha összetettebb függvényt látunk, akkor érdemes megnézni a paritást, mert megkönnyı́theti az
ábrázolást. Ha kiderül, hogy van paritása, elég a függvény felét, vagyis az y-tengelytől jobbra
eső részét ábrázolni, mert a másik felét egy tükrözéssel megkaphatjuk.

Példa 1 : p
Ábrázoljuk az f (x) = |x| függvényt.

Ez a függvény páros, hiszen


p p
|x| = |−x|

Ez azt jelenti, hogy elég megvizsgálni azt az esetet, amikor x ≥ 0 és√ábrázolni, mert utána
a grafikont tengelyesen tükrözzük az y-tengelyre. Ha x ≥ 0, akkor f x-re egyszerűsödik le.
Ezt tudjuk azonnal ábrázolni. Most tükrözzük a függvényt. A végeredmény egy ,,madár” alakú
függvény lett.♦

19
2. fejezet

Példa 2 :
Ábrázoljuk az f (x) = x2 − 4|x| + 3 függvényt.

Ez a függvény is páros, mert

x2 − 4|x| + 3 = (−x)2 − 4 |−x| + 3

Megint elég az x ≥ 0 esettel foglalkozni.


Ha x ≥ 0, akkor f (x) az x2 − 4x + 3 kifejezésre egyszerűsödik le. Csak a belső abszolút érték
tüntethető el, mert arról tudjuk biztosan, hogy nemnegatı́v. A külső megmarad.

x2 − 4|x| + 3 = (x − 2)2 − 1

Első lépésként ábrázoljuk (x−2)2 − 1 ↓-et. A függvénynek csak az y-tengelytől jobbra eső része

kell. Ezek után ábrázoljuk (x − 2)2 − 1 -et a 2.2. fejezet alapján.

Végül az egészet tükrözzük az y-tengelyre. A végeredmény itt lent látható.♦

20
2. fejezet

Példa 3 :
Ábrázoljuk az f (x) = x · |x| függvényt.

Ez a függvény páratlan, mert

x · |x| = −(−x) · |−x|

Ez most azt jelenti, hogy elég megvizsgálni azt az esetet, amikor x ≥ 0 és ábrázolni, mert utána
a grafikont középpontosan tükrözzük az origóra.
Ha x ≥ 0, akkor f (x) = x2 . A középpontos tükrözés után a végeredmény egy olyan függvény
lett, ahol az x2 parabola bal ágát ,,lecsavartuk”.♦

Példa 4 :
|x| − 1
Ábrázoljuk az f (x) = függvényt.
|x| − 2

Ez szintén páros függvény, mert

|x| − 1 |−x| − 1
=
|x| − 2 |−x| − 2

Megint csak az x ≥ 0 esetet nézzük.


|x| − 1 x−1
Ekkor = , mert az abszolút érték feleslegessé válik. Ezt pedig tovább lehet
|x| − 2 x−2
1
alakı́tani, ı́gy az ábrázolható alakja 1 + .
x−2

21
2. fejezet

 
1
Amikor ezt tükrözzük az y-tengelyre, figyeljünk oda, hogy a függvény a 0; pontjában
2
megtörik, nem folyamatos görbe, illetve, hogy az aszimptotákat is tükrözzük. ♦

2.4. Egyenletek és egyenlőtlenségek grafikus megoldása


Ha egy egyenlettel vagy egyenlőtlenséggel nem boldogulunk, mindig jó ötlet a két oldalon
található függvényt ábrázolni. Az egyenlet megoldásai a függvények metszéspontjai lesznek.
Egyenlőtlenségeknél pedig levetı́tjük a metszéspontokat az x-tengelyre, és megállapı́tjuk a he-
lyes intervallumokat. Ha egy függvény a másik függvény felett van, akkor nagyobb nála, ha
pedig alatta van, akkor kisebb nála.

Példa 1 : √
Oldjuk meg a valós számok halmazán: x + x ≥ 0


x ≥ −x


Látható, hogy x ≥ −x a teljes értelmezési tartományon igaz.
A végeredmény x ∈ [0; ∞[. ♦

22
2. fejezet

Példa 2 : √
Oldjuk meg a valós számok halmazán: x + 3 > x + 1

A bal oldal nagyobb, mint a jobb oldal, vagyis azt a részt nézzük, ahol a bal oldal grafikonja
magasabban van, mint a jobb oldalé. Egy metszéspont van. Ezt illetve a négyzetgyök kikötését
figyelembe véve a végeredmény x ∈ [−3; 1[. ♦

Példa 3 :
1
Oldjuk meg a valós számok halmazán: −|x + 1| + 4 ≥ − x + 2
2
A bal oldal megint nagyobb, mint a jobb oldal, vagyis a bal oldal grafikonja magasabban van,
mint a jobb oldalé. A metszéspontokat levetı́tve megkapjuk azt az intervallumot, amelyiken ez
igaz.
A végeredmény x ∈ [−2; 2].♦

23
2. fejezet

Példa 4 :
Oldjuk meg a valós számok halmazán: cos x = x2 + 1

Az egyetlen metszéspont tisztán leolvasható.


A végeredmény x = 0.♦

Példa 5 :
2
Oldjuk meg a valós számok halmazán: + 1 ≤ − (x − 3)2 + 3
x−2
A jobb oldal grafikonjának magasabban kell lennie. A metszéspontok szerencsére leolvasható
egész számok. Figyelni kell ebben a feladatban a hiperbola kikötésére is.
A végeredmény x ∈ [1; 2[ ∪ [3; 4].♦

24
2. fejezet

Példa 6 :
Oldjuk meg a valós számok halmazán: 2x ≤ −2 (x − 2)2 + 4

Az exponenciális függvénynek a parabola alatt kell lennie, ı́gy ezt a részt kell figyelembe venni
a metszéspontok levetı́tése után.
A végeredmény x ∈ [1; 2]. ♦

Példa 7 :
Oldjuk meg a valós számok halmazán: |2x − 1| < 5

A metszéspontok levetı́tése után látható, hogy az abszolút érték függvény hol van a konstans
függvény alatt.
A végeredmény x ∈ ]−2; 3[. ♦

25
2. fejezet

2.5. A teljes négyzetté alakı́tás


A másodfokú függvény polinom alakban nem ábrázolható a transzformációkkal. Ezt az ún. tel-
jes négyzetté alakı́tással olyan alakra hozzuk, ami viszont már ábrázolható. A teljes négyzetté
alakı́tás több helyen is előforduló gondolat.
A közismert nevezetes azonosságokon múlik az egész eljárás.
Vegyük példának azt, hogy

x2 − 4x + 4 = (x − 2)2

Ha ezt átrendezzük, akkor

x2 − 4x = (x − 2)2 − 4

A bal oldalon ı́gy egy hiányos másodfokú kifejezés van. Ekkor az elsőfokú tag együtthatóját
megfelezzük, ı́gy kapjuk meg a jobb oldalon a zárójelben lévő számot. Amit pedig levonunk a
végén, nem más, mint ennek a zárójelben lévő számnak a négyzete.

x2 − 4x = (x − 2)2 − 4

Ha nekünk adott az x2 +6x+11 kifejezés, akkor ezt ugyanı́gy kell elkezdeni, azzal a különbséggel,
hogy a végén lévő +11-et csak le kell másolni, majd a végén összeadni.

x2 + 6x + 11 = (x + 3)2 − 9 + 11 = (x + 3)2 + 2

Példa 1 :
Alakı́tsuk teljes négyzetté a következő másodfokú kifejezéseket:
x2 − 4x + 5; x2 + 6x + 7; x2 − 2x + 2; x2 + 7x + 14

x2 − 4x + 5 = (x − 2)2 − 4 + 5 = (x − 2)2 + 1
x2 + 6x + 7 = (x + 3)2 − 9 + 7 = (x + 3)2 − 2
x2 − 2x + 2 = (x − 1)2 − 1 + 2 = (x − 1)2 + 1
x2 + 7x + 14 = (x + 3,5)2 − 12,25 + 14 = (x + 3,5)2 + 1,75♦

Abban az esetben, ha az x2 -nek egytől különböző együtthatója van, az eljárás csak azzal bővül,
hogy ezt az együtthatót a legelején a másodfokú és az elsőfokú tagból ki kell emelni. Utána
ugyanazt csináljuk, mint eddig, majd a végén pedig visszaszorzunk vele.

3x2 + 12x + 10 = 3 x2 + 4x + 10 = 3 (x + 2)2 − 4 + 10 = 3(x + 2)2 − 12 + 10 = 3(x + 2)2 − 2


   

Példa 2 :
Alakı́tsuk teljes négyzetté a következő másodfokú kifejezéseket:
2x2 − 4x + 4; 3x2 + 18x + 6; −2x2 + 12x − 9; −x2 − 4x + 10

26
2. fejezet

= 2 (x − 1)2 − 1 + 4 2 (x − 1)2 + 2


 
2x2 − 4x + 4 = 2 [x2 − 2x] + 4 =
3x2 + 18x + 6 = 3 [x2 + 6x] + 6 = 3 (x + 3)2 − 9 + 6 = 3 (x + 3)2 − 21
−2x2 + 12x − 9 = −2 [x2 − 6x] − 9 = −2 (x − 3)2 − 9 − 9 = −2 (x − 3)2 + 9
−x2 − 4x + 10 = − [x2 + 4x] + 10 = − (x + 2)2 − 4 + 4 = − (x + 2)2 + 14♦

2.6. |x + 1| + |x − 3| tı́pusú függvények


Amikor abszolút értékek összege vagy különbsége szerepel egy függvényben, táblázatba foglalva
érdemes esetekre bontani.

Példa 1 :
Ábrázoljuk az f (x) = |x + 1| + |x − 3| függvényt.

Első lépésként megállapı́tjuk, hogy az abszolút értékben lévő kifejezések hol veszik fel a nulla
értéket: x = −1 és x = 3 esetén. Ezután egy táblázatot készı́tünk. Az oszlopokban a valós
számegyenest bontjuk fel az előbb kiszámolt zérushelyek szerint, növekvő sorrendben. A sorok
a függvényben szereplő kifejezések, legalul pedig az egész függvény található. A táblázatot úgy
kell kitölteni, hogy mindegyik intervallumból veszünk egy számot, azt behelyettesı́tjük az adott
sorba, és megnézzük (abszolút érték nélkül) pozitı́v, vagy negatı́v számot kapunk-e. Ha pozitı́v,
akkor elhagyjuk az abszolút értéket, ha negatı́v, akkor vesszük az ellentettjét.

x ≤ −1 −1 < x < 3 3≤x


|x + 1| −(x + 1) x+1 x+1
|x − 3| −(x − 3) −(x − 3) x−3
|x + 1| + |x − 3| −2x + 2 4 2x − 2

Az oszlopok szerint egyesével három különálló függvényt kell ábrázolni, a fent megadott inter-
vallumokon. Ha mindent jól csinálunk, ezeknek a daraboknak össze kell érniük és egy ,,lavór”
formát alkotnak.♦

27
2. fejezet

Példa 2 :
Ábrázoljuk az f (x) = |2x − 3| − |4x − 3| függvényt.
3 3
Most is megállapı́tjuk, hogy az abszolút értékben lévő kifejezések hol nullák. x = és x =
2 4
esetén.

3 3 3 3
x≤ <x< ≤x
4 4 2 2
|2x − 3| −(2x − 3) −(2x − 3) 2x − 3
|4x − 3| −(4x − 3) 4x − 3 4x − 3
|2x − 3| − |4x − 3| 2x −6x − 6 −2x

A függvényeknek csak az a darabja kell, ami az oszlopban szereplő intervallumban van, a többi
rész elhagyható. A daraboknak megint össze kell érniük és ebben az esetben egy ,,hegycsúcs”
alakú függvényt alkotnak.♦

2.7. Függvények kompozı́ciója


Függvények kompozı́ciója esetén fontos tudni, hogy melyik függvény számı́t belsőnek és melyik
külsőnek. Ennek gyakorlati jelentősége a lánc-szabálynál van. (12.1. fejezet)

(f ◦ g) (x) esetén az f függvény változójának helyére beillesztjük a teljes g függvényt. A külső


f függvény értelmezési tartományának a belső g függvény értékkészletével vett metszete nem
lehet üres, különben a kompozı́ció nem alkotható meg. Df ∩ Rg ̸= ∅.
(g ◦ f ) (x) esetén pont fordı́tva, a g függvény változójának a helyére illesztjük be az egész f
függvényt, és ilyenkor Dg ∩ Rf ̸= ∅ a feltétel.

28
2. fejezet

Példa 1 : √
Adott f (x) = log2 x és g (x) = x. Adjuk meg (f ◦ g) (x) és (g ◦ f ) (x) függvényeket.


(f ◦ g) (x) = plog2 x
(g ◦ f ) (x) = log2 x♦

Példa 2 :
1
Adott f (x) = és g (x) = 2x+1 . Adjuk meg (f ◦ g) (x) és (g ◦ f ) (x) függvényeket.
x−3

1
(f ◦ g) (x) =
2x+1
1
−3
(g ◦ f ) (x) = 2 x−3 +1 ♦

Példa 3 :
Adott f (x) = x2 + 4x + 3 és g (x) = sin x. Adjuk meg (f ◦ g) (x) és (g ◦ f ) (x) függvényeket.

(f ◦ g) (x) = sin2 x + 4 sin x + 3


(g ◦ f ) (x) = sin x2 + 4x + 3♦

Példa 4 :
Adott f (x) = log2 x és g (x) = −x2 . Adjuk meg (f ◦ g) (x) és (g ◦ f ) (x) függvényeket.

(f ◦ g) (x) = log2 (−x2 )


(g ◦ f ) (x) = −log2 2 x
Jelenleg viszont (f ◦ g) (x) nem létezik. Df =]0; ∞[ és Rg =] − ∞; 0]. Metszetük üres. ♦

2.8. A paraméteres egyenletek grafikus megoldása


A paraméteres egyenleteket (leginkább akkor, ha a paraméter egyedül van az egyenlőség egyik
oldalán) néha grafikusan érdemes megoldani, mert a másik oldalon lévő függvénnyel való
metszéspontjait (vagyis az egyenlet megoldásait) könnyű lehet leolvasni.

Példa 1 :
Hogyan függ a p paramétertől az x2 + 4x + 3 = p egyenlet megoldásainak száma a [−4; 1[
intervallumon?

Először is figyeljünk a kérdésre. Az a kérdés, hogy hány megoldás van, és nem az, hogy
mik a megoldások. Ez sugallhatja azt, hogy próbálkozzunk grafikus megoldással, hiszen a
metszéspontok számát mindig könnyebb leolvasni, mint azt, hogy hol vannak a metszéspontok,
hiszen egyáltalán nem garantált, hogy azok szép négyzetrácspontokra esnek és leolvashatók.
Az egyenlőség bal oldalát már könnyen tudjuk ábrázolni.

x2 + 4x + 3 = (x + 2)2 − 1

29
2. fejezet

A jobb oldal pedig egy konstans ,,egyenessereg”. A p azt mutatja meg, hol metszi az y-tengelyt.
Mivel p valós szám, ezért minden lehetőséget végig kell néznünk, tehát végigpásztázzuk a teljes
y-tengelyt, párhuzamos eltolással.

Az első ábrán jól látszik, hogy ha p < 0 (szı́nes rész), akkor az egyenes nem ér hozzá a
függvényhez, ı́gy nincs megoldás. Ha viszont p = 0 (vastag egyenes), akkor két metszéspont
van.

Növeljük p-t. A második ábrán az látszik, hogy ha 0 < p < 1, akkor négy metszéspont van, de
ha p = 1, akkor már csak három, hiszen a függvény érintője lesz.

30
2. fejezet

Megyünk tovább. Ha 1 < p ≤ 3, akkor két metszéspont van, a tömör karika miatt p = 3-nál is.

Ha viszont 3 < p < 8, akkor csak egy megoldás van.

31
2. fejezet

Végezetül ha 8 ≤ p, megint nincs megoldás, p = 8 esetén sem, az üres karika miatt.

A végeredmény összefoglalása lehet a megoldások száma, vagy a p értéke szerint is.

Nincs megoldás, ha p < 0, vagy 8 ≤ p. Ha p < 0 ⇒ nincs megoldás.


Egy megoldás van, ha 3 < p < 8. Ha p = 0 ⇒ két megoldás van.
Két megoldás van, ha p = 0, vagy 1 < p ≤ 3. Ha 0 < p < 1 ⇒ négy megoldás van.
Három megoldás van, ha p = 1. Ha p = 1 ⇒ három megoldás van.
Négy megoldás van, ha 0 < p < 1. Ha 1 < p ≤ 3 ⇒ két megoldás van.
Ha 3 < p < 8 ⇒ egy megoldás van.
Ha 8 ≤ p ⇒ nincs megoldás.

Ezzel a módszerrel az összes lehetőséget végignéztük.♦

Példa 2 :
Határozzuk meg a p paraméter értékét úgy, hogy az |x − 2| + x − 1 = px egyenletnek pontosan
egy megoldása legyen.

A bal oldal ábrázolása esetszétválasztást igényelne, ami hosszadalmas, ı́gy átalakı́tjuk az egyen-
letet.

|x − 2| = px − x + 1
|x − 2| = (p − 1)x + 1

Ezt már egyszerűbb ábrázolni. A bal oldal egy egyszerű abszolút értékes függvény, a jobb oldal
pedig egy a (0; 1) ponton áthaladó, p − 1 meredekségű egyenes. Itt érdemes a meredekséget
elkezdeni −∞-től, és haladni a +∞ felé.

32
2. fejezet

Ha a meredekség < −1, akkor egy metszéspont van.

1
Ha −1 ≤ meredekség < − , akkor nincs metszéspont. (−1 meredekség esetén az egyenes
2
párhuzamos a függvénnyel.)

1
Ha a meredekség = − , akkor megint egy metszéspont van.
2

33
2. fejezet

1
Ha − ≤ meredekség < 1, akkor két metszéspont van.
2

Végezetül, ha 1 ≤ meredekség, akkor megint csak egy metszéspont van.

Mivel a feladat az volt, hogy milyen p esetén lesz pontosan egy megoldása az egyenletnek, ezért
1
akkor, ha a meredekség p − 1 < −1, vagy p − 1 = − , vagy 1 ≤ p − 1.
  2
1
A végeredmény p ∈ ]−∞; 0[ ∪ ∪ [2; ∞[.♦
2

2.9. Racionális törtfüggvény egyszerűsı́tése és a folytonosság


polinom
Ha egy függvény alakú, akkor elképzelhető, hogy lehet egyszerűsı́teni, de figyeljünk ar-
polinom
ra, hogy ezzel a függvény értelmezési tartományát bővı́thetjük. Más szóval kifejezve, betömjük
a függvénynek a lyukait, megszüntethető szakadási helyeit.

Példa 1 :
x2 + x − 2
Ábrázoljuk az f (x) = függvényt.
x−1

34
2. fejezet

Első ránézésre nem tűnik könnyűnek, de a függvény algebrailag egyszerűsı́thető.

x2 + x − 2 (x − 1) (x + 2)
f (x) = = =x+2
x−1 x−1

Legyen ekkor g (x) = x + 2.


Ez azt jelenti, hogy f (x) és g (x) grafikonja ugyanúgy néz ki, azonban van egy különbség.
f (x) a nevező miatt x = 1-ben nem értelmezhető, viszont g (x) már igen.

A tört leegyszerűsı́tése ,,betömi a lyukat” a függvényen, vagyis folytonossá válik a függvény.♦

Példa 2 :
x3 − 3x2 − 4x
Ábrázoljuk az f (x) = függvényt.
x2 + x
Hasonló módon egyszerűsı́tjük a törtet.

x3 − 3x2 − 4x x (x2 − 3x − 4) x (x + 1) (x − 4)
f (x) = 2
= = =x−4
x +x x (x + 1) x (x + 1)

Legyen most g (x) = x − 4.


Most f (x) sem x = 0-ban, sem x = −1-ben nem értelmezhető, tehát mindkettő helyen van
rajta egy-egy ,,lyuk”, de ezt leszámı́tva a grafikonja megegyezik g (x) grafikonjával.

Ez a gondolatmenet megkönnyı́ti az ábrázolást, de fontos a függvények határértékénél is.♦

35
2. fejezet

2.10. Függvények határértéke


A témakör részletesebben az elsőéves egyetemi tananyagban szokott előfordulni. Középiskolában
a függvények határértékéről elég azt tudni, hogy szoros kapcsolatban áll a függvény folyto-
nosságával. Úgy is fogalmazhatunk, hogy egy függvény folytonos az a pontban, ha ott nem
emeljük fel a ceruzát rajzolás közben. (Pl. az első rajzon ez látható.) Ha egy függvény a-ban
folytonos, akkor a-ban értelmezhető és a határérték egyenlő a helyettesı́tési értékkel.

lim f (x) = f (a)


a

Ha egy függvény a-ban nem folytonos, vagyis valami kikötés miatt nem ı́rhatjuk bele a függvény-
be, akkor ott megszüntethető szakadási helye (második rajz) vagy meg nem szüntethető sza-
kadási helye (harmadik rajz) van. Ha megszüntethetővel találkozunk, akkor a 2.9. fejezetben
látható leegyszerűsı́téshez folyamodhatunk. A meg nem szüntethetővel jelenleg nem foglalko-
zunk.

Példa 1 :
Számoljuk ki lim (4x3 − 6x + 3) határértéket.
1

A függvénynek nincs kikötése, ezért 1-ben folytonos.

lim 4x3 − 6x + 3 = 4 · 13 − 6 · 1 + 3 = 1♦

1

Példa 2 :
x2 − 34
Számoljuk ki lim határértéket.
10 x−4

36
2. fejezet

A függvénynek nincs kikötése, ezért 10-ben folytonos.

x2 − 34 102 − 34
lim = = 11♦
10 x−4 10 − 4

Példa 3 :
x2 − 4
Számoljuk ki lim határértéket.
2 x−2
Ez a függvény 2-ben nem folytonos, mert a nevező 0-hoz tart.

x2 − 4 (x − 2) (x + 2)
lim = lim = lim (x + 2) = 2 + 2 = 4♦
2 x−2 2 x−2 2

Példa 4 :
x2 − 4x + 3
Számoljuk ki lim határértéket.
3 x2 − 5x + 6
Ez a függvény 3-ban nem folytonos, mert a nevező 0-hoz tart.

x2 − 4x + 3 (x − 1) (x − 3) x−1 3−1
lim 2 = lim = lim = = 2♦
3 x − 5x + 6 3 (x − 2) (x − 3) 3 x−2 3−2

Példa 5 : √
1+x+1
Számoljuk ki lim határértéket.
0 x
Ez a függvény 0-ban nem folytonos, mert a nevező 0-hoz tart.

√ √  √ 
1+x+1 1+x−1 1+x+1
lim = lim √  =
0 x 0 x 1+x+1
x 1 1 1
= lim √  = lim √ =√ = ♦
0 x 1+x+1 0 1+x+1 1+0+1 2

2.11. Függvény monotonitásának következményei


Ebben a részben az egyik, első ránézésre nem túl nyilvánvaló következményről lesz szó, ami-
nek jelentősége algebrailag mutatkozik meg leginkább. Ha egy függvény szigorúan monoton nő,
akkor az [a; b] intervallumban szereplő számok helyettesı́tési értéke az [f (a); f (b)] intervallum-
ban lesz benne. Másképp fogalmazva, minden [a; b] intervallumba eső x szám esetén, f (x) az
[f (a); f (b)] intervallumba fog esni. Szigorúan monoton csökkenő függvények esetén helyet cserél
az intervallum két végpontja, ezért f (x) az [f (b); f (a)] intervallumba fog beleesni. A következő
néhány elemi függvénynél egy szı́nes téglalap jelöli ezt a kapcsolatot. A téglalapok vı́zszintes
oldalai az értelmezési tartomány egy szeletét jelölik, mı́g a függőleges oldalai az értékkészlet
egy részét:

37
2. fejezet


f (x) = x

√ √ √ 
f (x) = x végig szigorúan monoton nő. Ha x ∈ [4; 9], akkor f (x) ∈ 4; 9 = [2; 3].

1
f (x) =
x

   
1 1 1 1
f (x) = végig szigorúan monoton csökken. Ha x ∈ [1; 2], akkor f (x) ∈ ; = ;1 .
x 1 2 2

f (x) = log2 x

f (x) = log2 x végig szigorúan monoton nő. Ha x ∈ [2; 8], akkor f (x) ∈ [log2 2; log2 8] = [1; 3].

38
2. fejezet

f (x) = 2x

 
1
f (x) = 2x végig szigorúan monoton nő. Ha x ∈ [−1; 3], akkor f (x) ∈ [2−1 ; 23 ] = ; 8 .
2

Nézzünk most két másik függvényt, amelyek nem a teljes értelmezési tartományon szigorúan
monotonak:

f (x) = |x|

(A szı́nes téglalapokra vonatkozó megfelelő hasáb alatta szerepel.)

39
2. fejezet

Piros téglalap: Zöld téglalap: Kék téglalap:


x ∈ [−5; −3] ⇒ f (x) ∈ [3; 5] x ∈ [−1; 3] ⇒ f (x) ∈ [0; 3] x ∈ [4; 7] ⇒ f (x) ∈ [4; 7]
|x| ezen az intervallumon |x| ezen az intervallumon |x| ezen az intervallumon
szigorúan monoton. NEM szigorúan monoton. szigorúan monoton nő.

f (x) = x2

(A szı́nes téglalapokra vonatkozó megfelelő hasáb alatta szerepel.)

Piros téglalap: Zöld téglalap: Kék téglalap:


x ∈ [−3; −2] ⇒ f (x) ∈ [4; 9] x ∈ [−1; 2] ⇒ f (x) ∈ [0; 4] x ∈ [2; 3] ⇒ f (x) ∈ [4; 9]
x2 ezen az intervallumon x2 ezen az intervallumon NEM x2 ezen az intervallumon
szigorúan monoton csökken. szigorúan monoton. szigorúan monoton nő.

Amikor nem monoton függvénydarabról van szó, az adott értelmezési tartományba eső értékek
helyettesı́tési értékeinek kiszámı́tásánál érdemes tisztában lenni a függvény grafikonjának
alakjával. Segı́thet a meghatározásban.

Összetett függvények értékkészleteit vizsgáljuk a következő feladatokban. A belső függvények


értékkészleteinek kiszámolása után ezekkel az intervallumokkal, mint a külső függvények
értelmezési tartományaival dolgozunk tovább.

Példa 1 :
Milyen intervallumban helyezkednek el 2sin x értékei?

sin x ∈ [−1; x
 1],és mivel a 2 végig szigorúan monoton nő, ezért 2
sin x
∈ [2−1 ; 21 ]. Az értékek
1
elemei az ; 2 intervallumnak.♦
2

40
2. fejezet

Példa 2 :
Milyen intervallumban helyezkednek el cos2 x értékei?

cos x ∈ [−1; 1] de ezen az intervallumon az x2 függvény nem végig monoton, ı́gy figyelni kell
arra, hogy most a négyzetre emelés után cos2 x ∈ [0; 1].♦

Példa 3 :
Milyen intervallumban helyezkednek el (cos x + 3)2 értékei?

cos x ∈ [−1; 1]. Hármat hozzáadva cos x + 3 ∈ [2; 4]. Az előző feladattal ellentétben ezen az
intervallumon az x2 függvény végig monoton nő, ı́gy (cos x + 3)2 ∈ [4; 16].♦

Példa 4 :  
2 1
Milyen intervallumban helyezkednek el log3 x értékei, ha x ∈ ;1 ?
27
   
1 1
Ha x ∈ ; 1 , akkor log3 x ∈ log3 ; log3 1 vagyis [−3; 0], mert a log3 x függvény szi-
27 27
gorúan monoton nő. Az x2 függvény pedig ezen az intervallumon szigorúan monoton csökken,
ı́gy log3 2 x ∈ [(−3)2 ; 0], vagyis az értékek elemei a [0; 9] intervallumnak.♦

Példa 5 :
Milyen intervallumban helyezkednek el a log2 (x2 ) értékei, ha x ∈ [−4; 1]?

Ha x ∈ [−4; 1], akkor x2 ∈ [1; 16], mert x2 szigorúan monoton csökken ezen az intervallumon.
A logaritmus függvény pedig szigorúan monoton nő, ı́gy log2 x2 ∈ [log2 1; log2 16]. Az értékek
tehát a [0; 4] intervallumnak elemei.♦

41
3. fejezet

3. Algebra
3.1. Az értelmezési tartomány és az értékkészlet fogalma
A két fogalom nemcsak a függvényeknél fontos, hanem az algebrai kifejezéseknél is. Sokkal
nagyobb jelentősége van emelt szinten, mint középszinten, ı́gy ezeket részletesen átbeszéljük. A
kettő közti különbséget a következő szemléltető rajzzal magyarázzuk:

Adott egy daráló. Ebbe a darálóba beledobunk számokat, megdaráljuk őket, és alul kipottyan-
nak számok. Nevezzük el ezt a darálót ( )2 -nek. Nyilvánvaló, hogy bele bármilyen számot lehet
dobni, hiszen mindegyik számot négyzetre tudjuk emelni. Ki viszont csak pozitı́v szám, vagy
nulla jöhet, mert minden valós szám négyzete pozitı́v, vagy nulla. Ha ez a művelet mondjuk
a log2 ( ), akkor itt pedig bele csak pozitı́v számokat dobhatunk, de kifelé bármilyen számot
kaphatunk. Magyarul, bizonyos műveletek megszorı́tásokat igényelnek a bedobott, vagy a ki-
pottyanó számokra, nem lehet akármilyen számot választani. Azt a legbővebb számhalmazt,
amit bele tudunk dobni a darálóba, vagyis bele tudunk tenni a műveletbe, értelmezési tar-
tománynak hı́vjuk. Azt a számhalmazt, ami kipottyan alul, tehát előáll eredményként, érték-
készletnek hı́vjuk. Az alábbiakban nézzük végig a listát, hogy az ismert műveletek milyen
megszorı́tásokat, kikötéseket követelnek.

Értelmezési tartományra vonatkozó kikötések Értékkészletre vonatkozó kikötések


A tört nevezője ̸= 0 x+1
|x| ≥ 0
loga b : a > 0, a ̸= 1, b > 0 x2 ≥ 0
π
tg x : x ̸= + kπ xpáros ≥ 0
2 √
ctg x : x ̸= 0 + kπ x√≥ 0

páros
x≥0
x√: x ≥ 0 −1 ≤ sin x ≤ 1
páros
x: x ≥ 0 −1 ≤ cos x ≤ 1

Példa 1 :
Határozzuk meg a következő
√ kifejezések
√ értékkészletét: √
x
2 − 4; 3 sin x + 1; 4
x + 1; 4
x + 1; 2 cos x
; |x − 7| + 4; x + 4 ; 3(tg x + 4)2 − 4; sin2 x

2x > 0, ha ezt letoljuk 4-gyel, 2x − 4 > −4


Az értékkészlet ]−4; ∞[.

−1 ≤ sin x ≤ 1, ha ezt megszorozzuk 3-mal,−3 ≤ 3 sin x ≤ 3, ha ezt feltoljuk 1-gyel,


−2 ≤ 3 sin x + 1 ≤ 4
Az értékkészlet [−2; 4].

42
3. fejezet

√ √
4
x ≥ 0, ha hozzáadunk 1-et, 4
x+1≥1
Az értékkészlet [1; ∞[.

4
x+1≥0
Az értékkészlet [0; ∞[.

−1 ≤ cos x ≤ 1, ha mindent


 felemelünk 2 kitevőjére, akkor 2−1 ≤ 2cos x ≤ 21 .(2.11. fejezet)
1
Az értékkészlet ;2 .
2

|x − 7| ≥ 0, ha hozzáadunk 4-et, |x − 7| + 4 ≥ 4
Az értékkészlet [4; ∞[.

A következő két feladat jól illusztrálja, hogy csak azért, mert abszolút értéket / négyzetre
emelést látunk, még nem kell rögtön azt mondani, hogy akkor az nagyobb, vagy egyenlő, mint
nulla. Előfordulhat, hogy az abszolút értékben / zárójelben, már eleve olyan kifejezés található,
aminek van kikötése, tehát lehet, hogy miután abszolút értékbe tesszük / négyzetre emeljük,
nem ,,csökken le” nulláig. Ezért fontos az alapos elemzés.
√ √
x ≥ 0, ha hozzáadunk 4-et, x + 4 ≥√4, vagyis biztosan pozitı́v. Ha abszolút értékbe teszünk
pozitı́v számot, nem változik semmi. x + 4 ≥ 4
Az értékkészlet [4; ∞[.

tg x ∈ R, ha hozzáadunk 4-et és négyzetre emeljük, akkor (tg x + 4)2 ≥ 0 . Ha megszorozzuk


3-mal, nem változik semmi. Ha kivonunk 4-et, 3(tg x + 4)2 − 4 ≥ −4
Az értékkészlet [−4; ∞[.

−1 ≤ sin x ≤ 1, ha négyzetre emeljük, 0 ≤ sin2 x ≤ 1 (2.11. fejezet)


Az értékkészlet [0; 1].♦

Példa 2 :
Határozzuk meg a valós számok mely legbővebb részhalmazán értelmezhetők a következő kife-
3 √ √ √ √ 2 p
jezések: lg (2x − 4); 2 ; 5−x + x − 4; 3x − 9; x − 3 ; (x − 3)2 ; log2 |x|;
√ x − 9
 π p √
x2 − 1; tg 3x + ; x+ x
3
lg (2x − 4), vagyis 2x − 4 > 0 ⇒ x > 2
Az értelmezési tartomány ]2; ∞[.
3
, tehát x2 − 9 ̸= 0 ⇒ x ̸= ±3
x2 −9
Az értelmezési tartomány R \ {−3; 3}.
√ √
5 − x + x − 4, vagyis 5 − x ≥ 0 és x − 4 ≥ 0 ⇒ 5 ≥ x és x ≥ 4
Az értelmezési tartomány [4; 5].
√ x
3 − 9, azaz 3x − 9 ≥ 0 ⇒ x ≥ 2
Az értelmezési tartomány [2; ∞[.
√ 2
x − 3 , vagyis x − 3 ≥ 0 ⇒ x ≥ 3
Az értelmezési tartomány [3; ∞[.

43
3. fejezet

p
(x − 3)2 = x − 3 , ı́gy nincs kikötés.
Az értelmezési tartomány R.

log2 |x|, szóval |x| > 0. Ez a nullán kı́vül minden számra igaz.
Az értelmezési tartomány R \ {0}.

x2 − 1, tehát x2 − 1 ≥ 0 ⇒ x2 ≥ 1 ⇒ x ≥ 1 vagy x ≤ −1
Az értelmezési tartomány ] − ∞; −1] ∪ [1; ∞[.
 π π π π π
tg 3x + , tehát 3x + ̸= + kπ ⇒ x ̸= + k , ahol k ∈ Z
3 3 2n 18 3
π πo
Az értelmezési tartomány R \ +k .
18 3
p √ √ √
x + x, szóval x ≥ 0 és x + x ≥ 0 ⇒ x ≥ −x, ez pedig mindig igaz, mert a bal oldal
egy nemnegatı́v szám, és az mindig nagyobb, mint a jobb oldal, egy nempozitı́v szám.
Az értelmezési tartomány [0; ∞[.♦

3.2. Polinomok szorzattá alakı́tása


Amikor szorzattá alakı́tunk egy algebrai kifejezést, akkor van egy rögzı́tett sorrend:
I. kiemelés
II. kiemelés csoportosı́tással
III. nevezetes azonosságok
IV. gyöktényezős alak
Ha az egyik módszer nem alkalmazható, ugrunk a következőre, de ha alkalmazható, akkor
muszáj megtenni, nem szabad átlépni.

Példa 1 :
Alakı́tsuk szorzattá a következő algebrai kifejezéseket: a3 − 4a; 5a2 − 20b2 ; 3xy 2 + 6xy + 3x;
a4 − 8a + 6a3 − 12a2 ; 2a3 − 4a2 − 30a kifejezéseket.

I. III.
a3 − 4a = a(a2 − 4) = a(a − 2)(a + 2)

I. III.
5a2 − 20b2 = 5(a2 − 4b2 ) = 5(a − 2b)(a + 2b)

I. III.
3xy 2 + 6xy + 3x = 3x(y 2 + 2y + 1) = 3x(y + 1)2

I.  II.
a4 − 8a + 6a3 − 12a2 = a a3 − 8 + 6a2 − 12a =

 II.
= a (a − 2)(a2 + 2a + 4) + 6a(a − 2) = a(a − 2)(a2 + 8a + 4)


I. IV.
2a3 − 4a2 − 30a = 2a(a2 − 2a − 15) = 2a(a − 5)(a + 3)♦

44
3. fejezet

Példa 2 :
3a2 − 3
Egyszerűsı́tsük a törtet.
6a2 + 6a − 12

3a2 − 3 I. 3(a2 − 1) III./IV. 3(a − 1)(a + 1) a+1


= = = ♦
6a2 + 6a − 12 6(a2 + a − 2) 6(a − 1)(a + 2) 2(a + 2)

3.3. Ekvivalens és nem ekvivalens átalakı́tások


Bizonyos algebrai műveletek kölcsönösen egyértelműek, ekvivalensek, oda-vissza kölcsönösen
átalakı́thatók. Más műveletek azonban nem kölcsönösen egyértelműek, vagy csak feltétellel
azok. Fontos tisztában lenni azzal, hogy melyik művelet számı́t ekvivalensnek és melyik nem,
mert ha egy nem ekvivalens átalakı́tást végzünk egy egyenletben, akkor bejöhetnek hamis meg-
oldások, illetve elveszthetünk jó megoldásokat, tehát az ellenőrzés elkerülhetetlen. De ezeket
a lépéseket nemcsak egyenletek levezetésénél használhatjuk, hanem akkor is lehet őket alkal-
mazni, ha meg kell mutatni két számról / algebrai kifejezésről, hogy egyenlők, vagy akkor, ha
egyszerűbb alakra kell hozni egy számot / algebrai kifejezést. Ilyen esetekben nem közvetlenül
a két számról / algebrai kifejezésről mutatjuk meg, hogy egyenlők, de ez nem probléma.

ˆ Logaritmus: x = y ⇔ loga x = loga y, ha x; y > 0, a > 0, a ̸= 1

ˆ Exponenciális: x = y ⇔ ax = ay , ha a > 0; a ̸= 1

ˆ Összeadás (számmal): x = y ⇔ x + 2 = y + 2

ˆ Kivonás (számmal): x = y ⇔ x − 5 = y − 5
x y
ˆ Osztás (számmal): x = y ⇔ =
4 4
ˆ Szorzás (számmal): x = y ⇔ 3x = 3y

ˆ Páratlan kitevőre emelés: x = y ⇔ x2k+1 = y 2k+1

ˆ Négyzetre emelés: x = y ⇔ x2 = y 2 csak akkor, ha mindkét szám előjele megegyezik.

ˆ Páros kitevőre emelés: x = y ⇔ x2k = y 2k csak akkor, ha mindkét szám előjele megegye-
zik.

ˆ Szinusz: x = y ⇒ sin x = sin y, de sin x = sin y ⇒ x = y +2kπ vagy x = (π − y)+2kπ

ˆ Koszinusz: x = y ⇒ cos x = cos y, de cos x = cos y ⇒ x = y+2kπ vagy x = (2π − y)+


+ 2kπ

Példa 1 :
Bizonyı́tsuk be, hogy ha a, b > 0, akkor alg b = blg a .

Vesszük mindkét oldal logaritmusát. Ezt megtehetjük, hiszen mindkét oldal pozitı́v szám.

lg alg b = lg blg a
lg b lg a = lg a lg b
Azonosságot kaptunk.♦

45
3. fejezet

Példa 2 :
lg a + lg b a+b
Bizonyı́tsuk be, hogy ha a, b > 0, akkor ≤ lg .
2 2

lg a + lg b a+b
≤ lg
2 2
a+b
lg ab ≤ 2 lg
 2 2
a+b
lg ab ≤ lg
2

Ekkor elhagyjuk mindkét oldalról a logaritmust, és mivel tı́zes alapú, nem változik meg a relációs
jel iránya.
 2
a+b
ab ≤
2
a + 2ab + b2
2
ab ≤
4
4ab ≤ a2 + 2ab + b2
0 ≤ a2 − 2ab + b2
0 ≤ (a − b)2
Erről pedig tudjuk, hogy igaz. ♦

Példa 3 : r √ r √
a+ a2 − b a− a2 − b p √
Bizonyı́tsuk be, hogy ha a; b > 0 és a2 ≥ b akkor + = a + b.
2 2
Mindkét oldalt négyzetre emeljük. Ezt szabad, mert mindkét oldal értéke nemnegatı́v.
r √ r √ !2
a + a2 − b a − a2 − b p √ 2
+ = a+ b
2 2
√ r √ r √ √
a + a2 − b a + a2 − b a − a2 − b a − a2 − b √
+2 · + = a+ b
2 2 2 2
r √ √
a + a2 − b a − a2 − b √
a+2 · = a+ b
2 r 2
a2 − (a2 − b) √
a+2 = a+ b
4 r
b √
a+2 = a+ b
4
√ √
a+ b = a+ b♦

Példa 4 : lg (lg a)
Hozzuk egyszerűbb alakra az a lg a = lg a kifejezést.

Figyelem! Most nem egy egyenlőséget kell bizonyı́tanunk, vagyis nem lehet azt mondani, ha
vesszük a logaritmusát, leegyszerűsı́tjük a kifejezést, akkor készen is vagyunk. Most egyszerűbb
alakra kell hozni, vagyis ha vesszük a logaritmusát, akkor később ,,vissza kell venni a loga-
ritmust”, azaz megcsinálni az inverz műveletet, különben nem kapunk az eredeti kifejezéssel
egyenlő értékű kifejezést. Tehát vesszük a logaritmusát.

46
3. fejezet

lg (lg a) lg (lg a)
lg a lg a = · lg a = lg (lg a)
lg a

Most ,,visszavesszük a logaritmust”. Ezt jelen esetben a külső logaritmus puszta eltörlésével
is meg lehet tenni, vagy ,,hivatalosan” 10 kitevőjébe tesszük. Mindkét gondolatmenettel azt
lg (lg a)
kapjuk, hogy a lg a = lg a. ♦

Példa 5 : √ √
p √ 6+ 2
Bizonyı́tsuk be, hogy 2 + 3 = .
2
Ismét mindkét oldalt négyzetre emeljük.
√ !2 √
p √ 2 6+ 2
2+ 3 =
2
√ √
√ 6 + 2 12 + 2 8+4 3
2+ 3 = =
√ √4 4
2 + 3 = 2 + 3♦

Példa 6 : p √ p √
Bizonyı́tsuk be, hogy 7 + 2 6 − 7 − 2 6 = 2.

Ismételten négyzetre emeljük a két oldalt.


√ p √ p √ √
7+2 6−2 7+2 6· 7−2 6+7−2 6 = 4
q √  √ 
14 − 2 7 + 2 6 7 − 2 6 = 4

14 − 2 49 − 4 · 6 = 4
14 − 2 · 5 = 4♦

47
4. fejezet

4. Egyenletek
4.1. Egyenlettı́pusok
Minden egyenlettı́pus megoldásának van egy ,,kulcslépése”. Ez a lépés csak akkor tehető meg,
ha előtte az egyenletnek arról az oldaláról, ahol a kifejezés található, minden mást eltüntettünk
(együtthatók, műveletek).
A teljesség kedvéért hozzátesszük, hogy mint minden szabály alól, most is vannak kivételek.
Ezeket a lépéseket más esetekben is meg lehet tenni, de sokkal egyszerűbb megjegyezni azt, hogy
mindegyiket ugyanúgy kell megcsinálni. Ellenkező esetben sok-sok kivételt és egyéb összefüggést
kellene megtanulni és átlátni.

4.1.1. Az abszolút értékes egyenlet


A kulcslépés az esetszétválasztás.

Példa 1 :
Oldjuk meg a valós számok halmazán: 3 − 5|2x − 5| = −7

−5|2x − 5| = −10
|2x − 5| = 2
és most jöhet az esetszétválasztás
↙↘
2x − 5 = 2 2x − 5 = −2
x = 3,5 x = 1,5♦

Példa 2 :
Oldjuk meg a valós számok halmazán: |5 − x| − 4 = x

|5 − x| = x+4
és most jöhet az esetszétválasztás
↙↘
5−x = x+4 5 − x = −(x + 4)
x = 0,5 nincs megoldás♦

Példa 3 :
Oldjuk meg a valós számok halmazán: x − |x| = 2x + 1

x − |x| = 2x + 1
jöhet azonnal az esetszétválasztás
↙↘
x − |x| = 2x + 1 x − |x| = −(2x + 1)
−x − 1 = |x| 3x + 1 = |x|
↙↘ ↙↘
−x − 1=x −(−x − 1)=x 3x + 1=x −(3x + 1)=x
x=−0,5 nincs megoldás x=−0,5 x=−0,25

Az ellenőrzés során az jön ki, hogy csak az x = −0,25 a helyes megoldás.♦

48
4. fejezet

Az |x| + |x + 1| = 3 tı́pusú egyenleteket, vagyis azokat, amikor két abszolút érték összege vagy
különbsége szerepel, másképp kell megoldani. (4.2. fejezet)

4.1.2. A hatványegyenlet
A kulcslépés a gyökvonás.

Példa 1 :
Oldjuk meg a valós számok halmazán: 3x4 − 7 = 41

3x4 = 48
4
x = 16
és most lehet gyököt vonni
↙↘
x=2 x = −2♦

Példa 2 :
Oldjuk meg a valós számok halmazán: 7(x − 1)6 − 1 = 5102

7(x − 1)6 = 5103


(x − 1)6 = 729
és most lehet gyököt vonni
↙↘
x−1 = 3 x − 1 = −3
x = 4 x = −2♦

4.1.3. A gyökös egyenlet


A kulcslépés a hatványra emelés.

Példa 1 : √
Oldjuk meg a valós számok halmazán: 4 + 3 2x − 1 = 10


3√2x − 1 = 6
2x − 1 = 2
és most lehet négyzetre emelni
2x − 1 = 4
x = 2,5♦

Példa 2 : √
Oldjuk meg a valós számok halmazán: 3 x + 5 + 2 = 6


3
x+5 = 4
és most lehet hatványozni
x+5 = 64
x = 59♦

49
4. fejezet

4.1.4. Az exponenciális egyenlet


A ,,szép” exponenciális egyenletnek a mivel az exponenciális függvény szigorúan monoton...
mondat, a ,,csúnya” exponenciális egyenletnek pedig a logaritmus behozatala a kulcslépése. Itt
még az is feltétel, hogy a másik oldalon is csak egy hatvány / szám legyen.

Példa 1 :
Oldjuk meg a valós számok halmazán: 5 − 35x+3 = −4

−35x+3 = −9
35x+3 = 32
mivel az exp...
5x + 3 = 2
x = −0,2
Ez az egyenlet attól ,,szép”, mert a mondat előtt a másik oldalon is egy háromhatvány szerepel.♦

Példa 2 :
1
Oldjuk meg a valós számok halmazán: 9x+1 · = 27x−4 · 3
81x−1

1
32(x+1) · = 33(x−4) · 31
34(x−1)
32(x+1) · 3−4(x−1) = 33(x−4) · 31
32x+2 · 3−4x+4) = 33x−12 · 31
3−2x+6 = 33x−11
Csak most lett egy darab hatvány
mindkét oldalon, szóval most jön, hogy
mivel az exp...
−2x + 6 = 3x − 11
17
x =
5
Ez az egyenlet is ,,szép”, mert csupa háromhatvány szerepel benne.♦

Példa 3 :
Oldjuk meg a valós számok halmazán: 2 · 36x−8 − 1 = 23

2 · 36x−8 = 24
36x−8 = 12
vesszük mindkét oldal logaritmusát
lg 36x−8 = lg 12
(6x − 8) lg 3 = lg 12
x ≈ 1,71
Ez az egyenlet attól ,,csúnya”, mert a 12 nem háromhatvány, tehát logaritmusra lesz szükség.♦

50
4. fejezet

4.1.5. A logaritmusos egyenlet


A kulcslépés a definı́ció / mivel a logaritmus függvény szigorúan monoton... mondat.

Példa 1 :
Oldjuk meg a valós számok halmazán: log2 (x + 2) = 7

Jöhet a logaritmus definı́ciója.


27 = x + 2
x = 126♦

Példa 2 :
Oldjuk meg a valós számok halmazán: log2 x + 2 = 7

log2 x = 5
most jöhet a logaritmus definı́ciója
x = 32♦

Oda kell figyelni, hogy van-e zárójel a logaritmus után, mert fontos, hogy egy szám hozzátartozik
a logaritmushoz, vagy nem.

Példa 3 :
Oldjuk meg a valós számok halmazán: 3 + 5 log2 (7x + 1) = 18

5 log2 (7x + 1) = 15
log2 (7x + 1) = 3
Most alkalmazzuk a logaritmus definı́cióját.
23 = 7x + 1
x = 1♦

Ezekben a példákban a logaritmus egy számmal egyenlő, tehát a definı́cióját használjuk.

Példa 4 :
Oldjuk meg a valós számok halmazán: 2 lg 5 + lg x = 1 − lg 2

lg 52 + lg x = lg 10 − lg 2
lg (25x) = lg 5
Csak most lett egy darab logaritmus
mindkét oldalon, szóval most jön, hogy
mivel a log...
25x = 5
1
x = ♦
5
A logaritmus egy logaritmussal egyenlő, tehát a mondatot használjuk.

A mondat és a definı́ció alkalmazása ugyanaz, szóval senki se gondolja azt, hogy kétféle
definı́ciója van a logaritmusnak.

51
4. fejezet

4.1.6. A trigonometrikus egyenlet


A kulcslépés a számológépbe beütés.

Példa 1 :
Oldjuk meg a valós számok halmazán: 2 cos x − 1 = 0

1
cos x =
2
↙↘

x = 60 + k · 360 ◦ x = 300◦ + l · 360◦
π 5π
x = + k2π x = + l2π
3 3
k; l ∈ Z ♦

Példa 2 :
Oldjuk meg a valós számok halmazán: 4 sin2 x − 3 = 0
3
sin2 x =
4
√ ↙↘ √
3 3
sin x = sin x = −
2 2
↙↘ ↙↘
x=60◦ + k · 360◦ x=120◦ + l · 360◦ x=−60◦ + m · 360◦ x=240◦ + n · 360◦
π 2π π 4π
x= + k2π x= + l2π x=− + m2π x= + n2π
3 3 3 3
k; l; m; n ∈ Z ♦

4.2. A |x + 1| + |x − 3| = 8 tı́pusú egyenletek


Amikor abszolút értékek összege vagy különbsége szerepel, táblázatba foglalva érdemes esetekre
bontani. A gondolatmenetet már 2.6. fejezetben részleteztük.

Példa 1 :
Oldjuk meg a valós számok halmazán: |x + 1| + |x − 3| = 8

x ≤ −1 −1 < x < 3 3≤x


|x + 1| −(x + 1) x+1 x+1
|x − 3| −(x − 3) −(x − 3) x−3
|x + 1| + |x − 3| −2x + 2 4 2x − 2
Az oszlopok szerint egyesével három különálló elsőfokú egyenletet kell most már megoldani.

−2x + 2 = 8 4=8 2x − 2 = 8
x = −3 nincs megoldás x = 5

A −3 és az 5 is beleesik az adott oszlop intervallumába, ı́gy jó megoldások.♦

52
4. fejezet

Példa 2 :
Oldjuk meg a valós számok halmazán: |x − 2| + |x − 3| + |2x − 8| = 9

Első lépésként most is megállapı́tjuk, hogy az abszolút értékben lévő kifejezések hol nullák.
x = 2; x = 3 és x = 4 esetén.
A táblázat ugyanúgy néz ki, csak most négy eset lehetséges.

x≤2 2<x≤3 3<x≤4 4<x


|x − 2| −(x − 2) x−2 x−2 x−2
|x − 3| −(x − 3) −(x − 3) x−3 x−3
|2x − 8| −(2x − 8) −(2x − 8) −(2x − 8) 2x − 8
|x − 2| + |x − 3| + |2x − 8| −4x + 13 −2x + 9 −3 4x − 13

A különálló egyenletek most a következők:

−4x + 13 = 9 −2x + 9 = 9 −3 = 9 4x − 13 = 9
x = 1 x = 0 nincs megoldás x = 5,5

Az x = 0 hamis megoldás, mert nem esik bele a megfelelő intervallumba, de a másik kettő jó.♦

4.3. A gyökös egyenletek


A gyökös egyenletek abból a szempontból különlegesebbek, hogy ott igazán fontos a kikötéssel
kezdeni, mert több is van, és lehet már az elején nem lesz megoldás.

Példa 1 : √
Oldjuk meg a valós számok halmazán: x − 3 = 2 − x

Kikötések:
x−3≥0⇒x≥3
2−x≥0⇒2≥x

Mivel a kikötések metszete üres, ezért biztosan nincs megoldás, vagyis hozzá sem kell fogni a
feladathoz.♦

Példa 2 : √ √
Oldjuk meg a valós számok halmazán: x − 3 = 1 − x

53
4. fejezet

Kikötések:
x−3≥0⇒x≥3
1−x≥0⇒1≥x

Itt sincs megoldás, mert a kikötések metszete üres.♦

Figyeljünk oda, hogy az első példában a jobb oldalnak a gyök értékkészlete miatt, mı́g a második
feladatban az értelmezési tartománya miatt kell nemnegatı́vnak lennie.

Példa 3 : √
Oldjuk meg a valós számok halmazán: x + 2 = x − 4

Kikötések:
x + 2 ≥ 0 ⇒ x ≥ −2
x−4≥0⇒x≥4
Négyzetre emeljük.

x+2 = x2 − 8x + 16
2
x − 9x + 14 = 0
x1 = 7
x2 = 2 hamis megoldás
Természetesen a kikötések elvégzése nélkül, a feladat végén ellenőrzéssel is megkapjuk, hogy a
2 hamis megoldás.♦

Példa 4 : √ √
Oldjuk meg a valós számok halmazán: x2 + 1 + x2 − 3 = 2

Kikötések:
x2 + 1 ≥ 0 mindig igaz
x2 − 3 ≥ 0 ⇒ x2 ≥ 3
Négyzetre emeljük.
√ √
x2 + 1 + 2 x2 + 1 ·√ x2 − 3 + 2
√x − 3 = 4
2√x2 + 1 · √x2 − 3 = 6 − 2x2
x2 + 1 · x2 − 3 = 3 − x2
Mivel ebben a stádiumban (a második négyzetre emelés előtt) a bal oldalon két nemnegatı́v
szám szorzata áll, ezért a jobb oldalnak is nemnegatı́vnak kell lennie.
Kikötés:
3 − x 2 ≥ 0 ⇒ 3 ≥ x2
Az új kikötés ismeretében nem szükséges folytatnunk az egyenletünket, mert x2 csak 3 lehet.
Figyelem! A kikötés nem azt mondja,√ hogy ez a megoldás, hanem, hogy ez lehet a megoldás,
tehát le kell ellenőriznünk. x = ± 3 esetén valóban jó megoldást kapunk.♦

54
4. fejezet

4.4. Egyenletrendszer megoldása új ismeretlen bevezetésével


Ha egy egyenletrendszerben bonyolult, de hasonló kifejezések vannak, érdemes megpróbálkozni
az új ismeretlen bevezetésével. Ezzel visszavezethető a feladat egyszerűbb egyenletrendszerre.

Példa 1 :  2 3
 + = 8
x−4 y−2


Oldjuk meg a valós számok halmazán:
 4 5

 + = 10
x−4 y−2
1 1
Legyen e = ; f=
x−4 y−2
 2 3
 + = 8
x−4 y−2
 
2e + 3f = 8

⇒ ⇒ e = −5; f = 6
 4 5 4e + 5f = 10

 + = 10
x−4 y−2

Visszahelyettesı́tve:
1 1 19 13
= −5 és =6 ⇒ x= és y =
x−4 y−2 5 6

Egyébként azt is észre lehet venni, hogy nem érdemes beszorozni a nevezőkkel, mert lesz x-es,
y-os és xy-os tag is, tehát nem elsőfokú az egyenletrendszer, bonyolultabb lett.♦

Példa 2 : 
2x + 3 · 2y = 9,5
Oldjuk meg a valós számok halmazán:
5 · 2x − 6 · 2y = 37

Legyen e = 2x ; f = 2y

 
2x + 3 · 2y = 9,5 e + 3f = 9,5 1
⇒ ⇒ e = 8; f =
5 · 2x − 6 · 2y = 37 5e − 6f = 37 2

Visszahelyettesı́tve:
1
2x = 8 és 2y = ⇒ x = 3 és y = −1
2

mivel az exponenciális függvény szigorúan monoton. . . ♦

Példa 3 :
lg2 x + lg2 y = 5

Oldjuk meg a valós számok halmazán:
lg x − lg y = 1

Legyen e = lg x; f = lg y

lg2 x + lg2 y = 5
 
e2 + f 2 = 5
⇒ ⇒ e1 = 2; f1 = 1 illetve e2 = −1; f2 = −2
lg x − lg y = 1 e−f = 1

55
4. fejezet

Visszahelyettesı́tve:

lg x = 2 és lg y = 1 ⇒ x1 = 100 és y1 = 10


illetve
lg x = −1 és lg y = −2 ⇒ x2 = 0,1 és y2 = 0,01♦

Példa 4 :  √
tg x + tg y = 2 3
Oldjuk meg a valós számok halmazán:
tg x · tg y = 3

Legyen e = tg x; f = tg y

√ √
√ √
 
tg x + tg y = 2 3 e+f = 2 3
⇒ ⇒ e = 3; f = 3
tg x · tg y = 3 ef = 3

Visszahelyettesı́tve:
√ √ π π
tg x = 3 és tg y = 3 ⇒ x= + kπ és y = + lπ, ahol k; l ∈ Z♦
3 3

4.5. Tömegszázalékos szöveges feladatok


A kémiás feladatoknak van egy hasznos megoldási eljárása, egy táblázat készı́tése. A táblázat
sorai az oldatok / ötvözetek, legalul pedig az összeöntött keverék. Az oszlopok pedig mindig:
oldat mennyisége, százalék, oldott anyag mennyisége. Ez független attól, hogy kénsavas, alko-
holos, ötvözetes vagy sóoldatos feladatról van szó. Az oldat az oldott anyagból és az oldószerből
áll. Az oszlopok között mindig az az összefüggés áll fenn, hogy
tömegszázalék
oldat mennyisége · = oldott anyag mennyisége
100

A mennyiségek természetesen összeadhatók. Az egyenlet pedig vagy abból adódik, hogy az ol-
dott anyagok összege egyenlő a keverékben található oldott anyag mennyiségével, vagy a fent
emlı́tett összefüggésből.

Példa 1 :
Mennyi 26%-os kénsavat kell hozzákeverni 40 kg 68%-os kénsavhoz, hogy 32%-os koncentrációjú
kénsavat állı́tsunk elő?

oldat mennyisége (kg) tömegszázalék (%) oldott anyag mennyisége (kg)


1. oldat x 26% x · 0,26
2. oldat 40 68% 40 · 0,68
keverék x + 40 32% (x + 40) · 0,32

56
4. fejezet

x · 0,26 + 40 · 0,68 = (x + 40) · 0,32


0,26x + 27,2 = 0,32x + 12,8
x = 240
240 kg kénsavra van szükség.♦

Példa 2 :
Két kénsavoldat közül az egyik 0,8 kg, a másik 0,6 kg tömény kénsavat tartalmaz. Ha a két ol-
datot összeöntjük, akkor 10 kg kénsavoldatot kapunk. Mekkora volt az első és a második oldat
tömege, illetve hány százalékosak az oldatok, ha a kénsavtartalom százaléka az első oldatban
tı́zzel több, mint a másodikban?

oldat mennyisége (kg) tömegszázalék (%) oldott anyag mennyisége (kg)


1. oldat x y + 10% 0,8
2. oldat 10 − x y% 0,6
keverék 10 − −

A táblázat elkészı́tése után látható, hogy túl kevés adat van megadva, tehát mindenképp két
ismeretlen kell, ı́gy viszont egyenletrendszer lesz.

 y + 10
 x· = 0,8 
100 x · (y + 10) = 80


 (10 − x) · y = 0,6
 (10 − x) · y = 60
100
80
Kifejezve az első egyenletből: x =
y + 10
Ezt behelyettesı́tve a második egyenletbe:
 
80
10 − y = 60
y + 10
80y
10y − = 60
y + 10
10y(y + 10) − 80y = 60(y + 10)
y 2 − 4y − 60 = 0
y1 = 10 ⇒ x1 = 4
y2 = −6 hamis megoldás
Az első oldat 4 kg és 20%-os, a második oldat 6 kg és 10%-os.♦

Példa 3 :
Összeöntünk 6%-os és 30%-os sósavat, s ı́gy 24 liter 15% töménységű sósavat kaptunk. Hány
litert vettünk az egyes összetevőkből?

oldat mennyisége (l) tömegszázalék (%) oldott anyag mennyisége (l)


1. oldat x 6% x · 0,06
2. oldat 24 − x 30% (24 − x) · 0,3
keverék 24 15% 24 · 0,15

57
4. fejezet

0,06x + (24 − x) · 0,3 = 24 · 0,15


0,06x + 7,2 − 0,3x = 3,6
x = 15
Az első oldatból 15 liter, a másodikból 9 liter kell.♦

4.6. Sebességes szöveges feladatok


Sebességes szöveges feladatokhoz az egyenes vonalú egyenletes mozgást és az azt leı́ró
s a 2
v = képletet, illetve az egyenes vonalú egyenletesen gyorsuló mozgást, és az s = ·t
t 2
négyzetes úttörvényt kell ismerni. Ami még fontos, hogy az egy irányba haladó testek se-
bessége kivonódik, ellentétes irányba haladóké pedig összeadódik. Az egyenes vonalú egyenletes
mozgásos feladatokhoz is javasolt elkészı́teni egy táblázatot. A sorok általában az utakat jelölik
(oda-vissza, le-fel, stb.). Az oszlopok pedig: sebesség, út, idő. Az egyenlet pedig abból adódik,
út
hogy sebesség = .
idő
Példa 1 :
A kikötőből induló hajó sebessége 16 km/h. Két órával később utána indı́tanak egy csónakot,
amelynek sebessége 24 km/h. A csónak indulásától számı́tva, mennyi idő múlva éri utol a hajót?

A csónak indulásakor a hajó a csónaktól 32 km-re van. Mivel egy irányba haladnak, ezért a se-
bességüket kivonjuk egymásból. Innentől nem a két test mozgásával számolunk, hanem van
egy 8 km/h-val haladó csónakunk és egy álló hajónk. Tehát mostantól az a kérdés, hogy egy
8 km/h-val haladó test mennyi idő alatt tesz meg 32 km-t. Ehhez még képlet sem kell, 4 óra
alatt.♦

Példa 2 :
Két folyóparti város távolsága 240 km. Ezt az utat a menetrend szerinti hajó oda-vissza 25 óra
alatt teszi meg. Mekkora sebességgel haladna a hajó állóvı́zben, ha a folyó 4 km/h sebességgel
folyik?

sebesség (km/h) út (km) idő (h)


oda v+4 240 t
vissza v−4 240 25 − t

v jelenti a hajó sebességét állóvı́zben. Ezt az egyik irányban a folyó lassı́tja, a másik irányban
gyorsı́tja. Kevés adatot tudunk, tehát kell a két ismeretlen. Egyébként abból is lehet sejteni,
hogy két irány van, tehát két összefüggést kell majd felı́rni.
240

 v+4 =


t
 240
 v−4 =

25 − t
240
Kifejezve az első egyenletből: v = −4
t

58
4. fejezet

Ezt behelyettesı́tve a második egyenletbe:


240 240
−8 =
t 25 − t
240(25 − t) − 8t(25 − t) = 240t
6000 − 240t − 200t + 8t2 = 240t
8t2 − 680t + 6000 = 0
t1 = 10 ⇒ v1 = 20
t2 = 75 hamis megoldás
A hajó 20 km/h sebességgel haladna állóvı́zben.♦

Példa 3 :
Két, egymástól 9 km távolságra lévő pontból egyszerre indul el egy-egy kerékpáros. Ha egymás-
sal szembe mennek, 20 perc múlva, ha egy irányban haladnak, 3 óra múlva találkoznak. Mekkora
a sebességük?

sebesség (km/h) út (km) idő (h)


1
egymás felé v2 + v1 9
3
egy irányban v2 − v1 9 3

Megint úgy képzeljük el a feladatot, hogy az egyik kerékpáros nem mozdul, és a másik mozog
felé egyszer a két sebesség összegével, egyszer pedig a különbségükkel. Most is két irány van és
két kerékpáros. Így sejthető, hogy két ismeretlen kell.
 9
 v2 + v1 = 1 = 27


3
⇒ v2 = 15; v1 = 12

 v −v =
 9
2 1 = 3
3
Az egyik sebessége 15 km/h, a másiké 12 km/h.♦

Példa 4 :
Egy szakadék mélységét úgy is meghatározhatjuk, ha lemérjük, hogy egy beleejtett kő kop-
panását az elengedés után mennyivel később halljuk meg. Milyen mély a szakadék, ha azt ta-
pasztaljuk, hogy ez az időkülönbség 10 másodperc? (A gravitációs gyorsulás értéke g ≈ 10 m/s2 ,
a hang terjedési sebessége 340 m/s)

sebesség (m/s) út (m) idő (s)


lefelé (kő) − h t
felfelé (hang) 340 h 10 − t

A kő lefelé egyenes vonalú egyenletesen gyorsuló mozgást végez, mı́g a hang felfelé egyenes
vonalú egyenletes mozgást.
 g 2
 h = ·t
2
h = 340(10 − t)

59
4. fejezet

5t2 = 340(10 − t)
t2 + 68t − 680 = 0
t1 = 8,84 ⇒ h1 = 390,7
t2 = −76,84 hamis megoldás
A szakadék 390,7 m mély.♦

4.7. Munkavégzéssel kapcsolatos szöveges feladatok


A munkavégzéssel kapcsolatos szöveges feladatoknál is egy táblázat a legpraktikusabb eljárás.
A sorok a szereplők, legalul pedig az együtt végzett munka. Az oszlopok pedig: mennyi idő
a munka, 1 időegység alatt végzett munka. Értelemszerűen, a két oszlop között reciprok vi-
szony áll fenn. Azért van szükség az 1 időegység alatt végzett munkára, mert a munka ilyenkor
összeadható. Az idő nem adható össze. Az egyenlet abból adódik, hogy a két szereplő időegység
alatt végzett munkájának összege megegyezik az együttes munkával. Fontos, hogy a teljes mun-
ka mindig 1 (100%). Ha például a munka 40%-át végezzük el, akkor az 0,4-nek számı́t.

Példa 1 :
A kertet az apa 3,5 óra alatt, a fia 6 óra alatt ásná fel egyedül. Mennyi idő alatt készülnek el a
kert felásásával, ha mindketten dolgoznak?

mennyi idő a munka (h) 1 h alatt végzett munka


7 2
apa
2 7
1
fia 6
6
1
együtt x
x
2 1 1
+ =
7 6 x
42
x =
19
Együtt 2,21 óra alatt végeznek.♦

Példa 2 :
Két munkás együtt dolgozva 8 óra alatt tud befejezni egy munkát. Mennyi idő alatt lenne
készen egyedül a munkával az egyik és a másik, ha az utóbbinak 12 órával több időre lenne
szüksége, mint az elsőnek?

mennyi idő a munka (h) 1 h alatt végzett munka


1
1. munkás x
x
1
2. munkás x + 12
x + 12
1
együtt 8
8

60
4. fejezet

1 1 1
+ =
x x + 12 8
8 = x + 12) + 8x = x(x + 12)
0 = x2 − 4x − 96
x1 = 12
x2 = −8hamis megoldás
Az egyik 12, a másik 24 óra alatt lenne készen.♦

Példa 3 :
Panni és Kati elvállalta, hogy szövegszerkesztővel legépelik Dani szakdolgozatát. A két lány
együttes munkával 12 munkaóra alatt végezne a gépeléssel. Kedden reggel 8 órakor kezdett
Panni a munkához, Kati 10 órakor fogott hozzá. Megállás nélkül, ki-ki egyenletes sebességgel
dolgozott kedden 14 óráig, ekkor a kéziratnak a 40%-ával végeztek, és abbahagyták a munkát.
Hány óra alatt gépelné le Panni, illetve Kati a teljes szakdolgozatot (állandó munkatempót, és
megszakı́tás nélküli munkát feltételezve)?
Szerdán reggel egyszerre kezdtek hozzá 9 órakor a gépeléshez, és együtt egyszerre fejezték
be. Szerdán Panni fél óra ebédszünetet tartott, Kati pedig a délelőtti munkáját egy órányi
időtartamra megszakı́totta. Hány órakor végeztek a lányok a munkával szerdán?

mennyi idő a munka (h) 1 h alatt végzett munka gépelésre fordı́tott munka
1 6
Panni x
x x
1 4
Kati y
y y
1
együtt 12 0,4
12
 1 1 1
 x + y = 12

 
e + f = 12 1 1
⇒ ⇒e= ; f =
6e + 4f = 0,4 30 20
 6 + 4 = 0,4


x y

Visszahelyettesı́tve:
1 1 1 1
= és = ⇒ x = 30 és y = 20
x 30 y 20

Panni 30, Kati 20 óra alatt gépelné le egyedül a teljes szakdolgozatot.

Most tegyük fel, hogy szerdán Panni t órát gépelt. Ekkor Kati t − 0,5 órát gépelt. (Tiszta mun-
kaidőt) Most már nem az a kérdés, hogy egyedül hány óra alatt gépelnék le a szakdolgozatot,
vagyis a nevező. Most már az a kérdés, hogy hány órát dolgoztak szerdán, vagyis a számláló
a kérdés a táblázat harmadik oszlopából. Mivel szerdára maradt a dolgozat 60%-a, ezért az
együtt végzett munka jelenleg 0,6.
t t − 0,5
+ = 0,6
30 20
t = 7,5

61
4. fejezet

Panni tiszta munkaideje 7,5 óra, vagyis 8 óra volt a ,,munkaideje”. Kati tiszta munkaideje ekkor
7 óra, vagyis neki is 8 óra volt a ,,munkaideje”. Tehát 17 órakor végeztek.♦

4.8. Számjegyes szöveges feladatok


A számjegyes feladatokhoz érdemes egy helyi érték táblázatot rajzolni. A sorok a számok
(eredeti-felcserélt, stb.), az oszlopok a helyi értékek, a legutolsó pedig a szám valódi értéke.
Az egyenlet általában a szövegben található összefüggésből adódik.

Példa 1 :
Egy kétjegyű szám tı́zeseinek jegye öttel több, mint az egyeseinek száma. Ha a felcserélt
számjegyű számot hozzáadjuk, akkor 143-at kapunk. Mekkora az eredeti kétjegyű szám?

tı́zesek egyesek szám értéke


eredeti x+5 x 10(x + 5) + x
felcserélt x x+5 10x + (x + 5)

10(x + 5) + x + 10x + (x + 5) = 143


x = 4
Az eredeti szám a 94.♦

Példa 2 :
Határozzuk meg azt a kétjegyű számot, amelyben az egyesek száma kettővel nagyobb a tı́zesek
számánál, és ha a számot megszorozzuk a számjegyeinek az összegével, akkor 144-et kapunk.
Melyik ez a szám?

tı́zesek egyesek szám értéke számjegyek összege


x x+2 10x + (x + 2) = 11x + 2 2x + 2

(11x + 2)(2x + 2) = 144


22x2 + 26x − 140 = 0
x1 = 2
35
x2 = − hamis megoldás
11
Az eredeti szám a 24.♦

Példa 3 :
Ha egy háromjegyű szám számjegyeit fordı́tott sorrendben ı́rjuk, és az eredetiből kivonjuk, a
különbség 500 és 600 között lesz. A középső számjegy hárommal kisebb a másik kettő összegénél,
a százasok helyén álló számjegy négyzete néggyel nagyobb a második számjegy kilencszeresénél.
Melyik ez a szám?

Három mondat, három információ van megadva, ı́gy az a sejtés, hogy három egyenlet kell,
három ismeretlennel.

62
4. fejezet

százasok tı́zesek egyesek szám értéke


eredeti x y z 100x + 10y + z
felcserélt z y x 100z + 10y + x

 500 ≤ (100x + 10y + z) − (100z + 10y + x) ≤ 600
x+z = y+3
 2
x − 4 = 9y
Foglalkozzunk az első összefüggéssel.

500 ≤ 99x − 99z ≤ 600
500 ≤ 99(x − z) ≤ 600

Az egyetlen 99 többszörös, ami ebben az intervallumban van az 594 ⇒ x − z = 6



 x−z = 6
x+z = y+3
 2
x − 4 = 9y

Kifejezve az első egyenletből: z = x − 6



x+x−6 = y+3
x2 − 4 = 9y

Kifejezve a már kéttagú egyenletrendszer első egyenletéből: y = 2x − 9

x2 − 4 = 9(2x − 9)
x2 − 18x + 77 = 0
x1 = 7 ⇒ y1 = 5 ⇒ z1 = 1
x2 = 11 hamis megoldás
Az eredeti szám a 751.♦

4.9. Egyenletre vagy egyenletrendszerre vezető szöveges feladatok


A javaslat az lenne, hogy azoknál a feladatoknál, ahol az adatok között százalékos összefüggés
van megadva, érdemes azt választani ismeretlennek, amihez képest viszonyı́tva van a többi
adat. Ebben az esetben a többi adatot úgy kapjuk meg, hogy az ismeretlent felszorozzuk.

Példa 1 :
Adott három város. A B város 30%-kal több energiát fogyaszt, mint az A város, a C város pe-
dig 12%-kal többet, mint a B város. A három város együtt egy hónapban 16 902 MJ-t fogyaszt.
Mennyit fogyasztanak külön-külön?

Ebben a feladatban B város az A-hoz, C város pedig B-hez van viszonyı́tva, tehát A várost
választjuk kiindulópontnak, viszonyı́tási pontnak.
Legyen akkor A = x, ı́gy B = 1,3x és C = 1,12 · 1,3x

x + 1,3x + 1,12 · 1,3x = 16902


3,756x = 16902
x = 4500
Vagyis A város 4500 MJ-t, B város 5850 MJ-t, és C város 6552 MJ-t fogyaszt.♦

63
4. fejezet

Példa 2 :
A mosogatógépünkön háromféle program van. Egy mosogatáshoz az A program 20%-kal több
elektromos energiát, viszont 10%-kal kevesebb vizet használ, mint a B program. A B program
30%-kal kevesebb elektromos energiát és 25%-kal több vizet használ egy mosogatáshoz, mint
a C program. Mindhárom program futtatásakor 40 F t-ba kerül az alkalmazott mosogatószer.
Egy mosogatás az A programmal 151 F t-ba, a B programmal 140 F t-ba kerül. Mennyibe kerül
a C programmal egy mosogatás?

Az alapos elolvasás után látható, hogy az A program a B-hez, mı́g a B program a C-hez
van viszonyı́tva. Tehát a C program vı́z- és áramfogyasztásához vezetjük be az ismeretleneket.
Készı́tünk egy táblázatot a jobb átláthatóság kedvéért.

áram vı́z
A 1,2 · 0,7x 0,9 · 1,25y
B 0,7x 1,25y
C x y

Kapjuk a következő egyenletrendszert:



0,84x + 1,125y + 40 = 151
⇒ x = 100; y = 24
0,7x + 1,25y + 40 = 140

Tehát a C program 164 F t-ba kerül.♦

A következő két feladatban az látható, hogy van egy ,,eredeti elképzelés”, és utána valami
miatt azt megváltoztatjuk, ı́gy adódik egy ,,új helyzet”. Ezek általában egy ismeretlennel nem
oldhatók meg, másodfokú egyenletrendszer lesz belőlük.

Példa 3 :
Nyelvvizsgára készülve minden nap ugyanannyi tesztet oldottam meg egy 720 tesztet tartalmazó
könyvből. Ha naponta 20 teszttel többet oldottam volna meg, akkor 3 nappal hamarabb lettem
volna készen vele. Hány napig tartott a tesztek megoldása?

Legyen x a naponta megoldott tesztek száma, y a napok száma.



xy = 720
(x + 20)(y − 3) = 720

720
Kifejezve az első egyenletből: y =
x
 
720
(x + 20) −3 = 720
x
14400
720 − 3x + − 60 = 720
x
0 = x2 + 20x − 4800
x1 = 60 ⇒ y1 = 12
x2 = −80 hamis megoldás
12 napig tartott és naponta 60 tesztet oldottam meg.♦

64
4. fejezet

Példa 4 :
Egy társaság 48 000 F t-ért kisbuszt bérel. Az utolsó pillanatban még egy utas csatlakozott, ı́gy
mindenkinek 200 F t-tal kevesebbet kellett fizetnie. Hány utas volt eredetileg?

Legyen x az utasok száma, y az egy fő által befizetett összeg.



xy = 48000
(x + 1)(y − 200) = 48000
48000
Kifejezve az első egyenletből: y =
x
 
48000
(x + 1) − 200 = 48000
x
48000
48000 − 200x + − 200 = 48000
x
0 = x2 + x − 240
x1 = 15 ⇒ y1 = 3200
x2 = −16 hamis megoldás
Eredetileg 15 utas volt, és fejenként 3200 F t-ot fizettek be.♦

A következő két feladat bonyolultnak tűnhet, hiszen háromismeretlenes egyenletrendszer lesz


belőlük, de egy trükkel pillanatok alatt megoldhatók lesznek úgy, hogy nem kell egyik ismeret-
lent sem kifejezni és behelyettesı́teni a többi egyenletbe.

Példa 5 :
Mekkorák a háromszög oldalai, ha két-két oldal összege 12; 15; 17 cm?


 a + b = 12
a + c = 15
b + c = 17

A trükk lényege az, hogy összeadjuk a három egyenletet, majd elosztjuk kettővel.

2a + 2b + 2c = 44
a + b + c = 22
Innen pedig azonnal leolvasható az egyes sorokhoz képest a különbség.
c = 10 cm; b = 7 cm; a = 5 cm.♦

Példa 6 :
Mekkorák a téglatest élei, ha három különböző oldallapja 72; 96; 108 cm2 területű?


 ab = 72
ac = 96
bc = 108

Most a trükk az, hogy összeszorozzuk a három egyenletet és gyököt vonunk belőle.

a2 b2 c2 = 746 496
abc = 864
Innen már leolvasható az egyes sorokhoz képest a hányados.
c = 12 cm; b = 9 cm; a = 8 cm.♦

65
4. fejezet

Zárásként annyit szeretnék még hozzátenni a példákhoz, hogy a szöveges feladatoknak nincs
egy általános, sablonos levezetése. Vannak tı́pusfeladatok, amiket mindig ugyanúgy érdemes,
ezeket igyekeztem bemutatni, de a legtöbb esetben valami egyéni gondolat kell. A bemutatott
példák csak ötleteket és módszereket adnak, hogy mivel érdemes elindulni. Néha egy rajz, néha
egy táblázat, néha csak egy apró észrevétel, vagy az ismeretlen jó helyre történő bevezetése
sokat segı́thet az elindulásnál. Nagyon fontos, hogy abból, hogy milyen hosszú a feladat, hány
mondat van, mennyi információ és összefüggés van megadva, lehet sejteni, hogy egyenletre vagy
egyenletrendszerre vezet a feladat. Ha rövid a szöveg, általában egy mondat van, akkor egy is-
meretlen a valószı́nű. Hosszabb szövegnél, vagy ha két mondat van, vagy ha két kérdés van,
akkor a kétismeretlenes egyenletrendszer a valószı́nűbb.

4.10. Gyökös egyenlet visszavezetése abszolút értékes egyenletre


A gyökös egyenleteknél, ha a gyök alatt másodfokú kifejezés van, érdemes megnézni még a
négyzetre emelés előtt (4.3. fejezet), hogy az teljes négyzetté alakı́tható-e, mert ha igen, akkor
abszolút értékes egyenlet lesz belőle.

Példa 1 : p
Oldjuk meg a valós számok halmazán: log3 2 x − 8 log3 x + 16 = 2

A gyök alatt teljes négyzet van.


p
(log3 x − 4)2 = 2
| log3 x − 4| = 2
↙↘
log3 x − 4 = 2 log3 x − 4 = −2
log3 x = 6 log3 x = 2
x = 729 x = 9♦

Példa 2 :
Oldjuk
p meg a valós számok
√ halmazán: p
sin x − 4 sin x + 4 + sin2 x + 4 sin x + 4 = sin2 x + 7 sin x + 12,25
2

A gyökök alatt mindenhol teljes négyzetek vannak. Ezeket visszaalakı́tjuk.


p p p
(sin x − 2)2 + (sin x + 2)2 = (sin x + 3,5)2
| sin x − 2| + | sin x + 2| = | sin x + 3,5|



 sin x − 2 ∈ [−3; −1] ⇒ biztosan negatı́v szám
sin x ∈ [−1; 1] ⇒ sin x + 2 ∈ [1; 3] ⇒ biztosan pozitı́v szám

sin x + 3,5 ∈ [2,5; 4,5] ⇒ biztosan pozitı́v szám

−(sin x − 2) + sin x + 2 = sin x + 3,5


0,5 = sin x
↙↘
x = 30◦ + k · 360◦ x = 150◦ + l · 360◦
π 5π
x = + k2π x = + l2π, ahol k, l ∈ Z.♦
6 6

66
4. fejezet

4.11. Szélsőérték feladatok


Az emelt érettségiben előfordulhatnak szélsőérték feladatok is. Ezek egy mennyiség maxi-
mumának vagy minimumának a megtalálását jelentik. Nem kell azonnal deriválással nekies-
ni, mert annak ellenére, hogy deriválni egy függvényt egyszerű, sok minden mást is le kell
ellenőrizni. A következőkben lássunk néhány másik szélsőérték kiszámolási módszert.

4.11.1. Elemi úton, másodfokú függvényábrázolással (diszkrimináns > 0)


Az elemi úton kifejezés arra utal, hogy nem használunk analı́zist azaz deriválást vagy integrálást.

Példa 1 :
A 20 cm kerületű téglalapok közül melyiknek a legnagyobb a területe?

A téglalap egyik oldala x, a másik 10 − x. Ekkor a terület T (x) = x (10 − x), vagyis az x-nek
egy függvénye. Ennek a függvénynek kell a szélsőértéke. Mivel ez egy parabola, és a parabola
szélsőértéke a két zérushely számtani közepénél van a tengelyes szimmetria miatt, ezért elég a
zérushelyeket megállapı́tani.

x(10 − x) = 0
x1 = 0
x2 = 10
Vagyis x = 5 cm esetén lesz ez a terület maximális. Ez a négyzet.♦

Példa 2 :
Egy trapéz alapja 4 cm, a másik alapjának és magasságának összege 10 cm. Hogyan válasszuk
meg a másik alapot és a magasságot, hogy a terület maximális legyen?

A szöveg alapján:

m + c = 10
c = 10 − m

Ekkor a terület:
a+c
T = ·m
2
4 + (10 − m)
T = ·m
2 2
14m − m
T =
2
Vagyis a terület m-nek egy függvénye. Ennek a függvénynek kell a szélsőértéke. Mivel ez is
egy parabola, és a parabola szélsőértéke a két zérushely számtani közepénél van a tengelyes
szimmetria miatt, ezért újfent elég a zérushelyeket kiszámolni.

67
4. fejezet

14m − m2
= 0
2
m(14 − m) = 0
m1 = 0
m2 = 14
Vagyis m = 7 cm és c = 3 cm esetén lesz a trapéz területe maximális.♦

4.11.2. Elemi úton, teljes négyzetté alakı́tással (diszkrimináns < 0)


Példa 1 :
Bontsuk fel a 30-at két szám összegére úgy, hogy a tagok négyzetösszege a legkisebb legyen.

Az egyik szám x, a másik 30 − x. Keressük x2 + (30 − x)2 legkisebb értékét.

x2 + (30 − x)2 = 2x2 − 60x + 900

Ebben az esetben sem kiemeléssel, sem a megoldóképlettel nem kapjuk meg a zérushelyeket.
Így teljes négyzetté alakı́tással fogjuk megkapni a szélsőértéket.

2x2 − 60x + 900 = 2 x2 − 30x + 900 = 2 (x − 15)2 − 225 + 900 = 2(x − 15)2 + 450 ↑
   

Vagyis a parabola jobbra 15-tel, felfelé 450-nel van eltolva. x = 15 esetén lesz a négyzetösszeg
a legkisebb.♦

Példa 2 :
A p valós paraméter mely értékénél lesz az x2 − (p − 2)x + p − 3 = 0 egyenletben a gyökök
négyzetösszege minimális?

a = 1; b = −(p − 2); c = p − 3
Mivel az egyenletnek két megoldása kell legyen, ezért a diszkrimináns nagyobb, vagy egyenlő,
mint nulla.

[−(p − 2)]2 − 4 · 1 · (p − 3) = p2 − 8p + 16 = (p − 4)2 ≥ 0

ami mindig igaz, vagyis p bármilyen értékére lesz megoldás. A diszkrimináns viszont csak a
gyökök számának megadására alkalmas, a gyökök közti összefüggésekre a Viète-formulákat kell
alkalmazni.
b

 x1 + x2 = −
 = p−2
a
c
= p−3

 x1 x2 =
a
A gyökök négyzetösszege x1 2 + x2 2 . Ennek értékéhez úgy juthatunk el, hogy az első egyenletet
négyzetre emeljük, a másodikat pedig megszorozzuk kettővel.
 2
x1 + 2x1 x2 + x2 2 = p2 − 4p + 4
2x1 x2 = 2p − 6

68
4. fejezet

Ezután a két egyenletet kivonjuk egymásból.

x1 2 + x2 2 = p2 − 4p + 4 − (2p − 6) = p2 − 6p + 10

Ennek a kifejezésnek kell a minimuma. A megoldóképlettel nem kapunk valós megoldásokat,


ı́gy megint a teljes négyzetté alakı́tást alkalmazzuk.

p2 − 6p + 10 = (p − 3)2 − 9 + 10 = (p − 3 )2 + 1 ↑

Tehát p = 3 esetén lesz a négyzetösszeg a legkisebb.♦

4.11.3. Elemi úton, számtani és mértani középpel


Példa 1 :
Bizonyı́tsuk be, hogy ha x > 0, akkor egy szám és a reciprokának összege legalább 2.
1
Felı́rjuk x-re és -re a számtani és mértani közép közti egyenlőtlenséget.
x
1
x+ r
1
x ≥ x·
2 x
1
x+
x ≥ 1
2
1
x+ ≥ 2
x
Kész a bizonyı́tás. Egyenlőség akkor áll fenn, ha x = 1.♦

Példa 2 :
A 10-et felbontjuk két pozitı́v szám összegére. Legfeljebb mekkora lehet a két szám szorzata?

Az egyik szám x, a másik 10 − x.


Felı́rjuk x-re és 10 − x-re a számtani és mértani közép közti egyenlőtlenséget.

x + (10 − x) p
≥ x · (10 − x)
2 p
5 ≥ x · (10 − x)
25 ≥ x(10 − x)

Vagyis legfeljebb 25 lehet a szorzat.


Figyelem! Most nem az volt a kérdés, hogy mi a két összetevő (bár ezeket is ki lehetne számolni,
ha folytatnánk a feladatot és x = 5 esetén teljesül is, hogy 25 az érték), hanem csak annyi,
hogy mekkora lehet legfeljebb a szorzat. ♦

4.11.4. Differenciálással
Ilyen esetben általában egy kétváltozós összefüggés szélsőértéke kell (terület, kerület, térfogat,
felszı́n. . .). Ahhoz, hogy az egyik változót kiküszöbölhessük, szükség van a két változó között

69
4. fejezet

egy másik összefüggésre, ahonnan az egyiket kifejezhetjük. Ezt visszahelyettesı́tve a kérdéses


összefüggésbe már egy egyváltozós függvényt kapunk. Mindegyik példában ez az alapvető gon-
dolatmenet. A feladatoknak van további feltétele is, melyről részletesebben a 12.2. fejezetben
lehet olvasni.

Példa 1 :
A 100 m2 területű téglalap alakú kertek közül melyiknek a legkisebb a kerülete?
100
Ismerjük a két ismeretlen között azt az összefüggést, hogy T = xy = 100 ⇒ y =
  x
100 200
Behelyettesı́tve a kerületbe: K = 2(x + y) = 2 x + = 2x +
x x
Vagyis maradt egy egyváltozós függvényünk, a kerület.
200
K(x) = 2x + , ahol K : R+ → R
x
200
K ′ (x) = 2 − 2
x
200
2− 2 = 0
x
2
2x − 200 = 0
x = 10 ⇒ y = 10

A legkisebb kerületű téglalap a négyzet.♦

Példa 2 :
Az y = ax + b egyenletű egyenes illeszkedik a (2; 6) pontra. Tudjuk, hogy a < 0. Jelölje az
x-tengely és az egyenes metszéspontját P , az y-tengely és az egyenes metszéspontját pedig Q.
Írjuk fel annak az egyenesnek az egyenletét, amelyre az OP Q háromszög területe a legkisebb,
és számı́tsuk ki ezt a területet (O a koordináta-rendszer origóját jelöli).

Mivel y = ax + b egyenesre illeszkedik a (2; 6), ezért:

6 = 2a + b
b = 6 − 2a

Vagyis megint ki van fejezve az egyik betű a másikkal.


OP · OQ
T△ = , tehát szükség van P és Q koordinátáira a; b-vel kifejezve.
2
P az egyenes x-tengellyel vett metszéspontja, emiatt:

70
4. fejezet

ax + b = 0
b
x = −
a
6 − 2a
x = −
a
6
x = 2−
a
 
6
Innen tudjuk, hogy P 2 − ; 0 .
a
Q az egyenes y-tengellyel vett metszéspontja, ezért:

y = a·0+b
y = 6 − 2a

Ebből következik, hogy Q(0; 6 − 2a).


Marad egy egyváltozós függvényünk.
 
6
2− (6 − 2a)
a
T (a) =
2
18
T (a) = 12 − 2a − , ahol T : R− → R
a
18
T ′ (a) = −2 + 2
a
18
−2 + 2 = 0
a
2a2 = 18
a = −3 ⇒ b = 12

Az egyenes egyenlete y = −3x + 12, a legkisebb terület pedig 24.♦

Példa 3 :
Mekkorák a 10 cm és 15 cm befogójú derékszögű háromszögbe ı́rható maximális területű téglalap
oldalai, ha a téglalap egyik szöge a háromszög derékszögével esik egybe?

Ismerjük a két változó között azt az összefüggést, hogy Ttéglalap = xy. Szükség van egy másik
összefüggésre, hogy az egyik változót kiküszöbölhessük. Ez a másik összefüggés a hasonlóság
lesz.

71
4. fejezet

10 − x 10
AED△ ∼ ABC△ ⇒ =
y 15
3(10 − x)
Ebből kifejezve: y = = 15 − 1,5x
2
Ttéglalap = x(15 − 1,5x)
T (x) = 15x − 1,5x2 , ahol T : ]0; 10[→ R
T ′ (x) = 15 − 3x
15 − 3x = 0
x = 5 ⇒ y = 7,5
A téglalap oldalai 5 cm és 7,5 cm (ED és F D tehát a háromszög középvonalai).♦

Példa 4 :
A fél literes henger alakú korsók közül melyiknek a legolcsóbb az előállı́tása? (Figyelmen kı́vül
hagyjuk az anyag vastagságát.)

0,5 l = 500 cm3

Ismerjük a két változó között a térfogatot, mint összefüggést.

500 = r2 π · M
500
M = 2
r π
A felszı́n minimuma kell.

A(r) = r2 π + 2rπ · M
500 1000
A(r) = r2 π + 2rπ · 2 = r2 π + , ahol A : ]0; ∞[→ R
r π r
1000
A′ (r) = 2rπ − 2
r
1000
2rπ − 2 = 0
r
2r3 π = 1000
r
500
r = 3 ≈ 5,42 cm ⇒ M = 5,42 cm♦
π

4.12. Exponenciális egyenletek


Az exponenciális egyenletek három nagy tı́pusba sorolhatók. Azért érdemes ezekről külön
beszélni, mert mindhárom tı́pust más lépésekkel kell megoldani, viszont az egy tı́pusba tar-
tozókat mindig ugyanúgy.

4.12.1. I. tı́pusú
Az első tı́pust onnan lehet felismerni, hogy az egész egyenletben egy közös szám tetszőleges
hatványai szerepelnek az alapokban, és ezek között mindig szorzás vagy osztás van. Beazo-
nosı́tjuk mi ez a közös szám, és a hatványozás azonosságai segı́tségével eljutunk addig a pontig,
hogy csak egy darab hatvány legyen mindkét oldalon. Ezután jön a ,,mondat”, majd a ma-
radékot megoldjuk.

72
4. fejezet

Példa 1 :
2
Oldjuk meg a valós számok halmazán: 3x = 27 · 9x

Mindenhol háromhatványok vannak, ezért mindent visszaı́runk egy darab háromhatványra.


2 x
3x = 33 · (32 )
2
3x = 32x+3
mivel az exp...
x2 = 2x + 3
x2 − 2x − 3 = 0
x1 = 3
x2 = −1♦

Példa 2 :
1 √ 8
Oldjuk meg a valós számok halmazán: · 2x+3 = √
4x+1 3
32x
Mindenhol kettőhatványok vannak most.

1 1 23
x+1 · (2x+3 ) 2 = h i 13
22 (25 )x
1 x+3 23
·2 2
5x =
22(x+1) 23
x+3 5x
2−2(x+1) · 2 x = 23− 3
x+3 5x
2−2x−2+ x = 23− 3
mivel az exp...
x+3 5x
−2x − 2 + = 3−
2 3
x = 21♦

Példa 3 :
32−3x √
3
4x+1
Oldjuk meg a valós számok halmazán: x+1
= 81 · 243x+6
9
Mindenhol háromhatványok vannak most is.

32−3x h i1
4 4x+1 5 x+6 3
x+1 = (3 ) · (3 )
32
32−3x 4(4x+1)
5(x+6)
= 3 · 3 3
32(x+1)
5(x+6)
32−3x−2(x+1) = 34(4x+1)+ 3
5x+30
3−5x = 316x+4+ 3

mivel az exp...
5x + 30
−5x = 16x + 4 +
3
21
x = − ♦
34

73
4. fejezet

Példa 4 :
Oldjuk meg a valós számok halmazán: 3x + 3x+1 + 3x+2 = 7x + 7x+1 + 7x+2

Nevezhetjük az első tı́pus speciális esetének azt, amikor két olyan szám hatványai szerepel-
nek az egyenletben, amiknek semmi közük egymás hatványaihoz. Ebben az esetben nem olyan
látványos, hogy mi ez a közös szám. A trükk itt az, hogy egy oldalra kell rendezni az x-es
tagokat, a számokat a másik oldalra, hasonlóan, mint egy egyszerű elsőfokú egyenletben. Majd
logaritmussal folytatni.

3x + 3x · 3 + 3x · 9 = 7x + 7x · 7 + 7x · 49
13 · 3x = 57 · 7x
3x 57
x
=
7 13
 x
3 57
=
7 13
 x
3 57
lg = lg
7 13
3 57
x lg = lg
7 13
57
lg
x = 13 ≈ −1,7445♦
3
lg
7

4.12.2. II. tı́pusú


A második tı́pusnál az egész egyenletben ugyanaz a szám szerepel az alapokban, de ezek között
összeadás vagy kivonás van. Illetve általában a másik oldalon szereplő szám nem ennek a
számnak a hatványa. A hatványozás azonosságai és a műveletek segı́tségével eljutunk addig
a pontig, hogy csak egy darab hatvány legyen az adott oldalon. (Gyakori hiba szokott lenni itt
a műveletek helytelen sorrendje. Figyeljünk erre is és az egyneműségre is!)

Példa 1 :
Oldjuk meg a valós számok halmazán: 2x+3 − 2x = 112

2x · 23 − 2x = 112
8 · 2x − 2x = 112
7 · 2x = 112
2x = 16
mivel az exp...
x = 4♦

Példa 2 :
Oldjuk meg a valós számok halmazán: 2 · 3x+3 − 5 · 3x−2 = 1443

74
4. fejezet

3x
2 · 3x · 33 − 5 · = 1443
32
3x
54 · 3x − 5 · = 1443
9
486 · 3x − 5 · 3x = 12 987
481 · 3x = 12 987
3x = 27
mivel az exp...
x = 3♦

Példa 3 :
Oldjuk meg a valós számok halmazán: 25 · 5x+1 + 4 · 5x + 5x−1 = 646

5x
25 · 5x · 5 + 4 · 5x + = 646
5
5x
125 · 5x + 4 · 5x + = 646
5
625 · 5 + 20 · 5 + 5x
x x
= 3230
646 · 5x = 3230
5x = 5
mivel az exp...
x = 1♦

4.12.3. III. tı́pusú


A harmadik tı́pusú egyenlet a másodfokúra visszavezethető. Kétféleképp nézhet ki.
Első lehetőség: a hatványalapok azonosak, a kitevők egymás kétszeresei.
Második lehetőség: a hatványalapok egymás négyzetei, a kitevők azonosak.

Példa 1 :
Oldjuk meg a valós számok halmazán: 32x + 2 · 3x − 3 = 0

Legyen y = 3x

y 2 + 2y − 3 = 0
y1 = 1
y2 = −3
Visszahelyettesı́tve:

3x = 1 ⇒ x = 0
3x = −3 nincs megoldás
mivel az exponenciális függvény szigorúan monoton. . . ♦

Példa 2 : √ √
Oldjuk meg a valós számok halmazán: 4 x − 3 · 2 x + 2 = 0

Kikötés:
x≥0

75
4. fejezet


x
Legyen y = 2

y 2 − 3y + 2 = 0
y1 = 1
y2 = 2
Visszahelyettesı́tve:
√ √
2√x = 1 ⇒ x = 0 ⇒ x = 0

2 x = 2⇒ x=1⇒x=1
mivel az exponenciális függvény szigorúan monoton. . . ♦

Példa 3 :
Oldjuk meg a valós számok halmazán: 25lg x = 5 + 4 · 5lg x

Kikötés:
x>0

Legyen y = 5lg x

y2 = 5 + 4y
y 2 − 4y − 5 = 0
y1 = 5
y2 = −1
Visszahelyettesı́tve:

5lg x = 5 ⇒ lg x = 1 ⇒ x = 10
5lg x = −1 nincs megoldás
mivel az exponenciális függvény szigorúan monoton... ♦

Példa 4 :
Oldjuk meg a valós számok halmazán: 3lg tg x + 3lg ctg x = 2

Nem minden feladatban látszik azonnal, hogy másodfokúra visszavezethető, időnként előtte
kicsit alakı́tgatni kell.

Kikötések:
π
x ̸= + kπ
2
x ̸= 0 + kπ
tg x > 0
ctg x > 0
1
3lg tg x + 3lg tg x = 2
−1
3lg tg x + 3lg (tg x) = 2
3lg tg x + 3− lg tg x = 2
1
3lg tg x + lg tg x = 2
3
lg tg x 2

3 + 1 = 2 · 3lg tg x

76
4. fejezet

Legyen y = 3lg tg x

y 2 + 1 = 2y
y 2 − 2y + 1 = 0
y = 1
Visszahelyettesı́tve:

3lg tg x = 1 ⇒ lg tg x = 0 ⇒ tg x = 1 ⇒
π
x = + kπ, ahol k ∈ Z
4
A kikötéseket megoldani túl hosszú, a klasszikus ellenőrzés gyorsabb most.♦

4.13. Logaritmusos egyenletek – új alapra áttérés


Amikor új alapra kell áttérni, mindig szám legyen az alapban és érdemes egy kisebb számú
alapot választani, mert akkor a nevezők egész számok lesznek.

Példa 1 :
Oldjuk meg a valós számok halmazán: log2 x + log4 x + log16 x = 7

Kikötés:
x>0
Mindent kettesalapú logaritmusra ı́runk át.
log2 x log2 x
log2 x + + = 7
log2 4 log2 16
log2 x log2 x
log2 x + + = 7
2 4
4 log2 x + 2 log2 x + log2 x = 28
7 log2 x = 28
log2 x = 4
x = 16♦

Példa 2 :
Oldjuk meg a valós számok halmazán: log32 x + log8 x = 8

Kikötés:
x>0
Mindent kettesalapú logaritmusra ı́runk át, mert a 32 és a 8 is kettőhatvány.

77
4. fejezet

log2 x log2 x
+ = 8
log2 32 log2 8
log2 x log2 x
+ = 8
5 3
3 log2 x + 5 log2 x = 120
8 log2 x = 120
log2 x = 15
x = 32768♦

Amikor x-alapú logaritmus szerepel, akkor is számra érdemes átı́rni, de valószı́nűleg másodfokú
egyenletre vezet.

Példa 3 :
Oldjuk meg a valós számok halmazán: 2 logx 25 − 3 log25 x = 1

Kikötés:
x>0
x ̸= 1

Mindent ötösalapú logaritmusra ı́runk át.


log5 25 log5 x
2· −3· = 1
log5 x log5 25
4 3 log5 x
− = 1
log5 x 2
Legyen y = log5 x
4 3y
− = 1
y 2
8 − 3y 2 = 2y
0 = 3y 2 + 2y − 8
4
y1 =
3
y2 = −2
Visszahelyettesı́tve:
4 √
log5 x = ⇒ x = 3 625
3
1
log5 x = −2 ⇒ x = ♦
25
Példa 4 : p √
Oldjuk meg a valós számok halmazán: logx 5x · log5 x = 2

Kikötés:
x>0
x ̸= 1
logx 5x ≥ 0

78
4. fejezet

Mindent ötösalapú logaritmusra ı́runk át.



r
log5 5x
· log5 x = 2
log5 x

r
log5 5 + log5 x
· log5 x = 2
log5 x

r
1 + log5 x
· log5 x = 2
log5 x
Legyen y = log5 x

r
1+y
·y = 2
y
1+y 2
·y = 2
y
(1 + y) · y = 2
y2 + y − 2 = 0
y1 = 1
y2 = −2
Visszahelyettesı́tve:

log5 x = ⇒ x=5
1
1
log5 x = −2 ⇒ x = hamis megoldás
25
A logaritmusos kikötés nehézsége miatt, és a négyzetre emelés miatt is, az ellenőrzés a célszerűbb
és kijön, hogy a második megoldás hamis.♦

4.14. Egyenletrendszerek trigonometrikus egyenlettel


Ha az egyenletrendszer egyik tagja trigonometrikus egyenlet, akkor érdemes a másikkal kezdeni
és ott kifejezni az egyik ismeretlent a másikkal, majd azt berakni a trigonometrikusba, mert
valószı́nűleg egyszerűbb addı́ciós tétel lesz, vagy könnyű egyenlet. Ha a trigonometrikussal
kezdjük, akkor a periódus miatt sok megoldás lehet.

Példa 1 : 
logx y + logy x = 2
Oldjuk meg a valós számpárok halmazán:
sin (2x + 3y) + sin (4x + y) = 1

Kikötések:
x; y > 0
x ̸= 1
y ̸= 1

Az első egyenletben észrevesszük, hogy logx y és logy x egymás reciprokai a logaritmus egyik
azonossága alapján, ı́gy biztos, hogy logx y = logy x = 1, mert tudjuk, hogy csak az 1 az a
szám, amit ha összeadunk a saját reciprokával 2-t kapunk. Ha ez nem jut eszünkbe, akkor sincs
1
semmi baj. Akkor logy x = -t alkalmazzuk, és másodfokúra vezető egyenlet lesz, amiből
logx y
csak az 1 jön ki megoldásként. Tehát x = y a logaritmus definı́ciója alapján.

79
4. fejezet

Ezt behelyettesı́tve a trigonometrikus egyenletbe:

sin (2x + 3y) + sin (4x + y) = 1


sin (5x) + sin (5x) = 1
sin (5x) = 0,5
↙↘
5x = 30◦ + k · 360◦ 5x = 150◦ + l · 360◦
x = 6◦ + k · 72◦ x = 30◦ + l · 72◦
π 2π π 2π
x=y = +k x=y = +l
30 5 6 5
Most k; l ∈ N a logaritmus kikötései miatt. ♦

Példa 2 : 
logx (x2 y 3 ) + logy (x3 y) = 9
Oldjuk meg a valós számpárok halmazán:
cos (x + y) + cos (x − y) = 0

Kikötés:
x; y > 0
x ̸= 1
y ̸= 1

Ebben a feladatban is az első egyenlettel kezdünk el foglalkozni.

logx x2 + logx y 3 + logy x3 + logy y = 9


2 logx x + 3 logx y + 3 logy x + logy y = 9
2 + 3 logx y + 3 logy x + 1 = 9
logx y + logy x + 1 = 2

Megint kilyukadtunk ahhoz a gondolathoz, amit az előző példánál is láttunk. Vagyis innen kap-
juk, hogy x = y. Most foglalkozunk csak a második egyenlettel.

cos (x + y) + cos (x − y) = 0
cos (2x) + cos 0 = 0
cos (2x) = −1
↙↘
2x = 180◦ + k · 360◦
x = 90◦ + k · 180◦ A másik megoldás is 180◦ ,
π tehát felesleges felı́rni kétszer.
x=y = + kπ
2
Most is k ∈ N a logaritmus kikötései miatt.♦

80
4. fejezet

4.15. Értékkészlet-vizsgálat egyenleteknél


Ha egy egyenletben láthatóan ,,nem megszokott” kifejezések szerepelnek, lehet, hogy az érték-
készlet vizsgálata lesz a megoldás. Ebben az esetben a két oldal értékkészletét kell megvizsgálni
és vagy nincs közös részük, vagy csak egy pontban találkoznak, ı́gy a megoldás leszűkı́thető
pillanatok alatt.

Példa 1 :
1
Oldjuk meg a pozitı́v számok halmazán: x + = sin x
x
1
A bal oldalról tudjuk a nevezetes x+ ≥ 2 egyenlőtlenség alapján, hogy az értékkészlete [2; ∞[.
x
A jobb oldal korlátos, értékkészlete [−1; 1].
Vagyis az értékük nem lehet egyenlő, mert nincs metszetük, ı́gy nincs megoldás.♦

Példa 2 :
1
Oldjuk meg a pozitı́v számok halmazán: x + = 2 sin x
x
A bal oldal értékkészlete [2; ∞[.
A jobb oldalé viszont most [−2; 2].
A két értékkészletnek egyetlenegy metszéspontja van, a 2. Tehát mindkét oldal értéke 2 kell
π
legyen, egyszerre. Ez a bal oldal esetében x = 1-nél teljesül, a jobb oldalnál viszont x = +
2
+ k2π esetén. Így megint nincs megoldás, mert nem ugyanarra az x-re teljesül az értékkészlet
feltétele.♦

Példa 3 :
Oldjuk meg a valós számok halmazán: cos x = x2 + 1

A bal oldal értékkészlete [−1; 1].


A jobb oldal értékkészlete [1; ∞[.
A két értékkészletnek egyetlenegy metszéspontja van, az 1. Tehát mindkét oldal értékének 1-nek
kell lennie, megint egyszerre. Ez mindkét oldal esetében x = 0-ra teljesül, ı́gy most van meg-
oldás, az x = 0. Más megoldás nincs, hiszen az értékkészletek más esetben nem érintkeznek.♦

Példa 4 :
Oldjuk meg a valós számok halmazán: sin x = x2 + 1

Ez a feladat hasonló az előzőhöz. A bal oldal értékkészlete [−1; 1].


A jobb oldal értékkészlete [1; ∞[.
Megint a két értékkészletnek egyetlenegy metszéspontja van, az 1. A jobb oldalnál ugyancsak
π
x = 0 esetén adódik ez, mı́g a bal oldalnál x = + k2π esetén. Így nincs megoldás, mert nem
2
ugyanarra az x-re teljesül az értékkészlet feltétele.♦

Példa 5 :
Oldjuk meg a valós számok halmazán: |2x − 5| + 1 = sin2 x

A bal oldal értékkészlete [1; ∞[.


A jobb oldal értékkészlete [0; 1].
5
Megint egyetlenegy metszéspontja van a két értékkészletnek, az 1. Ez a bal oldalnál x =
2
π
esetén igaz, a jobb oldalnál pedig x = + kπ esetén, ı́gy megint nincs megoldás.♦
2

81
5. fejezet

5. Egyenlőtlenségek
Az egyenlőtlenségek bizonyos szempontból nehezebbek, mint az egyenletek. Vannak lépések,
melyeket tilos megtenni, pedig az egyenleteknél szabad volt. De olyan lépések is vannak, ami-
ket meg most könnyelműen elengedhetünk. Az egyenlőtlenségeket nem lehet ellenőrizni, ezért
ezzel ne is erőlködjünk, hiszen a végeredmény egy intervallum és nem egy konkrét szám. Egy
intervallumot pedig nem lehet ellenőrizni. Rossz hı́r viszont, hogy a kikötéseket nem tudjuk
megúszni. Minden egyenlőtlenségnél (sajnos) a kikötéseket meg kell oldani, és a legvégén, a
végeredményként kapott intervallumot mindig le kell egyeztetni a kikötések intervallumaival.
A közös metszet lesz a végső megoldás.

5.1. Negatı́v számmal szorzás, osztás


Közismert, hogy ha negatı́v számmal szorzunk vagy osztunk egy egyenlőtlenséget, megváltozik
a relációs jel iránya.

Példa :
Oldjuk meg a valós számok halmazán: 5(3x − 4) > 13x + 16

15x − 20 > 13x + 16


−36 > −2x
18 < x

A végeredmény x ∈]18; ∞[. ♦

Természetesen sokkal egyszerűbb, ha az elsőfokú egyenlőtlenségeket mindig úgy rendezzük, hogy


pozitı́v legyen az ismeretlen együtthatója, mert akkor erre nem kell odafigyelni, de azért fontos
tisztában lenni vele.

5.2. Ismeretlent tartalmazó taggal szorzás, osztás


Egy egyenlőtlenséget tilos ismeretlent tartalmazó taggal beszorozni, vagy osztani. (Kivéve, ha
tudjuk, milyen előjelű.)

Példa 1 :
Oldjuk meg a valós számok halmazán: x3 > x2

Nem osztunk x2 -tel, hanem nullára rendezünk és kiemelünk.

x3 − x2 > 0
2
x (x − 1) > 0

Mivel x2 biztosan nemnegatı́v, ezért x − 1-nek pozitı́vnak kell lennie, hogy a szorzat pozitı́v
legyen, azaz x > 1
A végeredmény x ∈]1; ∞[. ♦

Példa 2 :
4
Oldjuk meg a valós számok halmazán: >0
x−3
4
Két esetben lehetséges, hogy a tört pozitı́v legyen.
x−3

82
5. fejezet

pozitı́v negatı́v
alakú alakú
pozitı́v negatı́v
A nevező: x − 3 > 0 ⇒ x > 3 Mivel a számláló egy pozitı́v szám, ez az ág lehetetlen.

A végeredmény x ∈]3; ∞[. ♦

Példa 3 :
9 − 3x
Oldjuk meg a valós számok halmazán: ≤0
2x + 4
9 − 3x
Két esetben lehetséges, hogy a tört negatı́v legyen.
2x + 4

pozitı́v negatı́v
alakú alakú
negatı́v pozitı́v
A számláló: 9 − 3x ≥ 0 és A számláló: 9 − 3x ≤ 0 és
a nevező: 2x − 4 < 0 a nevező: 2x − 4 > 0
3 ≥ x és x < −2 3 ≤ x és x > −2

Mivel a két ág közül valamelyik teljesül, azok uniója kell.


A végeredmény x ∈ ]−∞; −2[ ∪ [3; ∞[. ♦

Példa 4 :
2n + 3
Milyen az an = sorozat monotonitása?
n+4
Vizsgáljuk az an ≤ an+1 egyenlőtlenséget.

2n + 3 2(n + 1) + 3

n+4 (n + 1) + 4
2n + 3 2n + 5

n+4 n+5
2n + 3 2n + 5

n+4 n+5
Most be lehet szorozni a nevezőkkel, mert ezek biztosan pozitı́v számok.
(2n + 3)(n + 5) ≤ (2n + 5)(n + 4)
2n2 + 13n + 15 ≤ 2n2 + 13n + 20
15 ≤ 20

Ez igaz, tehát a sorozat monoton nő.♦

83
5. fejezet

5.3. A másodfokú egyenlőtlenség


A másodfokú egyenlőtlenségekre külön figyelmet fordı́tunk. Nemcsak azért, mert odafigyelést
igényelnek, hanem azért is, mert sok egyenletnek kikötései is lehetnek egyben. Egy másodfokú
egyenlőtlenséget négy lépésben kell megoldani.
I. Ha nincs nullára rendezve, akkor átrendezzük, majd megoldjuk, mint egyenletet és megkap-
juk a gyököket.
II. Ábrázoljuk a másodfokú függvényt. (Ilyenkor nem kell tökéletesnek lennie. Az a fontos, hogy
a két zérushelyen menjen át és megfelelő irányba álljon.)
III. A függvénynek a megfelelő részét megjelöljük.(Az egész fejezetben piros, ha pozitı́v, kék,
ha negatı́v.)
IV. A megjelölt részt levetı́tjük az x-tengelyre. (Az egész jegyzetben zöld szı́nű.)

A következő táblázat összegzi, hogyan néznek ki a parabolák a főegyüttható előjelét és a


zérushelyek számát figyelembe véve. A II. pontban az ábrázolásnál ezt észben kell tartani.

D>0 D=0 D<0


két zérushely van egy zérushely van nincs zérushely

a>0
felülről
nyitott

a<0
alulról
nyitott

Szemmel láthatóan a harmadik oszlopban a parabola nem metszi az x-tengelyt, ezért teljes
négyzetté alakı́tás nélkül a pontos helyzetét megállapı́tani nem tudjuk. Viszont ez nem is fon-
tos az egyenlőtlenség megoldása szempontjából.

84
5. fejezet

Példa 1 :
Oldjuk meg a valós számok halmazán: x2 − x − 2 ≥ 0

I. A megoldóképlettel a megoldások x = 2; x = −1.


II. −1 és 2 a két zérushely, és a parabola felfelé áll.
III. Mivel x2 − x − 2 ≥ 0, az x-tengely feletti rész és maga az x-tengely kell.
IV. Rávetı́tve az x-tengelyre a végeredmény x ∈ ]−∞; −1[ ∪ [2; ∞[. ♦

Példa 2 :
Oldjuk meg a valós számok halmazán: x2 − 10x + 24 < 0

I. A megoldóképlettel a megoldások x = 4; x = 6.
II. 4 és 6 a két zérushely, és a parabola felfelé áll.
III. Mivel x2 − 10x + 24 < 0, az x-tengely alatti rész kell.
IV. Rávetı́tve az x-tengelyre a végeredmény x ∈ ]4; 6[. ♦

85
5. fejezet

Példa 3 :
Oldjuk meg a valós számok halmazán: x2 < 4

Csak azért, mert ez a feladat rövidnek tűnik első látásra, még nem szabad egyik lépést sem
elhagyni. Pláne tilos gyököt vonni!
I. Nullára rendezzük, ı́gy x2 − 4 < 0. A megoldások x = 2; x = −2.
II. −2 és 2 a két zérushely, és a parabola felfelé áll.
III. Mivel x2 − 4 < 0, az x-tengely alatti rész kell.
IV. Rávetı́tve az x-tengelyre a végeredmény x ∈ ]−2; 2[. ♦

Példa 4 :
Oldjuk meg a valós számok halmazán: −2x2 − x + 1 ≥ 0
1
I. A megoldóképlettel a megoldások x = ; x = −1.
2
1
II. és −1 a két zérushely, és a parabola lefelé áll.
2
III. Mivel −2x2 − x + 1 ≥ 0, az x-tengely feletti rész és maga az x-tengely kell.
 
1
IV. Rávetı́tve az x-tengelyre a végeredmény x ∈ −1; .♦
2

86
5. fejezet

Példa 5 :
Oldjuk meg a valós számok halmazán: x2 − 6x + 10 ≥ 0

I. Amikor megpróbáljuk megoldani a megoldóképlettel, rájövünk, hogy nincsenek valós meg-


oldásai. De függvény természetesen van, csak ez azt jelenti, hogy nem ér hozzá az x-tengelyhez.
II. Vagyis ebben az esetben elegendő egy olyan felfelé álló parabolát rajzolni, ami valahol az
x-tengely felett ,,lebeg”.
III. Mivel x2 − 6x + 10 ≥ 0, az x-tengely feletti rész és maga az x-tengely kell, de az egész
függvény ott helyezkedik el.
IV. Ha ezt rávetı́tjük az x-tengelyre akkor a teljes valós számegyenest kapjuk, vagyis a megoldás
x ∈ R.♦

Példa 6 :
Oldjuk meg a valós számok halmazán: −x2 − 2x − 1 ≥ 0

I. A megoldóképlettel a megoldás x = −1.


II. x = −1 az egyetlen zérushely, és a parabola lefelé áll.
III. Mivel −x2 − 2x − 1 ≥ 0, az x-tengely feletti rész és maga az x-tengely kell.
IV. Ebben a feladatban a grafikonnak csak egy pontja van az x-tengely felett vagy rajta, ı́gy a
végeredmény most x = −1.♦

87
5. fejezet

5.4. A gyökös egyenlőtlenség


A gyökös egyenlőtlenségekre is külön figyelmet kell fordı́tani, de itt a kikötések miatt.

Példa 1 : √
Oldjuk meg a valós számok halmazán: 3x − 3 ≤ 3

Kikötés:
3x − 3 ≥ 0 ⇒ x ≥ 1

Négyzetre emeljük.

3x − 3 ≤ 9
x ≤ 4

A végeredmény x ∈ [1; 4].♦

Példa 2 : √
Oldjuk meg a valós számok halmazán: 5 − 4x ≥ 3

Kikötés:
5
5 − 4x ≥ 0 ⇒ ≥ x
4
Négyzetre emeljük.

5 − 4x ≥ 9
−1 ≥ x

A végeredmény x ∈ ]∞; −1].♦

Példa 3 : √
Oldjuk meg a valós számok halmazán: x + 3 ≤ x + 1

Ha ismeretlen van a másik oldalon is, akkor arra az oldalra is muszáj elvégezni a kikötést.

Kikötések:
x + 3 ≥ 0 ⇒ x ≥ −3
x + 1 ≥ 0 ⇒ x ≥ −1

88
5. fejezet

Négyzetre emeljük.

x + 3 ≤ x2 + 2x + 1
0 ≤ x2 + x − 2
x ≤ −2 vagy 1 ≤ x

A végeredmény x ∈ [1; ∞[ . ♦

Példa 4 : √
Oldjuk meg a valós számok halmazán: 3x − x2 < x − 4

Kikötések:
x−4≥0⇒x≥4
3x − x2 ≥ 0 ⇒ 0 ≤ x ≤ 3

Mivel a kikötéseknek metszete üres, ezért biztosan nincs megoldás, vagyis a feladathoz hozzá
sem kell fogni.♦

Példa 5 : √ √
Oldjuk meg a valós számok halmazán: 3x + 1 > 2 − x

Mivel most csak négyzetgyökök szerepelnek, azokról pedig tudjuk, hogy nemnegatı́v az érték-
készletük, ı́gy külön kikötés nem szükséges.

Kikötések:
1
3x + 1 ≥ 0 ⇒ x ≥ −
3
2−x≥0⇒2≥x
Négyzetre emeljük.

3x + 1 > 2 − x
1
x >
4



1
A végeredmény x ∈ ; 2 .♦
4

89
5. fejezet

5.5. Az abszolút értékes egyenlőtlenség


Ebben a fejezetben csak azokról az abszolút értékes egyenlőtlenségekről lesz szó, amikor a
másik oldalon egy szám szerepel. Ha ott is van ismeretlen, akkor a grafikus megoldást javaslom.
(2.4. fejezet)

5.5.1. Az |x| ≤ a tı́pusú


Ha |x| ≤ a, akkor − a ≤ x ≤ a, feltéve, hogy (a ≥ 0).

Ez egy ún. kétoldali egyenlőtlenség, vagyis a −a ≤ x és az x ≤ a egyenlőtlenségek egyben


felı́rva. Ezeknek egyszerre kell teljesülniük, közöttük logikai és művelet van.

Példa 1 :
Oldjuk meg a valós számok halmazán: 2|x| − 7 ≤ 1

|x| ≤ 4
−4 ≤ x ≤4

A végeredmény x ∈ [−4; 4].♦

Példa 2 :
Oldjuk meg a valós számok halmazán: |x − 3| ≤ 4

−4 ≤ x − 3 ≤ 4
Ilyenkor szabad, hogy a két egyenlőtlenséghez egyszerre adunk hozzá hármat
−1 ≤ x ≤7

A végeredmény x ∈ [−1; 7].♦

Példa 3 :
Oldjuk meg a valós számok halmazán: |2x + 1| < 3

−3 < 2x + 1 < 3
−4 < 2x <2
−2 < x <1

A végeredmény x ∈ ]−2; 1[.♦

Példa 4 :
Oldjuk meg a valós számok halmazán: 5|4 − 3x| < 20

|4 − 3x| < 4
−4 < 4 − 3x <4
−8 < −3x <0
8
> x >0
3
 
8
A végeredmény x ∈ 0; .♦
3

90
5. fejezet

5.5.2. Az |x| ≥ a tı́pusú


Ha |x| ≥ a, akkor x ≤ −a vagy a ≤ x, feltéve, hogy (a ≥ 0).

Ez a fajta egyenlőtlenség nem ı́rható fel egyben, muszáj szétszedni őket két részre. Ezeknek
viszont nem kell egyszerre teljesülniük, közöttük logikai vagy művelet van.

Példa 1 :
Oldjuk meg a valós számok halmazán: 5|x| − 3 ≥ 12

|x| ≥ 3
x ≤ −3 vagy 3 ≤ x

A végeredmény x ∈ ]−∞; −3] ∪ [3; ∞[.♦

Példa 2 :
Oldjuk meg a valós számok halmazán: |x + 6| > 7

x + 6 < −7 vagy 7 < x + 6


x < −13 vagy 1 < x

A végeredmény x ∈ ]−∞; −13[ ∪ ]1; ∞[.♦

Példa 3 :
Oldjuk meg a valós számok halmazán: |2x − 3| > 11

2x − 3 < −11 vagy 11 < 2x − 3


x < −4 vagy 7 < x

A végeredmény x ∈ ]−∞; −4[ ∪ ]7; ∞[.♦

5.6. Az exponenciális egyenlőtlenség


Exponenciális egyenlőtlenségeknél arra kell odafigyelni, hogy a hatványalap, amit elhagyunk a
mondattal egynél nagyobb, vagy kisebb. Ha nagyobb, nincs változás, ha kisebb, megváltozik a
relációs jel iránya.

Példa 1 :
Oldjuk meg a valós számok halmazán: 31−x ≥ 81

31−x ≥ 34
mivel az exp. fv. szig. mon. nő
1−x ≥ 4
−3 ≥ x

A végeredmény x ∈ ]−∞; −3].♦

91
5. fejezet

Példa 2 :  x+4 r  2x+3


2 4 3
Oldjuk meg a valós számok halmazán: > 5 ·
3 9 2
 x+4   25  −(2x+3)
2 2 2
> ·
3 3 3
 x+4   25 −2x−3
2 2
>
3 3
mivel az exp. fv. szig. mon. csökken
2
x+4 < − 2x − 3
5
11
x < −
5
 
11
A végeredmény x ∈ −∞; − .♦
5

Példa 3 :
2
Oldjuk meg a valós számok halmazán: 2x −2x−1 ≤ 4

2
2x −2x−1 ≤ 22
mivel az exp. fv. szig. mon. nő
x2 − 2x − 1 ≤ 2
x2 − 2x − 3 ≤ 0

Az 5.3. fejezet alapján a végeredmény x ∈ [−1; 3].♦

Példa 4 :
Oldjuk meg a valós számok halmazán: 0,253−|x| ≤ 16

0,253−|x| ≤ 0,25−2
mivel az exp. fv. szig. mon. csökken
3 − |x| ≥ −2
5 ≥ |x|

Az 5.5.1. fejezet alapján a végeredmény x ∈ [−5; 5].♦

Példa 5 : √
Oldjuk meg a valós számok halmazán: 5 x+1 < 5x−1

mivel az√exp. fv. szig. mon. nő


x+1 < x−1

Az 5.4. fejezet alapján ábrázoljuk a két oldalon található függvényeket koordináta-rendszerben


és leolvassuk, mikor igaz az egyenlőtlenség.
A végeredmény x ∈ ]3; ∞[.♦

92
5. fejezet

5.7. A logaritmusos egyenlőtlenség


Logaritmusos egyenlőtlenségeknél már van kikötés. A megoldás gondolatmenete viszont ugyan-
az, mint az exponenciális egyenlőtlenségeké.

Példa 1 :
Oldjuk meg a valós számok halmazán: log2 (x − 3) ≥ log2 (2x − 10)

Kikötések:
x−3>0⇒x>3
2x − 10 > 0 ⇒ x > 5

mivel a log. fv. szig. mon. nő


x − 3 ≥ 2x − 10
7 ≥ x
A kikötéseket és az eredményt egy közös számegyenesen ábrázoljuk, hogy lássuk mi a végső
megoldás.

A végeredmény x ∈ ]5; 7].♦

Példa 2 :
Oldjuk meg a valós számok halmazán: lg (2x − 11) ≤ lg (x2 − 2x − 8)

Kikötések:
2x − 11 > 0 ⇒ x > 5,5
x2 − 2x − 8 > 0 ⇒ x < −2 vagy 4 < x

mivel a log. fv. szig. mon. nő


2x − 11 ≤ x2 − 2x − 8
0 ≤ x2 − 4x + 3
x ≤ 1 vagy 3≤x
A kikötéseket és az eredményt egy közös számegyenesen ábrázoljuk, hogy lássuk mi a végső
megoldás.

A végeredmény x ∈ ]5,5; ∞[.♦

93
5. fejezet

Példa 3 :
Oldjuk meg a valós számok halmazán: log 1 (x + 4) > −2
3

Kikötés:
x + 4 > 0 ⇒ x > −4

log 1 (x + 4) > log 1 9


3 3
mivel a log. fv. szig. mon. csökken
x+4 < 9
x < 5
A kikötéseket és a végeredményt egy közös számegyenesen ábrázoljuk, hogy lássuk mi a végső
megoldás.

A végeredmény x ∈ ]−4; 5[.♦

5.8. A trigonometrikus egyenlőtlenség


Trigonometrikus egyenlőtlenségekből csak az egyszerűbb szinuszosokat és a koszinuszosokat
tárgyaljuk most. Mindegyiket az egységkör felrajzolásával oldjuk meg. A szögfüggvények kiter-
jesztéséből tudjuk, hogy az egységkörön mozgó pont koordinátái megegyeznek az x-tengellyel
bezárt pozitı́v irányú forgásszög koszinuszával és szinuszával.

Példa 1 :
1
Oldjuk meg a valós számok halmazán: sin x ≥
2
1
Megrajzoljuk az egységkört, és vı́zszintesen az y = egyenest. Ezek két pontban találkoznak,
2
1
30◦ -nál és 150◦ -nál. Mivel a sin x ≥ , ezért az egyenes feletti rész kell az egységkörből.
2

 
π 5π
A végeredmény x ∈ + k2π; + k2π , ahol k ∈ Z.♦
6 6

94
5. fejezet

Példa 2 : √
2
Oldjuk meg a valós számok halmazán: cos x < −
2

2
Megrajzoljuk megint az egységkört, és az x = − egyenest függőlegesen. Ezek 135◦ -nál és 225◦
√ 2
2
◦-nál találkoznak. Mivel a cos x < − , ezért az egyenestől balra lévő rész kell az egységkörből.
2

 
3π 5π
A végeredmény x ∈ + k2π; + k2π , ahol k ∈ Z.♦
4 4

Mindkét feladatban alkalmaztuk azt, amit az egyenlőtlenségek grafikus megoldásánál is, vagyis
az egységkörből azt a darabot kell nézni, ahol a konstanshoz képest elhelyezkedik.

Zárásként szeretném hozzátenni, hogy igyekeztem több módszert is mutatni arra, hogyan érde-
mes nekifogni egy egyenletnek vagy egyenlőtlenségnek, de a vizsgázó diák dönti el, hogy melyik
eljárást alkalmazza. Néha a grafikus módszer tud elbonyolódni, néha az algebrai. Néha a klasszi-
kus ellenőrzés jó az egyenleteknél, néha a kikötésekkel érdemes foglalkozni. A lehetőségeket
érdemes észben tartani, és ha az egyik eljárás befuccsol, a másikkal próbálkozni.

95
6. fejezet

6. Sı́kgeometria, térgeometria
6.1. Számolás a háromszög területével
A háromszög területének kiszámolásával könnyen meghatározható a háromszög bármelyik ma-
gassága, a beı́rható kör sugara vagy a körülı́rható kör sugara is. A módszer lényege az, hogy
valahogyan kiszámoljuk a területet, és utána ezt egyenlővé tesszük azzal a háromszögterülettel,
amelyikben az ismeretlen szerepel. Négyszögekre is alkalmazható az eljárás.

Példa 1 :
Egy derékszögű háromszög befogói 3 cm és 4 cm. Mennyi az átfogóhoz tartozó magasság hossza?

A háromszög területe:
3·4
T = = 6 cm2
2

Pitagorasz-tétel:

32 + 42 = c2
c = 5 cm
És most felı́rjuk a másik területképletet:
c·m
T =
2
5m
6 =
2
m = 2,4 cm♦

Példa 2 :
Egy háromszög oldalai 42 cm; 40 cm; 26 cm. Mennyi a beı́rt kör sugara?

42 + 40 + 26
s= = 54
2

96
6. fejezet

A háromszög területe Héron-képlettel:


p
T = s(s − a)(s − b)(s − c)

T = 54 · 12 · 14 · 28
T = 504 cm2

Most egyenlővé tesszük a másik területképlettel:

T = rs
504 = 54r
28
r = cm♦
3

Példa 3 :
Egy háromszög oldalai 13 cm; 14 cm; 15 cm. Mennyi a köré ı́rt kör sugara?

13 + 14 + 15
s= = 21
2

A háromszög területe Héron-képlettel:


p
T = s(s − a)(s − b)(s − c)

T = 21 · 8 · 7 · 6
T = 84 cm2

Megint egyenlővé tesszük a másik területképlettel:


abc
T =
4R
13 · 14 · 15
84 =
4R
R = 8,125 cm♦

Példa 4 :
Egy rombusz oldala 4,5 cm, egyik átlója 5,4 cm. Számı́tsuk ki a rombusz magasságát.

Pitagorasz-tétel:

4,52 = x2 + 2,72
x = 3,6 cm

A másik átló f = 7,2 cm hosszú ı́gy. A rombusz területe:


ef
= 19,44 cm2
2

Felı́rjuk a másik területképlettel is:

97
6. fejezet

T = am
19,44 = 4,5m
a = 4,32 cm♦

Ez a gondolatmenet hasznos lehet térgeometriában is, ha mondjuk egy testbe ı́rt gömb sugarát
szeretnénk meghatározni.

Példa 5 :
Az egyik csokoládégyárban egy újfajta, kúp alakú desszertet gyártanak. A desszert csokoládéból
készült váza olyan, mint egy tölcsér. A kisebb kúp adatai: alapkörének sugara 1 cm, magassága
2,5 cm hosszú. Az elkészült csokoládéváz üreges belsejébe marcipángömböt helyeznek, ezután
egy csokoládéból készült vékony körlemezzel lezárják a kúpot. Hány cm a sugara a lehető leg-
nagyobb méretű ilyen marcipángömbnek?

A lehető legnagyobb gömb a kúp beı́rható gömbje. A feladatot egy vágással visszavezetjük egy
kétdimenziós ábrára, vagyis a kúpba ı́rt gömbből egyenlő szárú háromszög beı́rt köre lesz.
A háromszög alapja 2 cm, magassága 2,5 cm.
Pitagorasz-tétel:

b2 = 12 + 2,52
b ≈ 2,69 cm

Most tudjuk csak kiszámolni a kerület felét:


2 + 2,69 + 2,69
s= = 3,69
2

A háromszög területe:
2 · 2,5
T = = 2,5 cm2
2
98
6. fejezet

Megint egyenlővé tesszük a másik területképlettel:

T = rs
2,5 = 3,69r
r ≈ 0,68

Megközelı́tőleg 0,68 cm a beı́rt kör sugara, vagyis a kúpba ı́rt gömb sugara is.♦

Más témakörben is eszünkbe juthat a gondolatmenet.

Példa 6 :
Egy derékszögű háromszög három magassága egy növekvő mértani sorozat első három eleme.
Bizonyı́tsuk be, hogy az átfogó is eleme a sorozatnak.

Felı́runk a háromszögre két területképletet:

mq · mq 2
T =
2
mc
T =
2
Most ezeket egyenlővé tesszük:

mq · mq 2 mc
=
2 2
2 3
m q = mc
mq 3 = c

Vagyis az átfogó a mértani sorozat következő tagja.♦

6.2. A Pitagorasz-tétel
A Pitagorasz-tétellel kapcsolatban hajlamosak vagyunk azt hinni, hogy csak akkor lehet hasz-
nálni, ha tudunk két oldalt konkrétan, és akkor a harmadik kiszámolható. A tétel akkor is
alkalmazható, ha egy oldalt sem tudunk, viszont minden oldal ugyanazzal a betűvel van kife-
jezve.

99
6. fejezet

Példa 1 :
Egy
√ forgáskúp alapkörének átmérője egyenlő a kúp alkotójával. A kúp magasságának hossza
5 3 cm. Mekkora a kúp felszı́ne?

Pitagorasz-tétel:
√ 2
5 3 + r2 = (2r)2
3r2 = 75
r = 5 cm

A felszı́n A = r2 π + raπ = 75π ≈ 235,62 cm2 ♦

Példa 2 :
Két közös középpontú kör sugarának különbsége 8 cm. A nagyobbik körnek egy húrja érinti a
belső kört és hossza a belső kör átmérőjével egyenlő. Mekkorák a körök sugarai?

Elkészı́tjük a rajzot, és megállapı́tjuk, hogy R = r + 8.

100
6. fejezet

Pitagorasz-tétel:

(r + 8)2 = r2 + r2
r2 + 16r + 64 = 2r2
0 = r2 − 16r − 64
r1 = 19,31
r2 = −3,31 hamis megoldás

A két sugár 19,31 cm és 27,31 cm hosszú.♦

Példa 3 :
Egy egyenes
√ körkúp kiterı́tett palástja R sugarú félkörlap. Mekkora a kúp felszı́ne, ha a ma-
gasság 12 3?

Ismert, hogy a kiterı́tett kúppalást egy körcikket alkot. Ennek a sugara behajtás után egyenlő
lesz a kúp alkotójával. Azt is tudjuk, hogy a körcikk ı́vhossza pedig egyenlő lesz a kúp alapköré-
nek kerületével.

Ha a sugár R, akkor a félkörı́v hossza Rπ.


R
Ekkor a kúp alapkörének kerülete 2rπ, ami egyenlő Rπ-vel. Innen tudjuk, hogy r = .
2
Pitagorasz-tétel:
 2
√ 2 R
12 3 + = R2
2
R2
432 + = R2
4
R = 24 ⇒ r = 12

A felszı́n A = r2 π + raπ = 432π ≈ 1357,17 ♦

Példa 4 :
Egy derékszögű háromszög oldalai egy számtani sorozat szomszédos tagjai. A háromszög terüle-
te 150 cm2 . Mekkorák a háromszög oldalai?

101
6. fejezet

Pitagorasz-tétel:

(a − d)2 + a2 = (a + d)2

A háromszög területe:

a(a − d)
= 150
2

A két ismeretlen egy egyenletrendszert alkot. Kifejezve a tételből:

2a2 − 2ad + d2 = a2 + 2ad + d2


a2 − 4ad = 0
a(a − 4d) = 0
a = 4d

Ezt behelyettesı́tve a területképletbe:

a(a − d)
= 150
2
a(a − d) = 300
4d · 3d = 300
d = 5 ⇒ a = 20

A háromszög oldalai: 15 cm; 20 cm; 25 cm.♦

Példa 5 :
Egy egyenlő szárú háromszög szára 4 cm hosszú. A háromszög szára, alapja és az alaphoz tartozó
magassága, ebben a sorrendben, egy mértani sorozat egymást követő három tagja. Mekkora a
háromszög alapja?

A szár 4, az alap 4q, a magasság 4q 2 .

102
6. fejezet

Pitagorasz-tétel:
2
(2q)2 + (4q 2 ) = 42
4q 2 + 16q 4 = 16

Legyen p = q 2

4p + 16p2 = 16
p1 = 0,88
p2 = −1,13

Visszahelyettesı́tve:

q 2 = 0,88 ⇒ q = 0,94
q 2 = −1,13 hamis megoldás

A háromszög alapja 3,76 cm.♦

Példa 6 :
Egy derékszögű trapézban az alapok hossza 4 cm és 10 cm. Milyen hosszú a két szár, ha tudjuk,
hogy a trapéz érintőnégyszög?

A rajz felrajzolása után világosan látszik, hogy az AD szár és a CT magasság megegyezik a kör
átmérőjével, ezért hosszuk 2r. Mivel a trapéz érintőnégyszög, a szemben lévő BC szár hossza
14 − 2r.

103
6. fejezet

Pitagorasz-tétel:

(2r)2 + 62 = (14 − 2r)2


4r2 + 36 = 196 − 56r + 4r2
20
r = cm
7
40 58
Az AD szár cm, a BC szár cm hosszú.♦
7 7

6.3. A körszelet területe


Egy körszelet területét (ha a középponti szög kisebb, mint 180◦ ) úgy érdemes kiszámolni, hogy
a körcikk területéből kivonjuk a háromszög területét.

Példa 1 :
Egy 13 cm sugarú körben mekkora területű részt vág le egy 24 cm-es húr?

ABC△-ben:
12
sin β = ⇒ β = 67,38◦
13

Az ACD△ szárszöge 134,76◦ .


A körcikk területét egyenes arányossággal számoljuk ki:

360◦ Tkör = 132 π ≈ 530,93 cm2


134,76◦ Tcikk ≈ 198,57 cm2
13 · 13 · sin 134,76◦
TABC△ = = 60 cm2
2
Tszelet = Tcikk − T△ = 138,57 cm2 ♦

104
6. fejezet

Példa 2 :
Egy pillepalack alakja olyan forgáshenger, amelynek alapköre 8 cm átmérőjű. A palack fedő-
körén található a folyadék kiöntésére szolgáló szintén forgáshenger alakú nyı́lás. A két hengernek
közös a tengelye. A kiöntő nyı́lás alapkörének átmérője 2 cm. A palack magassága a kiöntő nyı́lás
nélkül 30 cm. A palack vı́zszintesen fekszik úgy, hogy annyi folyadék van benne, amennyi még
éppen nem folyik ki a nyitott kiöntő nyı́láson keresztül. Hány deciliter folyadék van a palackban?

Lerajzoljuk a palackot szemből.

ADF △-ben:
1
cos α = ⇒ α = 75,52◦
4

Az AED△ szárszöge 151,04◦ .


A körcikk területét megint egyenes arányossággal számoljuk ki.

360◦ Tkör = 42 π ≈ 50,27 cm2


151,04◦ Tcikk ≈ 21,09 cm2

105
6. fejezet

4 · 4 · sin 151,04◦
TABC△ = = 3,87 cm2
2
Tszelet = Tcikk − T△ = 17,22 cm2
V = 17,22 · 30 = 516,6 cm3

5,17 dl folyadék van a palackban.♦

6.4. Háromszögek hasonlósága


Ebben a részben a háromszögek hasonlóságának alkalmazásáról lesz szó, de azokban az ese-
tekben, amikor az a feladat, hogy fejezzük ki az egyik változót egy másik segı́tségével. Nagyon
fontos megértenünk, hogy nem az a feladat, hogy egy konkrét számot kapjunk. Az a feladat,
hogy egy összefüggést találjunk vagy gyártsunk a két változó között, és utána az egyiket kife-
jezzük a másik segı́tségével, azaz a másik függvényeként. Ha két változóval találkozunk, akkor
egy kapcsolat készı́tése a két változó között mindig egy jó gondolat.

Példa 1 :
Az 1. ábra szerinti padlástér egy 6 × 6 méteres négyzet alapú gúla, ahol a tető csúcsa a négyzet
középpontja felett 5 méter magasan van. Hasznos alapterületnek számı́t a tetőtérben az a
terület, amely fölött a (bel)magasság legalább 1,9 méter. Mennyi lenne a tetőtér beépı́tésekor
a hasznos alapterület? A tető cseréjekor a hasznos alapterület növelésének érdekében a ház ol-
dalfalait egy ún. koszorúval kı́vánják magası́tani. A ház teljes magassága – épı́tészeti előı́rások
miatt – nem növelhető, ezért a falak magası́tása csak úgy lehetséges, ha a tető sı́kjának me-
redekségét csökkentik (2. ábra). Jelölje x a koszorú magasságát és T a hasznos alapterületet.
Írjuk fel a T (x) függvény hozzárendelési szabályát.

Felrajzoljuk az eredeti ház metszetének a felét és felvesszük az adatokat.

106
6. fejezet

Legyen h a hasznos alapterület oldalának fele. A két piros háromszög hasonló, ezért:
3,1 1,9
=
h 3−h
3,1(3 − h) = 1,9h
h = 1,86

A hasznos alapterület tehát egy 2h oldalú négyzet területe, vagyis 13,8384 m2 .


A koszorú elkészı́tése után az első gondolat az, hogy x < 1,9. Ha a koszorú magassága legalább
1,9 méter, akkor a teljes 36 m2 -nyi alapterület hasznos terület.

Két változó szerepel a rajzban. A cél az, hogy egy összefüggést találjunk közöttük. Az előző
feladathoz hasonlı́t valamennyire a mostani rajz, ezért megint a két háromszög közötti kapcso-
latot ı́rjuk fel.

107
6. fejezet

A két sárga háromszög hasonló, ı́gy:


3,1 1,9 − x
=
y 3−y
3,1(3 − y) = y(1,9 − x)
9,3 = 5y − xy
9,3 = y(5 − x)
9,3
y =
5−x
 2
2 18,6
Az első kérdés alapján tudjuk, hogy a hasznos alapterület (2y) =
5−x
 2
 18,6

ha 0 ≤ x < 1,9
Összefoglalva: T (x) = 5−x ♦

36 ha 1,9 ≤ x

Példa 2 :
Az ABC háromszög oldalai AB = 42; BC = 40; CA = 26. Írjunk téglalapot a háromszögbe
úgy, hogy a téglalap egyik oldala illeszkedjen a háromszög AB oldalára, másik két csúcsa pe-
dig a háromszög CA, illetve BC oldalára essen. Tekintsük az ı́gy beı́rható téglalapok közül a
legnagyobb területűt. Mekkorák ennek a téglalapnak az oldalai?

42 + 40 + 26
s= = 54
2

A háromszög területe:
p
T = s(s − a)(s − b)(s − c)

T = 54 · 12 · 14 · 28
T = 504

Egyenlővé tesszük a másik területképlettel:


42m
504 =
2
m = 24

108
6. fejezet

Ismét az a teendő, hogy a téglalap x és y oldalai között egy összefüggést találjunk. Nagyon sok
hasonló háromszög van az ábrában, de a legtöbben nem szerepel x és y egyszerre. Az egyetlen
ilyen pár a CDE△ és a CAB△.
24 − y 24
=
x 42
42(24 − y) = 24x
4
y = 24 − x
7
Most már felı́rhatjuk a téglalap területét egy változóval:
 
4
T = xy = x 24 − x
7

Elemi úton, parabolaábrázolással számoljuk ki a terület szélsőértékét, mivel ez egy parabola, és
a parabola szélsőértéke a két zérushely számtani közepénél van a tengelyes szimmetria miatt.
A zérushelyeket számoljuk ki.
 
4
x 24 − x = 0
7
x1 = 0
x2 = 42

Innen tudjuk, hogy x = 21, vagyis a téglalap oldalai 21 és 12 hosszúak.♦

6.5. Testek összeillesztése


Amikor testeket összeillesztünk, figyelni kell, hogy a térfogatok összeadhatók, de a felszı́nek
nem. Az új felszı́nről kétféleképp is lehet gondolkozni. Lehet az összeillesztett testet borı́tó la-
pokat összeadni, vagy lehet az összetevők felszı́nképletéből a ,,láthatatlan” részeket lehúzni, és
összeadni a maradék felszı́ndarabokat.

Példa 1 :
Három tömör játékkockát az ábrának megfelelően rakunk össze. Mindegyik kocka éle 3 cm.
Mekkora a keletkező test felszı́ne és térfogata?

Vtest = 3a3 = 81 cm3

A felszı́nnél összeszámoljuk hány négyzet borı́tja a testet.

Atest = 14a2 = 126 cm2 ♦

109
6. fejezet

Példa 2 :
Egy henger alakú bot hossza 90 cm, alapkörének sugara 2 cm. Az egyik végére egy félgömböt
helyezünk, a másik végére pedig egy 8 cm hosszú kúpot. Mekkora a felszı́ne és a térfogata az
ı́gy készült sétapálcának?

A térfogatok:

4 · 23 · π
Vfélgömb = = 16,76 cm3
6
Vhenger = 22 · π · 90 = 1130,97 cm3
22 · π · 8
Vkúp = = 33,51 cm3
3
Vbot = 16,76 + 1130,97 + 33,51 = 1181,24 cm3

Pitagorasz-tétel a kúpban:

a2 = 2√2 + 82
a = 68

A felszı́nek:

Agömb = 4r2 π ⇒ Abúra = 2r2 π = 25,13 cm2


Ahenger = 2r2 π + 2rπM ⇒ Apalást = 2rπM = 1130,97 cm2
Akúp = r2 π + raπ ⇒ Akúppalást = raπ = 51,81 cm2

A felszı́nképletekben beazonosı́tjuk melyik területképlet melyik sı́kidomé, és azokat, amik az


összeillesztéskor eltűnnek, kihagyjuk a képletből. Ezek után a maradékok már valóban össze-
adhatók.

110
6. fejezet

Abot = 25,13 + 1130,97 + 51,81 = 1207,91 cm2

Rossz az a gondolkozás, ha azt hisszük, hogy összesen két darab kör esik ki az összeillesztéskor,
mert két darab kör látszik. Valójában három darab esik ki, mert a kúp tetején és a henger alján
is van egy-egy kör, valamint a henger tetején is van még egy.♦

Példa 3 :
Egy facölöp egyik végét csonkakúp alakúra, másik végét forgáskúp alakúra formálták. (Így egy
forgástestet kaptunk.) A középső, forgáshenger alakú rész hossza 60 cm és átmérője 12 cm. A
csonkakúp alakú rész magassága 4 cm, a csonkakúp fedőlapja pedig 8 cm átmérőjű. Az elkészült
cölöp teljes hossza 80 cm.
Hány egész m3 fára volt szükség 5000 darab cölöp gyártásához, ha a gyártáskor a felhasznált
alapanyag 18%-a hulladék?
Az elkészült cölöpök felületét vékony lakkréteggel vonják be. Hány egész m2 felületet kell be-
lakkozni, ha 5000 cölöpöt gyártottak?

A térfogatok:
Mπ 2 4π 2
Vcsonkakúp = (R + Rr + r2 ) = (6 + 6 · 4 + 42 ) = 318,34 cm3
3 3
Vhenger = r2 · π · M = 62 · π · 60 = 6785,84 cm3
r2 · π · M 62 · π · 16
Vkúp = = = 603,18 cm3
3 3
Vcölöp = 318,34 + 6785,84 + 603,18 = 7707,36 cm3

111
6. fejezet

5000 darab cölöpben:

5000 · 7707,36 = 38 536 800 cm3 = 38,53 m3 fa van.

A felhasznált alapanyag 18%-a hulladék, vagyis a 38,53 m3 a kiindulási anyag 82%-a.

38,53 : 0,82 = 46,98 m3

47 m3 fára volt szükség.

Pitagorasz-tétel a csonkakúpban:

b2 = 4√2 + 22
b = 20

Pitagorasz-tétel a kúpban:

a2 = 6√2 + 162
a = 292

A felszı́nek:

Acsonkakúp = R2 π + r2 π + bπ(R + r) ⇒ Ateteje = 42 π + 20π(6 + 4) = 190,76 cm2
Ahenger = 2r2 π + 2rπM ⇒ Apalást = 2 · 6 · π · 60 = 2261,94 cm2

Akúp = r2 π + raπ ⇒ Akúppalást = 6 · 292 · π = 322,1 cm2

A felszı́nképletekből megint kitöröljük a ,,láthatatlan” részeket.

Acölöp = 190,76 + 2261,94 + 322,1 = 2774,8 cm2

5000 cölöp felülete 5000 · 2774,8 = 13 874 000 cm2 = 1387,4 m2


1387 m2 felületet kell belakkozni.

Hasonlóan az előző feladathoz, most is úgy tűnik, mintha két darab kör esne ki, mert ennyi
látszik a rajzban. Valójában négy darab kör területét kell levonni az összfelszı́nből, mivel a
henger alján és tetején, a csonkakúp alján és a kúp tetején is van egy-egy kör.♦

112
7. fejezet

7. Trigonometria
7.1. Másodfokúra vezető trigonometrikus egyenletek
Ha csak egyfajta szögfüggvény található benne, akkor annak a helyére vezetjük be az új isme-
retlent. Ha viszont többfajta van benne, akkor azt a szögfüggvényt érdemes meghagyni, amelyik
első fokon szerepel az egyenletben, és a másodfokút addı́ciós tételekkel vagy Pitagorasz-tétellel
át kell ı́rni.

Példa 1 :
Oldjuk meg a valós számok halmazán: sin2 x − 3 sin x + 2 = 0

Egyfajta szögfüggvény van benne, ı́gy elvégezzük az y = sin x helyettesı́tést.

y 2 − 3y + 2 = 0
y1 = 1
y2 = 2

Visszahelyettesı́tve:

x = 90◦ + k · 360◦

sin x = 1 ⇒
(x = 90◦ a másik megoldás is, tehát felesleges felı́rni kétszer)
sin x = 2 nincs megoldás
π
x= + k2π, ahol k ∈ Z ♦
2
Példa 2 :
Oldjuk meg a valós számok halmazán: 2 cos2 x = 4 − 5 sin x

Most kétfajta szögfüggvényünk van. A szinuszt hagyjuk meg, mert az elsőfokú, a koszinuszt
ı́rjuk át a Pitagorasz-tétellel.

2 1 − sin2 x = 4 − 5 sin x


Most végezzük el az y = sin x helyettesı́tést.

2 (1 − y 2 ) = 4 − 5y
2y 2 − 5y + 2 = 0
y1 = 2
1
y2 =
2
Visszahelyettesı́tve:

x = 30◦ + k · 360◦

1
sin x = ⇒
2 x = 150◦ + l · 360◦
sin x = 2 nincs megoldás
π 5π
x= + k2π vagy + l2π, ahol k; l ∈ Z ♦
6 6

113
7. fejezet

Példa 3 :
Oldjuk meg a [−π; π] halmazon: 2 sin x − 2 sin2 x = cos2 x

Most is kétfajta szögfüggvényünk van. A szinuszt hagyjuk meg, mert az elsőfokú (és ennek
következtében a sin2 x-t is), a koszinuszt ı́rjuk át a Pitagorasz-tétellel.

2 sin x − 2 sin2 x = 1 − sin2 x

Most jön az y = sin x helyettesı́tés.

2y − 2y 2 = 1 − y 2
2
y − 2y + 1 = 0
y = 1

Visszahelyettesı́tve:

x = 90◦

sin x = 1 ⇒
(x = 90◦ a másik megoldás is, ezért felesleges felı́rni kétszer)
π
x= ♦
2
Példa 4 : √
Oldjuk meg a [0; 2π] halmazon: 3(tg x + ctg x) = 4

Most is kétfajta szögfüggvényünk van. (Arról, hogy a tangenst és a kotangenst mire érdemes
átı́rni, külön példák vannak a 7.8. fejezetben.)
√ 1
3(tg x + )=4
tg x

Jöhet is az y = tg x helyettesı́tés.
√ 1
3(y + ) = 4
y

√ 3
3y + = 4
y
√ 2 √
3y − 4y + 3 = √
0
y1 = 3
1
y2 = √
3
Visszahelyettesı́tve:
√ x = 60◦

tg x = 3⇒
x = 240◦ hamis megoldás
x = 30◦

1
tg x = √ ⇒
3 x = 210◦ hamis megoldás
π 5π
x= vagy ♦
6 6

114
7. fejezet

7.2. Addı́ciós tételes feladatok


Trigonometrikus egyenleteknél az addı́ciós tételek felismerése (mindkét irányba alkalmazva) na-
gyon megkönnyı́theti a feladatot.

Példa 1 :
Oldjuk meg a valós számok halmazán: cos 2x = 2 − 5 cos x

Kibontjuk az addı́ciós tételt.

cos2 x − sin2 x = 2 − 5 cos x

Alkalmazzuk a Pitagorasz-tételt.

cos2 x − 1 − cos2 x = 2 − 5 cos x




Jöhet is az y = cos x helyettesı́tés.

y 2 − (1 − y 2 ) = 2 − 5y
2y 2 + 5y − 3 = 0
1
y1 =
2
y2 = −3

Visszahelyettesı́tve:

x = 60◦ + k · 360◦

1
cos x = ⇒
2 x = 300◦ + l · 360◦
cos x = −3 nincs megoldás
π 5π
x= + k2π, vagy + l2π, ahol k; l ∈ Z ♦
3 3
Példa 2 :
1
Oldjuk meg a valós számok halmazán: 4 cos x =
sin x
Kikötés:
sin x ̸= 0 ⇒ x ̸= kπ
A nevezővel beszorzás megszokott lépés.

4 sin x cos x = 1

Azt kell meglátnunk, hogy a bal oldalon egy addı́ciós tétel kétszerese szerepel.

2 sin 2x = 1
1
sin 2x =
2
2x = 30 + k · 360 ⇒ x = 15◦ + k · 180◦
◦ ◦

2x = 150◦ + l · 360◦ ⇒ x = 75◦ + l · 180◦


π 5π
x= + kπ vagy + lπ, ahol k; l ∈ Z. A kikötés egyik végeredményt sem zárja ki.♦
12 12
115
7. fejezet

Példa 3 :
Oldjuk meg a valós számok halmazán: sin x + cos x = 1

Négyzetre emeljük az egyenletet. A bal oldal közepén egy addı́ciós tétel jön létre.

sin2 x + 2 sin x cos x + cos2 x = 1


1 + 2 sin x cos x = 1
sin 2x = 0
2x = 0◦ + k · 360◦ ⇒ x = 0◦ + k · 180◦
2x = 180◦ + l · 360◦ ⇒ x = 90◦ + l · 180◦
π
x = 0 + kπ vagy + lπ, ahol k; l ∈ Z
2
Mivel négyzetre emeltünk, és az nem ekvivalens átalakı́tás, az ellenőrzés elkerülhetetlen. Csak
π
az x = 0 + k2π illetve + k2π alakú megoldások jók.♦
2
Példa 4 :
2
Oldjuk meg a valós számok halmazán: 4cos 2x + 4cos x = 3

Kibontjuk az addı́ciós tételt.


2 x−sin2 x 2
4cos + 4cos x
=3

Alkalmazzuk a Pitagorasz-tételt, hogy csak egyfajta szögfüggvény legyen.


2 x−(1−cos2 x) 2
4cos + 4cos x
= 3
2 cos2 x−1 cos2 x
4 +4 = 3
2 cos2 x
4 2
+ 4cos x
= 3
4
Itt észrevesszük, hogy másodfokúra vezető exponenciális egyenletünk van. Elvégezzük az
2
y = 4cos x helyettesı́tést.

y2
+y = 3
4
y 2 + 4y − 12 = 0
y1 = 2
y2 = −6

Visszahelyettesı́tve:
2
4cos x = 2
2 1
4cos x = 4 2
mivel az exp...
1
cos2 x =
2
r
x = 45◦ + k · 360◦

1
cos x = ⇒
2 x = 315◦ + l · 360◦
r
x = 135◦ + m · 360◦

1
cos x = − ⇒
2 x = 225◦ + n · 360◦

116
7. fejezet

2
4cos x
= −6 nincs megoldás

π 3π 5π 7π
x= + k2π ; x = + l2π ; x = + m2π vagy x = + n2π, ahol k; l; m; n ∈ Z
4 4 4 4
π π
Ebben az esetben a négy megoldás egybeolvasztható, ı́gy mondhatjuk, hogy x = + k ♦
4 2
Példa 5 :
Egy hegyesszögű háromszög α és β szögeire sin (α + β) = cos (α − β). Bizonyı́tsuk be, hogy a
háromszög egyik oldala a magasságpontból 135◦ -os szögben látszik.

Első olvasásra nem nagyon látunk kapcsolatot a trigonometrikus összefüggés és a látószög
között. Indı́tásként kibontjuk az addı́ciós tételeket.

sin α cos β + cos α sin β = cos α cos β + sin α sin β

Nullára rendezzük az egyenletet, mert hasonló dolgok szerepelnek benne, és esetleg csopor-
tosı́tásos kiemeléssel szorzattá lehet alakı́tani. (3.2. fejezet)

sin α cos β + cos α sin β − cos α cos β − sin α sin β = 0


sin α (cos β − sin β) − cos α (cos β − sin β) = 0
(cos β − sin β) (sin α − cos α) = 0
↙↘
cos β = sin β sin α = cos α

Azt kaptuk, hogy vagy az alfának, vagy a bétának a szinusza és koszinusza megegyezik. Az
egyetlen hegyesszög, amire ez igaz, a 45◦ , vagyis az α és β közül valamelyik biztosan 45◦ .

117
7. fejezet

Megrajzoljuk az ábra szerint a háromszög magasságvonalait.


Ekkor AT M S négyszögben T M S∢ = 135◦ . Ebből következik, hogy CM B∢ = 135◦ , mert
csúcsszögek. Kész a bizonyı́tás, mert a BC oldal a magasságpontból 135◦ -os szögben látszik.♦

Példa 6 :
Bizonyı́tsuk be, hogy sin2 x ≥ sin 2x − cos2 x

Kibontjuk az addı́ciós tételt.

sin2 x ≥ 2 sin x cos x − cos2 x

A nullára rendezés egy általában előhúzható módszer.

sin2 x − 2 sin x cos x + cos2 x ≥ 0

Észrevesszük a nevezetes azonosságot.

(sin x − cos x) ≥ 0

Erről tudjuk, hogy igaz.♦

Példa 7 :
Oldjuk meg a [0; 2π] halmazon: sin 2x cos x + cos 2x sin x = 0

Kibontjuk az addı́ciós tételeket.

2 sin x cos2 x + cos2 x − sin2 x sin x



= 0
3 sin x cos2 x − sin3 x = 0
sin x 3 cos2 x − sin2 x = 0
↙↘
sin x = 0 3 cos2 x − sin2 x = 0
x = 0; π; 2π 3 cos2 x = sin2 x
3 = tg2 x
√ ↙↘ √
tg x = 3 tg x = − 3
π 4π 2π 5π
x = ; x = ;
3 3 3 3
Ha cos x = 0, akkor nem lehet osztani vele a 3 cos2 x = sin2 x sorban, ı́gy meg kell vizsgálni
π 3π
külön azt az esetet, amikor ez fennáll. Ez x = , esetekben fordulhat elő, de egyik sem
2 2
megoldása az egyenletnek.♦

Példa 8 :
Igazoljuk, hogy ha egy háromszög szögeire érvényes, hogy sin α : sin β = cos (α + γ) : cos (β + γ),
akkor a háromszög egyenlő szárú vagy derékszögű.

Mivel α + γ és β + γ szerepel a szövegben, érdemes átı́rni ezeket (180◦ − β) és (180◦ − α)-ra
egyrészt azért, mert ezek tovább alakı́thatók, másrészt pedig ı́gy csak két szög lesz a három
helyett.

118
7. fejezet

sin α cos (α + γ)
=
sin β cos (β + γ)
sin α cos (180◦ − β)
=
sin β cos (180◦ − α)
sin α cos 180◦ cos β + sin 180◦ sin β
=
sin β cos 180◦ cos α + sin 180◦ sin α
sin α − cos β
=
sin β − cos α
sin α cos β
=
sin β cos α
sin α cos α = sin β cos β

Most nem érdemes nullára rendezni, mert nincsenek közös tényezők, amik kiemelhetők lennének.
Helyette észrevesszük, hogy mindkét oldal egy addı́ciós tétel fele.

2 sin α cos α = 2 sin β cos β


sin 2α = sin 2β
↙↘
2α = 2β 2α = 180◦ − 2β
α = β α + β = 90◦

Az első esetben van két egyenlő szög, vagyis a háromszög egyenlő szárú, a második esetben a
háromszög harmadik szöge 90◦ , tehát derékszögű.♦

Példa 9 :
cos2 x − sin2 x
Oldjuk meg a [−π; π] halmazon: sin x =
2 cos 2x
Kikötés:
π 3π
cos 2x ̸= 0 ⇒ x ̸= ± , ±
4 4
cos 2x
sin x =
2 cos 2x
1
sin x =
2
↙↘
x = 30◦ x = 150◦
π 5π
A kikötés a és végeredmények közül egyiket sem zárja ki.♦
6 6

7.3. A cos x − sin x = 0 és cos x + sin x = 0 tı́pusú egyenletek


Ezeknek az egyenlettı́pusoknak van egy gyors megoldása.

Ha cos x = sin x, a megoldás mindig a 45◦ ; 225◦ , ha pedig cos x = − sin x, a megoldás mindig a
135◦ ; 315◦ . Ezek azok a szögek, ahol a két szögfüggvény értéke és előjele is stimmel.

119
7. fejezet

Összegezve:
π
cos x − sin x = 0 ⇒ x = + kπ
4

cos x + sin x = 0 ⇒ x = + kπ
4
ahol k ∈ Z

7.4. A sin x = sin y, a cos x = cos y és a tg x = tg y tı́pusú tı́pusú egyen-


letek
Ezeknek az egyenleteknek a megoldása is fontos, hogy gyorsan és hibátlanul menjen.

7.4.1. A sin x = sin y tı́pusú egyenletek


Két lehetőség van. x = y vagy x = 180◦ − y, a periódus mindig 360◦ .

Példa :
Oldjuk meg a valós számok halmazán: sin (x + 60◦ ) = sin (2x − 60◦ )

sin (x + 60◦ ) = sin (2x − 60◦ )


↙↘
x + 60◦ = (2x − 60◦ ) + k · 360◦ x + 60◦ = 180◦ − (2x − 60◦ ) + l · 360◦
120◦ = x + k · 360◦ 3x = 180◦ + l · 360◦
x = 120◦ − k · 360◦ x = 60◦ + l · 120◦

Mivel k; l ∈ Z, ezért mindegy, hogy az első megoldásnál k-t, vagy −k-t ı́runk átrendezés után,
de az algebra kedvéért ı́rjunk −k-t.
2π π 2π
A végeredmény x = − k2π illetve x = + l ♦
3 3 3

120
7. fejezet

7.4.2. A cos x = cos y tı́pusú egyenletek


Két lehetőség van. x = y vagy x = 360◦ − y, a periódus mindig 360◦ .

Példa :
Oldjuk meg a valós számok halmazán: cos 5x = cos 4x

cos 5x = cos 4x
↙↘
5x = 4x + k · 360◦ 5x = 360◦ − 4x + l · 360◦
x = 0◦ + k · 360◦ x = 40◦ + l · 40◦
2π 2π
A végeredmény x = 0 + k2π illetve x = + l , ahol k; l ∈ Z ♦
9 9

7.4.3. A tg x = tg y tı́pusú egyenletek


Egy lehetőség van. x = y, a periódus mindig 180◦ .

Példa :
Oldjuk meg a valós számok halmazán: tg (2x − 30◦ ) = tg x

2x − 30◦ = x + k · 180◦
x = 30◦ + k · 180◦ ♦

π
A végeredmény x = + kπ, ahol k ∈ Z ♦
6

7.5. A sin2 x + cos2 x = 1 használata


Trigonometrikus egyenletben az 1-et néha érdemes visszaı́rni sin2 x + cos2 x-re.

Példa 1 :
Oldjuk meg a valós számok halmazán: cos 2x + sin 2x = −1

Az addı́ciós tételekkel átalakı́tjuk az egyenletet. Mivel ezek után sin2 x és cos2 x lesz az egyen-
letben, ez sugallja, hogy érdemes átı́rni az 1-est.
cos2 x − sin2 x + 2 sin x cos x − sin2 x + cos2 x

=
cos2 x − sin2 x + 2 sin x cos x = − sin2 x − cos2 x
2 cos2 x + 2 sin x cos x = 0
2 cos x (cos x + sin x) = 0
↙↘
cos x = 0 cos x + sin x = 0
↙↘ 3π
◦ ◦ ◦ ◦
x = + mπ
x = 90 + k · 360 x = 270 + l · 360 4
π 3π
x = + k2π x = + l2π
2 2
k; l; m ∈ Z ♦

121
7. fejezet

Példa 2 :
Oldjuk meg a valós számok halmazán: 1 + tg2 x = cos2 x

sin2 x + cos2 x + tg2 x = cos2 x


sin2 x + tg2 x = 0

Mivel egy tetszőleges valós szám négyzete mindig nagyobb, vagy egyenlő, mint nulla, ezért ez a
kifejezés csak akkor lehet nulla, ha sin x = 0 és tg x = 0 egyszerre. A végeredmény x = 0 + kπ,
k∈Z♦

Példa 3 :
1 + cos 2x 1 − cos 2x
Bizonyı́tsuk be, hogy cos2 x = és sin2 x =
2 2

1 + cos 2x sin2 x + cos2 x + cos2 x − sin2 x 2 cos2 x


= = = cos2 x
2 2 2
1 − cos 2x sin2 x + cos2 x − cos2 x + sin2 x 2 sin2 x
= = = sin2 x♦
2 2 2

7.6. A koszinusz-tétel
A 6.2. fejezetben tárgyalt Pitagorasz-tételhez hasonlóan ez a tétel is alkalmazható abban az
esetben is, ha egy oldalt sem tudunk a háromszögből, viszont minden oldal ugyanazzal a betűvel
van kifejezve.

Példa 1 :
A derékszögű koordináta-rendszerben az ABC háromszög csúcsai A (2; 1) , B (7; −4) , C (11; p).
Határozzuk meg a p paraméter pontos értékét, ha a háromszög B csúcsánál levő szöge 60◦ -os.

√ √
AB = p52 + 52 = 50 p
AC = p92 + (p − 1)2 = pp2 − 2p + 82
AB = 42 + (p + 4)2 = p2 + 8p + 32
AC 2 = AB 2 + BC 2 − 2 · AB · BC · cos 60◦
√ p 1
p2 − 2p + 82 = 50 + p2 + 8p + 32 − 2 50 · p2 + 8p + 32 ·
√ p 2
10p = 50 · p2 + 8p + 32 (p > 0 következik ebből a sorból)
100p2 = 50p2 + 400p + 1600
p2 − 8p − 32 = 0 √
p1 = 4 + 4√3
p2 = 4 − 4 3 hamis megoldás a feltétel miatt

A valós paraméter értéke 4 + 4 3.♦

Példa 2 :
Egy r sugarú körbe meghúztuk AB és BC húrokat. AB egyenlő a körbe ı́rható szabályos hatszög
oldalával, BC pedig egyenlő a körbe ı́rható négyzet oldalával. Milyen hosszú az AC szakasz?
A választ r függvényében adjuk meg és pontos értékkel számoljunk.

122
7. fejezet


A hatszög oldala r, a négyzet oldala r 2.

√ 2
AC 2 = r2 + r 2 − 2 · r · r · cos 105◦
√ √
√ 2 − 6
AC 2 = 3r2 − 2 2r2 ·
4
√ √ !
√ 2 − 6
AC 2 = r2 3 − 2 2 ·
4
√ !
2 2 4 − 2 12
AC = r 3 −
4
√ !
4 − 4 3
AC 2 = r2 3 −
4
√ 
AC 2 = r2 3 − 1 + 3
√ 
AC 2 = r2 2 + 3
p √
AC = r 2 + 3♦

Példa 3 :
Egy háromszög egyik szöge 60◦ . E szög felezője a szemközti oldalt 6 cm-es és 8 cm-es részekre
osztja. Mekkorák a háromszög oldalai?

123
7. fejezet

A szögfelező-tétel miatt tudjuk, hogy a b és c oldalak aránya megegyezik a szögfelező által
kettéosztott oldal darabjainak arányával. Így felı́rható, hogy b = 6x és c = 8x.

142 = (6x)2 + (8x)2 − 2 · 6x · 8x · 60◦


196 = 52x2 √
7 7 13
x = √ =
13 13
√ √
42 13 56 13
A b oldal hossza cm, a c oldal hossza pedig cm.♦
13 13

7.7. A dupla koszinusz-tétel


Ha egy általános háromszögben tudunk két oldalt/egy egyenlő szárú háromszögben az alapot, és
behúzunk egy súlyvonalat/tetszőleges egyenest, amiről tudjuk, hogy mekkora darabokra vágja
a szemközti oldalt, akkor az oldal kiszámolásához érdemes megpróbálni a két háromszögben
felı́rt koszinusz-tételekből álló egyenletrendszert.

Példa 1 :
Az ABC háromszögben AB = 2; AC = 1, a BC oldal hossza pedig megegyezik az A csúcsból
induló súlyvonal hosszával. Mekkora a BC oldal hossza? A hossz pontos értékét adjuk meg.

Túl kevés adat áll rendelkezésünkre akár egy Pitagorasz-tételhez. Ezekkel a kiindulási ada-
tokkal jusson eszünkbe a két koszinusz-tételből álló egyenletrendszer. Persze ennek csak akkor
van értelme, ha két ismeretlenünk van. Az egyik biztosan az x lesz. A másik pedig az egyik
szög. A súlyvonal behúzása után három háromszögünk lett, ı́gy három lehetőségünk van a szög
megválasztására. Nagyon fontos, hogy ilyenkor azt a két háromszöget kell választani, amiben
benne van a szög, és arra az oldalra felı́rni a koszinusz-tételt, amivel szemben van a választott
szög.

124
7. fejezet

1. lehetőség: B csúcsnál lévő β szög a másik ismeretlen.

ABF △-ben és ABC△-ben ı́rjuk fel a koszinusz-tételeket.



(2x)2 = 22 + x2 − 2 · 2 · x · cos β
12 = 22 + (2x)2 − 2 · 2 · 2x · cos β
 2
4x = 4 + x2 − 4x cos β
1 = 4 + 4x2 − 8x cos β

Célunk a β szög kiejtése az egyenlő együtthatók módszerével, ezért szorozzuk kettővel az első
egyenletet:
 2
8x = 8 + 2x2 − 8x cos β
1 = 4 + 4x2 − 8x cos β

Most kivonjuk egymásból őket:

8x2 − 1 = 4 − 2x2
10x2 = 5√
.
2
x =
2

Vagyis a BC oldal hossza 2.

125
7. fejezet

2. lehetőség: C csúcsnál lévő γ szög a másik ismeretlen.

ACF △-ben és ABC△-ben ı́rjuk fel a koszinusz-tételeket.



(2x)2 = 12 + x2 − 2 · 1 · x · cos γ
22 = 12 + (2x)2 − 2 · 2 · x · cos γ
 2
4x = 1 + x2 − 2x cos γ
4 = 1 + 4x2 − 4x cos γ

Megint a szög kiejtése a cél, ezért szorozzuk kettővel az első egyenletet:


 2
8x = 2 + 2x2 − 4x cos γ
4 = 1 + 4x2 − 4x cos γ

Megint kivonjuk őket egymásból:

8x2 − 4 = 1 − 2x2
10x2 = 5√
2
x =
2

Vagyis a BC oldal hossza 2.

126
7. fejezet

3. lehetőség: F csúcsnál lévő φ szög a másik ismeretlen.

ABF △-ben és ACF △-ben ı́rjuk fel a koszinusz-tételeket. Ebben az esetben a kiegészı́tőszög is
szerepelni fog.
 2
1 = x2 + (2x)2 − 2 · x · 2x · cos φ
22 = x2 + (2x)2 − 2 · x · 2x · cos (180◦ − φ)

1 = 5x2 − 4x2 cos φ
4 = 5x2 − 4x2 cos (180◦ − φ)

1 = 5x2 − 4x2 cos φ
4 = 5x2 + 4x2 cos φ
Most tudni kell azt, hogy cos (180◦ − φ) = − cos φ
Összeadjuk a két egyenletet.

10x2 = 5√
2
x =
2

Vagyis a BC oldal hossza 2.

A harmadik lehetőség egy picivel összetettebb az első kettőnél, úgyhogy én ezt kevésbé javaso-
lom, de látható, hogy bármelyik lehetőséggel ugyanazt az eredményt kapjuk meg.♦

Példa 2 :
Egy egyenlő szárú háromszögnek az alapja 10 cm. Az alap egyik csúcsát összekötjük a szemközti
szár egyik belső pontjával úgy, hogy ez a pont a szárat 3 : 5 arányban osztja. Milyen hosszúak
a háromszög szárai, ha ez a szakasz 12 cm?

127
7. fejezet

Elkészı́tjük a rajzot. Mivel a háromszög egyenlő szárú, ezért tudjuk, hogy az AB szár 8x hosszú.

A B csúcsnál lévő β szöget választjuk másik ismeretlennek, ı́gy az ABF △-ben és az ABC△-
ben ı́rjuk fel a koszinusz-tételeket.
 2
12 = (3x)2 + (8x)2 − 2 · 3x · 8x · cos β
102 = (8x)2 + (8x)2 − 2 · 8x · 8x · cos β

144 = 73x2 − 48x2 cos β
100 = 128x2 − 128x2 cos β

A szöget akarjuk kiejteni, ezért megszorozzuk mindkét egyenletet a megfelelő számokkal:



1152 = 584x2 − 384x2 cos β
300 = 384x2 − 384x2 cos β

Most jön az egyenletek kivonása:

200x2
852 = √
x = 4,26 ≈ 2,06

A szárak 16,51 cm hosszúak.♦

7.8. A tg és a ctg


sin x cos x
Ezeket a szögfüggvényeket néha ; -re kell átı́rni, néha azt kell kihasználni, hogy
cos x sin x
1
egymás reciprokai, tehát új ismeretlen bevezetése után y; lesz belőlük.
y

128
7. fejezet

Példa 1 :
Oldjuk meg a valós számok halmazán: tg x + ctg x = 2

Kikötések:
π
x ̸= + kπ
2
x ̸= 0 + kπ

Mivel a kotangens a tangensnek a reciproka, ı́gy most ezt az átalakı́tást végezzük el, mert akkor
csak tangens lesz benne.
1
tg x + =2
tg x

Bevezetjük az új ismeretlent. Legyen y = tg x.


1
y+ =2
y

Tudjuk, hogy az 1 az egyetlen szám, amit ha összeadunk a saját reciprokával, 2-t kapunk, ezért
y = 1.
Visszahelyettesı́tünk:

tg x = 1
π
x = + kπ
4
k ∈ Z. ♦

Példa 2 :
Oldjuk meg a valós számok halmazán: sin x = tg x

Kikötés:
π
x ̸= + kπ
2
sin x
Most a tangens mellett szinusz szerepel, tehát -re ı́rjuk át.
cos x
sin x
sin x =
cos x
sin x cos x = sin x
sin x cos x − sin x = 0
sin x (cos x − 1) = 0
↙↘
sin x = 0 cos x = 1
x = 0 + l2π x = 0 + n2π
x = π + m2π

A bal oldali ág megoldásai tartalmazzák a jobb oldali ág megoldásait is, ı́gy a végeredmény
x = 0 + l2π illetve x = π + m2π, ahol k; l; m; n ∈ Z. ♦

129
7. fejezet

Példa 3 :
Oldjuk meg a valós számok halmazán: (tg x + ctg x) sin 2x = 2

Kikötések:
π
x ̸= + kπ
2
x ̸= 0 + lπ

Az addı́ciós tételt már rutinosan bontjuk ki, és ı́gy látjuk, hogy szerepel benne szinusz és ko-
sin x cos x
szinusz is. Vagyis most -re, illetve -re ı́rjuk át őket.
cos x sin x
 
sin x cos x
+ 2 sin x cos x = 2
cos x sin x
2 sin2 x + 2 cos2 x = 2
2 = 2
n π o
Azonosságot kaptunk. A végeredmény x ∈ R \ 0 + lπ; + kπ , ahol k; l ∈ Z.♦
2
Példa 4 :
sin 2x
Oldjuk meg a valós számok halmazán: cos x(sin x + tg x) − =1
2

Kikötés:
π
x ̸= + kπ
2
sin x
Az addı́ciós tételt kibontjuk. Most a tangenst koszinusszal szorozzuk, ı́gy -re ı́rjuk át.
cos x
 
sin x 2 sin x cos x
cos x sin x + − = 1
cos x 2
sin x cos x + sin x − sin x cos x = 1
sin x = 1
↙↘
x = 90◦ + k · 360◦ a másik megoldás is 90◦
π
A kikötés miatt az x = + k2π hamis megoldás. ♦
2

130
8. fejezet

8. Koordináta-geometria
Sok diákot összezavar, hogy egy feladatra több megoldás is létezik, és ez a témakör olyan, hogy
többféle megoldás létezik szinte minden feladatra. Így alpontokba szedve igyekeztem a legegy-
szerűbbet megmutatni mindenhol és ezzel közelebb hozni a témakört.

8.1. A kör zárt egyenlete


A teljes négyzetté alakı́tást alkalmazzuk, hogy visszaalakı́tsuk a kör egyenletét polinom alakból
zárt alakba. Zárt alakban olvasható le ugyanis a kör középpontja és a sugara.

Példa 1 :
Mi a k : x2 + y 2 − 8x − 4y − 16 = 0 egyenletű kör középpontja és sugara?

(x − 4)2 − 16 + (y − 2)2 − 4 − 16 = 0
(x − 4)2 + (y − 2)2 = 36

A kör középpontja C (4; 2), sugara 6.♦

Példa 2 :
Egy háromszög két csúcsa A (−2; 2) ; B (6; 2). Határozzuk meg a harmadik csúcs koordinátáit,
ha a háromszög területe 8, és a háromszög köré ı́rható kör egyenlete x2 + y 2 − 4x − 10y + 4 = 0.

(x − 2)2 − 4 + (y − 5)2 − 25 + 4 = 0
(x − 2)2 + (y − 5)2 = 25

A kör középpontja K (2; 5), sugara 5.

131
8. fejezet

A és B pontok távolsága 8. Mivel a terület is 8, ebből az következik, hogy a háromszög ma-
gassága 2.

=m|y − 2|
=2|y − 2|
↙↘
y=4 y=0

Ha y = 4:

(x − 2)2 + (−1)2 = 25
x2 − 4x − 20 = 0 √
x1 = 2 + 2√6
x2 = 2 − 2 6
√  √ 
A harmadik csúcs koordinátái C 2 + 2 6; 4 , vagy D 2 − 2 6; 4 .
Ha y = 0:

(x − 2)2 + (−5)2 = 25
(x − 2)2 = 0
x = 2

E (2; 0) a harmadik csúcs.♦

Példa 3 :
Az x2 + y 2 − 2x − 4y + a = 0 és x2 + y 2 − 14x − 4y + b = 0 egyenletű körök kı́vülről érintik
egymást és a területeik összege 26π. Határozzuk meg a és b paraméterek értékét.

(x − 1)2 − 1 + (y − 2)2 − 4 + a = 0
(x − 1)2 + (y − 2)2 = 5 − a

K1 középpontja (1; 2), sugara 5 − a.
(x − 7)2 − 49 + (y − 2)2 − 4 + b = 0
(x − 7)2 + (y − 2)2 = 53 − b

K2 középpontja (7; 2), sugara 53 − b.

132
8. fejezet

Két ismeretlenünk van, de két összefüggésünk. Egyrészt tudjuk a területek összegét, másrészt
a rajz alapján tudjuk a körök középpontjainak távolságát is.

T1 + T2 = 26π
(5 − a) π + (53 − b) π = 26π
5 − a + 53 − b = 26
32 = a+b

Kifejezve az egyenletből: b = 32 − a
Mivel a két sugár összege 6, ide behelyettesı́tjük az előző összefüggést:
√ √
5 − a + 53 − b = 6
√ p
5 − a + 53 − (32 − a) = 6
√ √
5 − a + a + 21 = 6
√ √
5 − a + 2 5 − a a + 21 + a + 21 = 36
p
(5 − a)(a + 21) = 5
(5 − a) (a + 21) = 25
a2 + 16a − 80 = 0
a1 = 4
a2 = −20

Ha a = 4, akkor b = 28. Ha a = −20, akkor b = 52. Mindkét megoldás jó. ♦

8.2. A távolságképlet
Példa 1 :
Határozzuk meg az A (−2; 2) ; B (6; 7) pontok távolságát.

q √ √
AB = (−2 − 6)2 + (2 − 7)2 = 64 + 25 = 89♦

Példa 2 :
Egy háromszög csúcspontjának koordinátái A (2x ; 4x ) ; B (4x ; 2x ) ; C (4x ; 4x ). Határozzuk meg
a háromszög beı́rható körének sugarát, ha a háromszög területe 72.

q √ x
AB = (4x − 2x )2 + (4x − 2x )2 = 2 4 − 2x
q
AC = (4x − 2x )2 + 02 = 4x − 2x
q
BC = 02 + (4x − 2x )2 = 4x − 2x

Két oldal ugyanolyan hosszú, a harmadik pedig ennek 2-szerese. Ebből az következik, hogy a
háromszög egyenlő szárú és derékszögű.

133
8. fejezet

T△ = 72
x x 2
(4 − 2 )
= 72
2
(4 − 2x )2
x
= 144
↙↘
4x − 2x = 12 4x − 2x = −12 nincs megoldás

Ez azt jelenti, hogy a háromszög oldalai 12; 12; 12 2 hosszúak.
Jelenleg felesleges kiszámolni mennyi x értéke, mert a sugár meghatározásához elég tudnunk a
kerületet.

T△ = r · s √
(4x − 2x ) + (4x − 2x ) + 2 · (4x − 2x )
72 = r ·
√  x 2
x
144 = r · 2 + 2√ (4  −2 )
144 = 12r · 2 + 2
12
r = √ ♦
2+ 2

8.3. Egy belső szög kiszámolása


Ha egy háromszög vagy sokszög belső szögének értéke kell, több lehetséges megoldási módszer
közül lehet választani.

Példa :
Egy háromszög csúcsainak koordinátái A (1; 5) ; B (−4; 2) ; C (4; −2). Mekkora az α szög?

1. lehetőség:
Felı́rjuk a három oldalra a távolságképleteket és utána koszinusz-tétellel kiszámoljuk a szöget.

134
8. fejezet

q √
AB = (1 + 4)2 + (5 − 2)2 = 34
q √
AC = (1 − 4)2 + (5 + 2)2 = 58
q √
BC = (−4 − 4)2 + (2 + 2)2 = 80
√ √
80 = 34 +√ 58 − 2√· 34 · 58 · cos α
−12 = −2 · 34 · 58 · cos α
0,1351 = cos α
α ≈ 82,24◦

2. lehetőség:
Kiszámoljuk az adott csúcsból kiinduló két vektort, kiszámoljuk a skaláris szorzatukat a koor-
dináták, majd a hosszak segı́tségével, és ezeket egyenlővé tesszük.

A két vektor:
−→ −→
AB (−5; −3) ; AC (3; −7)

A skaláris szorzatuk:
−→ −→
AB · AC = −15 + 21 = 6

A két vektor hossza:


−→ √ √
AB = 25 + 9 = 34
−→ √ √
AC = 9 + 49 = 58
Egymással egyenlővé tesszük:
√ √
6 = 34 · 58 · cos α
0,1351 = cos α
α ≈ 82,24◦

135
8. fejezet

3. lehetőség:
A sı́kidomot kiegészı́tjük arra a legszűkebb téglalapra, aminek az oldalai párhuzamosak a ten-
gelyekkel, és tartalmazzák a sı́kidomot. A szı́nes háromszögek oldalai könnyen leolvashatók.

EBA△-ben:
3
tg φ =
5
φ = 30,96◦

CDA△-ben:
7
tg ε =
3
ε = 66,8◦

A rajz alapján látszik, hogy:

α = 180◦ − φ − ε
α ≈ 82,24◦

Ha kiszámolni kell a szöget egy feladatban, akkor a harmadik lehetőséget javasolom, mert
nem igényel a derékszögű háromszög szögfüggvényeinek ismeretén kı́vül mást (,,koordináta-
geometria mentes”). Ráadásul bármilyen sokszögben alkalmazható. De ha bizonyı́tani kell, ak-
kor inkább a skaláris szorzatos megoldást javasolom.♦

8.4. Vektorok párhuzamosságának bizonyı́tása


Ha két vektor párhuzamos, akkor biztosan egymás skalár szorosai.

136
8. fejezet

Példa :
Bizonyı́tsuk be, hogy az A (−2; −3) ; B (5; 4,5) ; C (3; −0,5) ; D (0; 2) pontok egy paralelog-
ramma csúcspontjai.

A feladat az, hogy bebizonyı́tsuk a szemközti oldalak párhuzamosak.

−→ −−→ −−→ −−→


AC (5; 2,5) ; DB (5; 2,5) és DA (−2; −5) ; BC (−2; −5)

A szemközti vektorok egyenlők, ı́gy ABCD paralelogramma.♦

8.5. Vektorok merőlegességének bizonyı́tása


Két vektor merőleges akkor és csak akkor, ha skaláris szorzatuk 0.

Példa 1 :
Bizonyı́tsuk be, hogy ha egy paralelogramma rombusz, akkor az átlói merőlegesek egymásra.

137
8. fejezet

Vizsgáljuk (a + b) (a − b) szorzatot.

(a + b)(a − b) = a2 − b2 = |a|2 − |b|2

Mivel a paralelogramma rombusz is egyszerre, ezért |a| = |b|.


Emiatt:

|a|2 − |b|2 = |a|2 − |a|2 = 0

A két vektor ı́gy merőleges.♦

Példa 2 :  
−→ −−→ b
Az A pont helyvektora OA (lg a; lg b), a B pont helyvektora OB lg ab; lg , ahol a és b olyan
a
valós számokat jelölnek, melyekre 0 < a < 1, illetve 1 < b teljesül. Bizonyı́tsuk be, hogy az
−→ −−→ −→
OA − OB vektor merőleges az OA vektorra.

−−→
OB (lg a + lg b; lg b − lg a)
−→ −−→
OA − OB (− lg b; lg a)

Vesszük a vektorok skaláris szorzatát, mert ha nulla, akkor merőlegesek:


−→ −→ −−→
OA OA − OB = − lg a lg b + lg a lg b = 0♦

8.6. Egyenesek párhuzamosságának bizonyı́tása


Két egyenes párhuzamos, ha:

8.6.1. Normálvektoruk egymás skalár szorosa


Példa 1 :
Határozzuk meg p értékét úgy, hogy az e : 3x−y = 7 és f : px = 11−3y egyenesek párhuzamosak
legyenek.

ne (3; −1) ; nf (p; 3)

Leolvasható, hogy a második koordináta −3-szorosára változott, akkor az elsőnek is ı́gy kell
változnia, ı́gy p = −9.♦

Példa 2 :
A k mely értékére lesz egymással párhuzamos az e : 2x + ky = 15 és f : kx + 8y = 7 egyenes?

ne (2; k) ; nf (k; 8)

138
8. fejezet

Most nem olvasható le azonnal a változás mértéke, ı́gy aránypárral kell dolgoznunk.
2 k
=
k 8
k = ±4

Egyik k esetén sem lesz a két egyenes azonos, ı́gy ,,csak” párhuzamosak.♦

8.6.2. Meredekségük egyenlő


Példa 1 :
Az e egyenes egyenlete y = 2x + 3. Határozzuk meg f egyenes y = ax + 5 egyenletében a értékét
úgy, hogy a két egyenes párhuzamos legyen.

me = 2; mf = a vagyis a = 2♦

Példa 2 :
Bizonyı́tsuk be, hogy az A (−7; 6) ; P (−1; 3) ; B (3; 1) pontok egy egyenesre illeszkednek.

3−6 1
mAP = = −
−1 − (−7) 2
3−1 1
mP B = = −
−1 − 3 2
A két szakasz meredeksége egyenlő és emellett van közös pontjuk, ı́gy a három pont egy egye-
nesre esik.♦

139
8. fejezet

8.7. Egyenesek merőlegességének bizonyı́tása


Két egyenes merőleges, ha:

8.7.1. A két egyenes normálvektorának skaláris szorzata 0


Példa :
Merőlegesek-e egymásra az e : 6x − 5y = 8 és f : 7x + 8y = 1 egyenesek?

ne (6; −5) ; nf (7; 8)

A két egyenes nem merőleges, mert:

ne · nf = 42 − 40 = 2♦

8.7.2. A két egyenes meredekségének szorzata -1


Példa :
6
Írjuk fel az e : y = − x + 8 egyenesre merőleges f egyenes egyenletét, amely átmegy a P (4; −3)
5
ponton.

6 5
me = − ⇒ mf =
5 6
5
A keresett egyenes egyenlete: f : y + 3 = (x − 4) ♦
6

8.8. Derékszög bizonyı́tása


Derékszög bizonyı́tásánál egy háromszögben / egy sokszögben / két egyenes metszéspontjánál
következő lehetőségek közül lehet választani:

Példa :
Adott egy háromszög P (−5; 5) ; Q (−1; 7) ; R (3; −1) csúcsa. Bizonyı́tsuk be, hogy a háromszög
derékszögű.

Készı́tünk egy pontos rajzot, amiről látjuk, hogy melyik a háromszög leghosszabb oldala illetve
azt is, hogy melyik szögnél lehetséges a 90◦ .

140
8. fejezet

1. lehetőség:
Kiszámoljuk a három oldal pontos hosszát és felı́rjuk a Pitagorasz-tételt.

q √
PQ = (−5 + 1)2 + (5 − 7)2 = 20
q
PR = (−5 − 3)2 + (5 + 1)2 = 10
q √
QR = (−1 − 3)2 + (7 + 1)2 = 80
√ 2 √ 2
102 = 20 + 80
100 = 100

Igaz a tétel, vagyis a Q csúcsnál derékszög van.

2. lehetőség:
Kiszámoljuk a Q-ból kifelé mutató két vektort, és vesszük azok skaláris szorzatát.

141
8. fejezet

−→ −→
QP (−4; −2) ; QR (4; 8)

Skaláris szorzatuk:
−→ −→
QP · QR = −16 + 16 = 0

Merőlegesek, vagyis a Q csúcsnál derékszög van.

3. lehetőség:
Kiszámoljuk a két oldal meredekségét. A merőleges egyenesek meredekségének szorzata −1.
7−5 1
mP Q = =
−1 − (−5) 2
−1 − 7
mQR = = −2
3 − (−1)

A meredekségek szorzata:

mP Q · mQR = −1♦

8.9. Pont és egyenes távolsága


A pont és egyenes távolsága képlet nem kötelező ugyan, de rendkı́vül hasznos lehet.
Paraméterezést nem igénylő feladatoknál egy több lépésből álló megoldás helyett egy villámgyors
eredményt ad. Paraméterezéses feladatoknál az egyik lehetséges megoldást jelenti.

Ax0 + By0 + C
d= √
A2 + B 2

A képletben a számláló az egyenes nullára rendezett egyenlete, belehelyettesı́tve az adott pont


koordinátái. A nevező az egyenes normálvektorának hossza.

142
8. fejezet

Példa 1 :
Határozzuk meg a 3x − 4y = 8 egyenes távolságát a P (7; 1) ponttól.

A normálvektor: n (3; −4)


Egyenes nullára rendezett egyenlete: 3x − 4y − 8 = 0

3·7−4·1−8 9
d= p = ♦
2
3 + (−4)2 5

Példa 2 :
Határozzuk meg a −5x + 12y = 7 egyenes távolságát a Q (−4; 3) ponttól.

A normálvektor: n (−5; 12)


Egyenes nullára rendezett egyenlete: −5x + 12y − 7 = 0

−5 · (−4) + 12 · 3 − 7 49
d= p = ♦
2
(−5) + 12 2 13

Példa 3 :
Határozzuk meg annak a körnek az egyenletét, amelynek középpontja az O (−3; −2) pont, és
érinti a 2x + y = 3 egyenletű egyenest.

A kör sugara egyenlő a középpont egyenestől való távolságával.


A normálvektor: n (2; 1)
Egyenes nullára rendezett egyenlete: 2x + y − 3 = 0

143
8. fejezet

2 · (−3) + (−2) − 3 11
d= √ =√
22 + 12 5

d általánosságban jelöli a távolságot. Senkit ne tévesszen meg a jelölés, hiszen ez most a kör
121
sugarát jelöli. A kör egyenlete (x + 3)2 + (y + 2)2 = ♦
5
Példa 4 :
Írjuk fel annak az első sı́knegyedbe eső körnek az egyenletét, amely érinti a koordinátatenge-
lyeket és a 3x + 4y = 10 egyenest.

Mivel egyszerre érinti a koordinátatengelyeket, a kör középpontjának koordinátái egyenlők.


K (r; r)

A normálvektor: n (3; 4)
Egyenes nullára rendezett egyenlete: 3x + 4y − 10 = 0

3r + 4r − 10
r = √
32 + 42
7r − 10
r =
5
↙↘
7r − 10 7r − 10
r = −r =
5 5
5
r = 5 r =
6
 2  2
2 2 5 5 25
A két kör egyenlete: (x − 5) + (y − 5) = 25 illetve x − + y− = .♦
6 6 36

144
8. fejezet

Példa 5 :
Egy háromszög csúcspontjai A (1; 0) ; B (0; 4) ; C (c; 6). Határozzuk meg c értékét úgy, hogy a
háromszög területe 13 legyen.

A háromszög AB oldalát vesszük alapnak.


q √
AB = (1 − 0)2 + (0 − 4)2 = 17

Ekkor a terület: √
17m 26
T△ = = 13 ⇒ m = √
2 17

A háromszög magassága nem más, mint a C pont AB egyenestől való távolsága.

Felı́rjuk ı́gy az egyenes egyenletét.


−→
AB (−1; 4) = v AB ⇒ nAB (4; 1) ⇒ AB : 4x + y = 4

Az AB egyenes nullára rendezett egyenlete: 4x + y − 4 = 0

4c + 6 − 4 26
√ = √
42 + 12 17
4c + 2 26
√ = √
17 17
↙↘
4c + 2 26 4c + 2 26
√ = √ √ = −√
17 17 17 17
c = 6 c = −7

A két lehetséges harmadik pont C1 (6; 6) ; C2 (−7; 6). ♦

145
8. fejezet

8.10. Paraméterezés (sugársor)


A sugársor az egy ponton átmenő összes egyenest jelenti. Ahhoz, hogy felı́rhassuk az összes egye-
nest általánosan, be kell vezetnünk egy valós paramétert. Ehhez az egyenes meredekséges egyen-
letét használjuk. A P (x0 ; y0 ) ponton áthaladó összes egyenes egyenlete: y − y0 = m (x − x0 )
Egy egyenes azonban nincs ezek között, mégpedig az y-tengellyel párhuzamos egyenes. En-
nek egyenlete x = x0 . A sugársor csak ezzel a plusz egyenessel együtt teljes. A sugársoros
paraméterezésre általában akkor van szükség, amikor fel kell ı́rni egy adott ponton átmenő
egyenest, ami érint egy kört / parabolát. A sugársor elkerülhetetlen, de kétféleképpen is meg
lehet oldani a feladatokat.

8.10.1. Sugársor egyenletrendszerrel


Egy rajz mindig javasolt, mert több dologra is oda kell figyelni. Először is, ha az a helyzet áll
elő, hogy az egyik érintő párhuzamos az x-tengellyel, mert ez a rajzról leolvasható, akkor a
végén az egyenletnek egyik megoldása 0 kell legyen.
Másrészt, a rajz segı́t abban is, hogy meglássuk az y-tengellyel párhuzamos egyenes érintő-e.
Ha igen, akkor a végén az egyenletnek elsőfokúnak kell lennie.

Példa 1 :
Az (x + 1)2 + (y − 2)2 = 25 egyenletű körhöz érintőket húzunk a P (14; 7) pontból. Írjuk fel az
érintők egyenletét.

146
8. fejezet

Egy pontos rajzzal látható, hogy az egyik érintő most épp párhuzamos az x-tengellyel.
Egyenlete: y = 7
Felı́rjuk a P ponton áthaladó sugársort:

y − 7 = m (x − 14) és x = 14

y = mx − 14m + 7
Az x = 14 jelenleg nem érintő.
Mivel egyenesek és kör metszéspontja kell, fel kell ı́rni az egyenletrendszerüket.

(x + 1)2 + (y − 2)2 = 25
mx − 14m + 7 = y

Behelyettesı́tjük a kör egyenletébe y-t.

(x + 1)2 + (mx − 14m + 5)2 = 25


x2 + 2x + 1 + m2 x2 + 196m2 + 25 − 28m2 x + 10mx − 140m = 25

Kiemeléssel az egynemű kifejezéseket összevonjuk.

1 + m2 x2 + −28m2 + 10m + 2 x + 196m2 − 140m + 1 = 0


 

Ez most a végtelen sok egyenes és a kör metszéspontjait tartalmazó egyenlet. Mivel érintőt kell
keresni, és annak pontosan egy metszéspontja van a körrel, ezért ennek a másodfokú egyen-
letnek pontosan egy valós megoldása van, vagyis a diszkriminánsnak nullának kell lennie. A
diszkriminánsból álló egyenletben viszont már a paraméter lesz az ismeretlen.

a = 1 + m2 ; b = −28m2 + 10m + 2; c = 196m2 − 140m + 1

b2 − 4ac = 0
2
(−28m2 + 10m + 2) − 4 · (1 + m2 ) · (196m2 − 140m + 1) = 0

Nem kell megijedni ettől az egyenlettől, mert ha mindent jól számolunk, akkor másodfokúnak
kell lennie.

784m4 + 100m2 + 4 − 560m3 − 112m2 + 40m


− 784m2 + 560m − 4 − 784m4 + 560m3 − 4m2 = 0

Összevonva:

−800m2 + 600m = 0
m1 = 0
3
m2 =
4
Az m = 0 eredmény igazolja a kezdeti gondolatunkat, hogy az egyik érintő párhuzamos az
x-tengellyel.
3
A másik érintő egyenlete: y − 7 = (x − 14).♦
4

147
8. fejezet

Példa 2 :
Az (x + 1)2 + (y − 2)2 = 25 egyenletű körhöz érintőket húzunk a P (4; 9) pontból. Írjuk fel az
érintők egyenletét.

A pontos rajz segı́t, hogy észrevegyük, az egyik érintő most épp párhuzamos az y-tengellyel.
Egyenlete: x = 4.
Felı́rjuk a P ponton áthaladó sugársort (már a függőleges egyenes nélkül):

y − 9 = m (x − 4)
y = mx − 4m + 9

Mivel egyenesek és kör metszéspontja kell, fel kell ı́rni az egyenletrendszerüket.

(x + 1)2 + (y − 2)2 = 25
mx − 4m + 9 = y

Behelyettesı́tjük a kör egyenletébe y-t:

(x + 1)2 + (mx − 4m + 7)2 = 25


x2 + 2x + 1 + m2 x2 + 16m2 + 49 − 8m2 x + 14mx − 56m = 25

Kiemeléssel megint az egynemű kifejezéseket összevonjuk.

1 + m2 x2 + −8m2 + 14m + 2 x + 16m2 − 56m + 25 = 0


 

Mivel érintőt kell keresni, ezért megint a diszkrimináns 0.

a = 1 + m2 ; b = −8m2 + 14m + 2; c = 16m2 − 56m + 25

148
8. fejezet

b2 − 4ac = 0
2
(−8m2 + 14m + 2) − 4 · (1 + m2 ) · (16m2 − 56m + 25) = 0

Felbontjuk a zárójeleket:

64m4 + 196m2 + 4 − 224m3 − 32m2 + 56m − 64m2 +


+ 224m − 100 − 64m4 + 224m3 − 100m2 = 0

Összevonva:

280m − 96 = 0
12
m =
35
Ahogy a bevezetőben is leı́rtam, nem lepődünk meg, hogy most elsőfokú maradt az egyenlet,
hiszen már megvan az egyik érintő.
12
A másik érintő egyenlete: y − 9 = (x − 4).
35
Tanulság, hogy mindig rajzoljuk le a feladatot. Az egyenletünkből csak egy megoldást kaptunk,
vagyis úgy tűnik, mintha csak egy érintő lenne, holott kettő van. Azt fogjuk hinni, hogy el-
számoltunk valamit, pedig a másik érintő nem is jöhet ki az egyenletrendszerből, mert az a
függőleges egyenes.♦

Példa 3 :
1
Írjuk fel az y = x2 parabolához húzható érintő egyenletét a P (9; 2) pontból.
36

Felı́rjuk a P ponton áthaladó sugársort:

y − 2 = m (x − 9) és x = 9
y = mx − 9m + 2
Az x = 9 jelenleg nem érintő.
Mivel egyenesek és parabola metszéspontja kell, fel kell ı́rni az egyenletrendszerüket.
( 1 2
y = x
36
y = mx − 9m + 2

149
8. fejezet

Egymással egyenlővé téve az egyenleteket:


1 2
x = mx − 9m + 2
36
x2 − 36mx + 324m − 72 = 0

A diszkrimináns most is 0.

a = 1; b = −36m; c = 324m − 72

b2 − 4ac = 0
(−36m)2 − 4 · (324m − 72) = 0
1296m2 − 1296m + 288 = 0
9m2 − 9m + 2 = 0
2
m1 =
3
1
m2 =
3
2 1
A két érintő egyenlete: y − 2 = (x − 9) és y − 2 = (x − 9).♦
3 3

8.10.2. Sugársor a pont és egyenes távolsága képlettel (csak körnél)


Ennél a megoldási útnál azt használjuk ki, hogy csak az érintők vannak pontosan sugárnyi
távolságra a kör középpontjától. Vagyis felı́rjuk a sugársort, belerakjuk a kör középpontját a
pont és egyenes távolsága képletbe, és ennek egyenlőnek kell lennie a sugárral. Az abszolút
érték miatt időnként két megoldás lehetséges.

Példa :
Az (x − 3)2 + (y − 4)2 = 13 egyenletű körhöz érintőket húzunk a P (−5; 5) pontból. Írjuk fel az
érintők egyenletét.

A kör középpontja C (3; 4), sugara r = 13.

150
8. fejezet

Felı́rjuk a P ponton áthaladó sugársort:

y − 5 = m (x + 5) és x = −5

Az x = −5 jelenleg nem érintő.


A sugársor nullára rendezett egyenlete: mx − y + 5m + 5 = 0
n (m; −1)

m · 3 − 4 + 5m + 5 √
p = 13
m2 + (−1)2
8m + 1 √
√ = 13
m2 + 1
↙↘
8m + 1 √ 8m + 1 √
√ = 13 √ = − 13
m2 + 1 m2 + 1
A négyzetre emelés mindkét egyenletet ugyanabba viszi át.
↘↙
2
64m + 16m + 1
= 13
m2 + 1
64m2 + 16m + 1 = 13m2 + 13
2
51m + 16m − 12 = 0
6
m1 =
17
2
m2 = −
3
6 2
A két érintő egyenlete: y − 5 = (x + 5) és y − 5 = − (x + 5).♦
17 3

8.11. Paraméterezés (egyenessereg)


Az egyenessereg az egy egyenessel párhuzamos összes egyenest jelenti. Ahhoz, hogy felı́rhassuk
az összes egyenest általánosan, most is be kell vezetnünk egy valós paramétert. Ehhez az egye-
nes normálvektoros egyenletét használjuk. A P (x0 ; y0 ) ponton áthaladó n (A; B) normálvektorú
egyenes egyenlete: Ax+By = Ax0 +By0 , amit most Ax+By +C = 0 alakban fogunk használni.
A paramétert a konstans helyére vezetjük be, mert tudjuk, hogy a párhuzamos egyenesek egyen-
lete csak a konstansban különbözik.
Az egyenessereges paraméterezésre általában akkor van szükség, amikor fel kell ı́rni egy adott
egyenessel párhuzamos/egyenesre merőleges egyenest, ami érint egy kört/parabolát.
Az egyenessereg elkerülhetetlen, de most is kétféleképpen lehet megoldani a feladatokat.

151
8. fejezet

8.11.1. Egyenessereg egyenletrendszerrel


A rajz most is erősen javasolt. A megoldás gondolatmenete ugyanaz, mint a sugársornál.

Példa 1 :
Határozzuk meg annak az egyenesnek az egyenletét, mely párhuzamos az e : − 3x + 4y = 10
egyenessel és érinti az O (3; 1) középpontú r = 5 sugarú kört.

e-vel párhuzamos egyenessereg egyenlete: −3x + 4y + c = 0


A kör egyenlete: (x − 3)2 + (y − 1)2 = 25

Felı́rjuk az egyenletrendszerüket.

(x − 3)2 + (y − 1)2 = 25
−3x + 4y + c = 0
3x − c
Kifejezve az első egyenletből: y =
4
 2
2 3x − c
(x − 3) + −1 = 25
4
 2
2 3x − c 3x − c
x − 6x + 9 + −2· +1 = 25
4 4
9x2 − 6cx + c2 3x − c
x2 − 6x + 9 + −2· +1 = 25
16 4
16x2 − 96x + 144 + 9x2 − 6cx + c2 − 24x + 8c + 16 = 400
25x2 + (−120 − 6c) x + c2 + 8c − 240 = 0

A diszkrimináns megint 0.

(−120 − 6c)2 − 4 · 25 · (c2 + 8c − 240) = 0


14400 + 1440c + 36c2 − 100c2 − 800c + 24000 = 0
−64c2 + 640c + 38400 = 0
c2 − 10c − 600 = 0
c1 = 30
c2 = −20

A két érintő egyenlete: −3x + 4y + 30 = 0 és −3x + 4y − 20 = 0.♦

152
8. fejezet

Példa 2 :
Határozzuk meg annak az egyenesnek az egyenletét, mely párhuzamos az e : y = −x + 4 egye-
1
nessel és érinti az y = x2 parabolát.
8
e-vel párhuzamos egyenessereg egyenlete: y = −x + c

Felı́rjuk az egyenletrendszerüket.
( 1 2
y = x
8
y = −x + c

Egymással egyenlővé téve az egyenleteket:


1 2
x = −x + c
8
x2 + 8x − 8c = 0

A diszkrimináns 0.

82 − 4 · 1 · (−8c) = 0
c = −2

Az érintő egyenlete: y = −x − 2.♦

8.11.2. Egyenessereg a pont és egyenes távolsága képlettel (csak körnél)


Példa :
Határozzuk meg annak az egyenesnek az egyenletét, mely párhuzamos az e : 4x − 3y = 1 egye-
nessel és érinti az O (4; −7) középpontú r = 3 sugarú kört.

153
8. fejezet

e-vel párhuzamos egyenessereg egyenlete: 4x − 3y + c = 0


n (4; −3)

4 · 4 − 3 · (−7) + c
p = 3
42 + (−3)2
37 + c
= 3
5
↙↘
37 + c 37 + c
= 3 = −3
5 5
c = −22 c = −52

A két érintő egyenlete: 4x − 3y − 22 = 0 és 4x − 3y − 52 = 0.♦

8.12. Húrnégyszög bizonyı́tása


Skaláris szorzattal kiszámoljuk a két szemközti szög koszinuszát. Ha egymás ellentettjei, akkor
180◦ -ra egészı́tik ki egymást a cos (180◦ − φ) = − cos φ összefüggés miatt. Figyelem! Ha be kell
bizonyı́tani, hogy húrnégyszög, nem lehet a szemközti szögek értékeit kiszámolni, mert az csak
kerekı́tés.

154
8. fejezet

Példa 1 :
A P QRS négyszög csúcsai: P (3; −1) ; Q (1; 3) ; R (−6; 2) ; S (−5; −5). Bizonyı́tsuk be, hogy
P QRS négyszög húrnégyszög.

−→ −→
P Q (−2; 4) RQ (7; 1)
−→ −→
P S (−8; −4) RS (1; −7)
−→ −→ −→ −→
PQ · PS = 0 RQ · RS = 0

Vagyis két szemközti szöge derékszög, ı́gy húrnégyszög.♦

Példa 2 :
Az ABCD konvex négyszög oldalegyeneseinek egyenlete a következők: DA : 3x − 4y − 20 = 0;
AB : 3x + 5y − 20 = 0; BC : 4x − 3y + 12 = 0; CD : 5x + 3y + 15 = 0. Bizonyı́tsuk be, hogy
ez a négyszög húrnégyszög.

Szükség van a négy csúcsra, ı́gy megoldjuk a négy egyenletrendszert.

  
DA : 3x − 4y − 20 = 0 20
⇒A ;0
AB : 3x + 5y − 20 = 0 3

AB : 3x + 5y − 20 = 0
⇒ B (0; 4)
BC : 4x − 3y + 12 = 0

BC : 4x − 3y + 12 = 0
⇒ C (−3; 0)
CD : 5x + 3y + 15 = 0

CD : 5x + 3y + 15 = 0
⇒ D (0; −5)
DA : 3x − 4y − 20 = 0
Kiválasztjuk az A és C csúcsokat, létrehozzuk a kifelé mutató vektorokat és kiszámı́tjuk a
hosszukat, illetve a skaláris szorzatuk értékét.

155
8. fejezet

−−→ −−→ √
CD (3; −5) ⇒ CD = 34
−−→ −−→
CB (3; 4) ⇒ CB = 5
−−→ −−→
√CD · CB= −11
5 · 34 · cos γ
= −11
11
cos γ = − √
5 34
   
−→ 20 −−→ 20
AB − ; 4 ; AD − ; −5
3 3
−→ −→
Észrevesszük, hogy AB és AC vektorok koordinátáival egyszerűbb lesz dolgozni, ha vesszük
egy skalár szorosukat úgy, hogy koordinátáik egész számok legyenek. Ekkor természetesen a
hosszuk nem lesz ugyanakkora, de ez nem probléma, mert a hajlásszögük változatlan marad,
ı́gy a skaláris szorzatból ugyanazt kapjuk.

−−→
 
−→ 20
AB − ; 4 ⇒ AB ′ (−5; 3)
3 
−−→

−−→ 20
AD − ; −5 ⇒ AD′ (−4; −3)
3

A skalár szorosokkal számolunk tovább:


−−→′ −−→′ √
AB (−5; 3) ⇒ AB = 34
−−→′ −−→′
AD (−4; −3) ⇒ AD = 5
−−→′ −−→′
√AB · AD = −11
5 · 34 · cos α = 11
11
cos α = √
5 34

A szögeket nem szabad kiszámolni, mert az csak egy kerekı́tés. A koszinuszok értékét kell
megvizsgálni. Mivel α és γ koszinusza egymás ellentettje, egymást 180◦ -ra egészı́tik ki, vagyis
ABCD húrnégyszög.♦

156
8. fejezet

Példa 3 :
Az ABCD négyszög csúcsai A(lg 2; lg 4); B(lg 4; lg 2); C(lg x; lg 4); D(lg 4; lg 8). Határozzuk
meg x értékét úgy, hogy ABCD húrnégyszög legyen.

−→
AB (lg 4 − lg 2; lg 2 − lg 4) = (lg 2; − lg 2)
−−→
AD (lg 4 − lg 2; lg 8 − lg 4) = (lg 2; lg 2)

A koordináták alapján látható, hogy a két vektor egyenlő hosszú, vagyis a négyszög két oldala
egyenlő hosszú.
−→ −−→
AB · AD = lg2 2 − lg2 2 = 0

Innen tudjuk, hogy merőlegesek, vagyis ABCD négyzet, hiszen húrnégyszögnek kell lennie.
Ebből következik, hogy a szemközti szöge is derékszög.

 
−−→ 4
CD (lg 4 − lg x; lg 8 − lg 4) = lg ; lg 2
 x 
−−→ 4
CB (lg 4 − lg x; lg 2 − lg 4) = lg ; − lg 2
x
−−→ −−→
Ezeknek a vektoroknak is egyenlő a hossza. CD · CB skaláris szorzatnak is nullának kell lennie.
−−→ −−→
CD · CB = 0
2 4
lg − lg2 2 = 0
  x 
4 4
lg − lg 2 lg + lg 2 = 0
x x
↙↘
4 4
lg = lg 2 lg = − lg 2
x x
x = 2 4 1
lg = lg
x 2
x = 8

Az első ág eredménye maga az A pont. Az x = 8 viszont jó megoldás. ♦

157
8. fejezet

Példa 4 :
Egy négyszög csúcsai A (2; 0) ; B (8; 0) ; C (2; 8) ; D (0; 4). Bizonyı́tsuk be, hogy a négyszög
húrnégyszög.

−−→ −−→ −−→ −−→ √


BC (−6; 8) ⇒ BC = 10 DC (2; 4) ⇒ DC = 20
−→ −→ −−→ −−→ √
BA (−6; 0) ⇒ BA = 6 DA (2; −4) ⇒ DA = 20
−−→ −→ −−→ −−→
BC · BA = 36 DC · DA = −12
√ √
10 · 6 · cos β = 36 20 · 20 · cos δ = −12
cos β = 0,6 cos δ = −0,6

Mivel β és δ koszinusza egymás ellentettje, egymást 180◦ -ra egészı́tik ki, vagyis a négyszög
húrnégyszög.♦

8.13. Lineáris programozás


A lineáris programozás egy optimalizálási probléma, ami azt jelenti, hogy több ismeretlenből álló
lineáris függvény szélsőértékét kell meghatározni, ha feltételként adottak lineáris egyenlőtlen-
ségek. Minden lineáris programozáshoz rajzolunk. Ezek az adott lineáris egyenlőtlenségek félsı́-
kokat határoznak meg, ı́gy amikor ábrázoljuk mindet, általában egy korlátos, konvex sı́kidom
jön létre. Ez a tartomány az, amiben az összes feltétel teljesül. Ha ez megvan, akkor ezen belül
kell megtalálni egy lineáris függvény szélsőértékét.

158
8. fejezet

Példa 1 :
Oldjuk meg a következő egyenlőtlenség-rendszert az egész számpárok halmazán.

x + 3y < 15

3x + 2y > 14

2x − y < 5

Átrendezzük y-ra az egyenlőtlenségeket, hogy az egyenesek ábrázolása egyszerűbbé váljon.


1


 y < 5− x


 3
3
y > 7 − x
2




y > 2x − 5

Ezek ábrázolása úgy történik, hogy megrajzoljuk az egyeneseket, amiket egyenlőség esetén
kapnánk. Ezek után el kell dönteni az egyenes által létrehozott félsı́kok közül melyik a helyes
félsı́k. Ehhez érdemes behelyettesı́teni az origó koordinátáit (mivel ezek a legegyszerűbbek) az
egyenlőtlenségbe. Ha igazságot kapunk, akkor az origót tartalmazó félsı́k kell. Ha nem kapunk
igazságot, akkor az origót nem tartalmazó félsı́k kell.
1 1
(Pl. y < 5 − x-be behelyettesı́tjük (0; 0)-t, ı́gy 0 < 5, ami igaz, vagyis az y = 5 − x egyenes
3 3
alatti félsı́kra van szükség.)

A három nyı́lt félsı́k közös része meghatároz egy nyı́lt háromszöget. Az egyetlen megoldása az
egyenlőtlenség-rendszernek a (3; 3) mivel az egész számpárok halmazán kell megoldani.♦

Példa 2 : 

 x ≥ 0
y ≥ 0



Keressük meg a 2x + 3y kifejezés maximális értékét, ha x + 2y ≤ 10
x+y ≤ 6




x−4 ≤ 0

Megint átrendezzük az egyenlőtlenségeket könnyen ábrázolható alakra.

159
8. fejezet



 x ≥ 0
y ≥ 0




 1
y ≤ 5− x

 2
y ≤ 6−x




x ≤ 4

Az egyenlőtlenségek ábrázolása után látjuk, hogy a konvex sı́kidom jelenleg az ABCDE pontok
által meghatározott ötszög.
1
A D és E pontokat összekötő egyenes meredeksége − , a D és C pontokat összekötő egyenesé
2
pedig −1.
2
2x + 3y egy egyenest jelent, aminek meredeksége − .
3
2 1
Mivel −1 < − < − , az ötszöget határoló DE és DC egyenesek meredeksége között helyezke-
3 2
dik el 2x + 3y meredeksége, ezért biztos, hogy a D pontban veszi fel a maximumát az ötszögön
belül vagy annak határán. D (2; 4), vagyis 2x + 3y legnagyobb értéke 2 · 2 + 3 · 4 = 16.♦

Példa 3 :
Egy összejövetelre szendvicseket készı́tünk. A sonkás szendvicshez 2 dkg vaj, 2 dkg sonka és
3 dkg franciasaláta kell, mı́g a sajtos szendvicshez 1 dkg vaj, 3 dkg sonka és 2 dkg sajt kell.
Kenyér korlátlanul van, de csak 50 dkg vaj, 80 dkg sonka, 40 dkg sajt és 60 dkg franciasaláta
van. Maximálisan hány szendvicset készı́thetünk?

Legyen x a sonkás szendvicsek száma, y a sajtos szendvicsek száma.


Ekkor a következő egyenlőtlenség-rendszer ı́rható fel:

160
8. fejezet



 2x + y ≤ 50
2x + 3y ≤ 80


 3x ≤ 60
2y ≤ 40

Átrendezve:

≤ 50 − 2x

 y

80 2


y ≤ − x

3 3
≤ 20



 x
≤ 20

y

Az egyenlőtlenségek ábrázolása után látható, hogy most a konvex sı́kidom az ABCDEF pon-
tok által meghatározott hatszög.
2
A D és E pontokat összekötő egyenes meredeksége − , a D és C pontokat összekötő egyenesé
3
pedig −2.
x + y egy egyenest jelent, aminek meredeksége −1.
2
Mivel −2 < −1 < − , a hatszöget határoló DE és DC egyenesek meredeksége között helyez-
3
kedik el x + y meredeksége, ezért biztos, hogy a D pontban veszi fel a maximumát a hatszögön
belül vagy annak határán.
80 2
D koordinátái most nem olvashatók le, ı́gy ki kell számolni a − x = 50 − 2x egyenlet
3 3
megoldását. Ez x = 17,5, ı́gy y = 15.
D (17,5; 15) vagyis x + y legnagyobb értéke 17,5 + 15 = 32,5.
Ez azt jelenti, hogy maximum 32 szendvicset tudunk készı́teni.♦

161
9. fejezet

9. Kombinatorika
9.1. A ,,csomagos” feladatok
Több feladat is arról szól, hogy sorba rendezéskor két ember egymás mellett akar ülni, vagy
ennek meg annak egymás mellett kell elhelyezkednie. Ezeket hı́vjuk ,,csomagos” feladatoknak.
Mindegyiknek az a lényege, hogy azokat az elemeket, amiknek egymás mellett kell elhelyez-
kedniük, be kell rakni egy csomagba, és együtt mozgatni őket. Ha a csomagon belüli sorrend
is változhat, akkor a bent lévő elemek összes lehetséges sorrendjével is meg kell szorozni az
eredményt.

Példa 1 :
Hatan moziba mentek, egymás mellé kaptak jegyet. Hányféleképp ülhetnek le, ha kettő közülük
feltétlen egymás mellé szeretne ülni?

A B C D E F

A hat ember helyett maradt öt elem, ezeket kell permutálni: 5!


A csomagon belüli lehetséges elhelyezkedések száma 2.
A végeredmény 5! · 2 = 240.♦

Példa 2 :
Hányféleképp helyezkedhet el négy házaspár egy egyenes padon úgy, hogy a férjek a feleségük
mellé ülnek?

Férfi Nő Férfi Nő Férfi Nő Férfi Nő

A nyolc ember helyett van négy csomagunk, ezeket permutáljuk: 4!


Minden csomagon belüli a párok kétféleképp ülhetnek le.
A végeredmény 4! · 24 = 384.♦

Példa 3 :
Hányféleképp ülhet le egy három-, egy négy- és egy ötfős család egymás mellé egy egyenes
padra, ha a családtagok egymás mellett akarnak ülni?

A B C D E F G H I J K L

A tizenkét ember helyett most három csomagunk van, ezek permutációinak száma: 3!
A csomagokon belül az emberek 3! , 4! illetve 5! féleképp ülhetnek le.
A végeredmény 3! · 3! · 4! · 5! = 103 680.♦

Példa 4 :
A 12.c osztálynak hétfőn van angol, matek, testnevelés, biológia, irodalom, történelem és fizika
órája. Hányféle órarend készı́thető hétfőre, ha a testnevelést történelem követi?

Angol Tesi Töri Biológia Irodalom Fizika Matek

162
9. fejezet

A hét tantárgy helyett most hat van, ezek permutációinak száma: 6!


A feladat most úgy szól, hogy a testnevelést történelem követi, tehát a csomagon belül most
nem változtatható a sorrend.
A végeredmény 6! = 720.♦

9.2. Esetszétválasztás a nulla miatt


A nulla fejtörést okozhat a helyi értékes feladatokban, mert nem állhat a legelső helyi érték
helyén. Emiatt időnként esetszétválasztásra lehet szükség. A következő példákban azt szeretném
érzékeltetni, hogy mikor van erre szükség és mikor nincs, emiatt mindegyik feladatot rögtön
kétféle feltétel mellett is megoldunk: amikor a számjegyeknek szabad ismétlődniük, és amikor
nem szabad.

Példa 1 :
Hány ötjegyű szám létezik a tı́zes számrendszerben?

Ha a számjegyek ismétlődhetnek Ha a számjegyek különböznek

9 10 10 10 10 9 9 8 7 6

Az első helyen nulla nem állhat, a többi he- Az első helyen nulla nem állhat. A második
lyen bármelyik számjegy szerepelhet. helyen megint kilenc lehetőség van, mert el-
vesztettük azt a számjegyet, ami legelőre
került, de visszajött a nulla, mint lehetőség.
Utána folyamatosan fogynak a lehetőségek,
egyesével.

A végeredmények: 9 · 10 · 10 · 10 · 10 = 90 000 illetve 9 · 9 · 8 · 7 · 6 = 27 216. ♦

Példa 2 :
Hány ötjegyű páratlan szám létezik a tı́zes számrendszerben?

Amikor a számok végződései korlátozottak, például egy oszthatósági feltétel miatt, akkor ezek-
nek a meghatározásával kell kezdeni, és csak utána lehet elölről kiszámolni a lehetőségeket.

Ha a számjegyek ismétlődhetnek Ha a számjegyek különböznek

9 10 10 10 5 8 8 7 6 5

Az utolsó helyi értéken csak öt lehetőség Az utolsó helyi értéken csak öt lehetőség
közül lehet választani: {1; 3; 5; 7; 9}. Az első közül lehet választani: {1; 3; 5; 7; 9}. Az első
helyen nulla nem állhat, a többi helyen helyen nulla sem állhat, és a hátul lerögzı́tett
bármelyik számjegy szerepelhet. számjegy sem. A második helyen megint
nyolc lehetőség van, mert elvesztettük a leg-
elsőt és a leghátsót, de nulla ide már le-
hetséges. Utána folyamatosan fogynak a le-
hetőségek, egyesével.

A végeredmények: 9 · 10 · 10 · 10 · 5 = 45 000 illetve 8 · 8 · 7 · 6 · 5 = 13 440. ♦

163
9. fejezet

Példa 3 :
Hány ötjegyű páros szám létezik a tı́zes számrendszerben?

Ha a számjegyek ismétlődhetnek Ha a számjegyek különböznek

9 10 10 10 5 8 8 7 6 4

Az utolsó helyi értéken csak öt lehetőség Az utolsó helyi értéken {2; 4; 6; 8} halmazból
közül lehet választani: {0; 2; 4; 6; 8}. Az első lehet választani. Az első helyen nulla sem
helyen nulla nem állhat, a többi helyen állhat, és a hátul lerögzı́tett számjegy sem. A
bármelyik számjegy szerepelhet. második helyen megint nyolc lehetőség van,
mert elvesztettük a legelsőt és a leghátsót, de
nulla ide már lehetséges. Utána folyamatosan
fogynak a lehetőségek, egyesével.
VAGY
0
9 8 7 6
Az utolsó helyi értéken a 0 szerepel. Ekkor
elöl kilenc lehetőség van, utána nyolc, stb.

A végeredmények: 9 · 10 · 10 · 10 · 5 = 45 000 illetve 8 · 8 · 7 · 6 · 4 + 9 · 8 · 7 · 6 · 1 = 13 776. ♦

Példa 4 :
Hány ötjegyű öttel osztható szám létezik a tı́zes számrendszerben?

Ha a számjegyek ismétlődhetnek Ha a számjegyek különböznek


5
9 10 10 10 2 8 8 7 6
Az utolsó helyi értéken csak két lehetőség Az utolsó helyi értéken ötös szerepel. Az első
van: {0; 5}. Az első helyen nulla nem állhat, helyen nulla sem állhat, és a hátul lerögzı́tett
a többi helyen bármelyik számjegy szerepel- ötös számjegy sem. A második helyen megint
het. nyolc lehetőség van, mert elvesztettük a leg-
elsőt és az ötöst, de nulla ide már lehetséges.
Utána folyamatosan fogynak a lehetőségek,
egyesével.
VAGY
0
9 8 7 6
Az utolsó helyi értéken a nulla van. Ekkor
elöl kilenc lehetőség van, utána nyolc, stb.

A végeredmények: 9 · 10 · 10 · 10 · 2 = 18 000 illetve 8 · 8 · 7 · 6 · 1 + 9 · 8 · 7 · 6 · 1 = 5712. ♦

164
9. fejezet

Példa 5 :
Hány ötjegyű néggyel osztható szám létezik a tı́zes számrendszerben?

Ha a számjegyek ismétlődhetnek Ha a számjegyek különböznek

9 10 10 | {z } 7 7 6 | {z }
25 16
Az utolsó két helyi értéken összesen 25 le- A bal oldalon szereplő 25 lehetőségből három
hetőség van: {00; 04; 08; 12; . . . ; 96}. Az első kiesik az ismétlődés miatt: {00; 44; 88}, ı́gy
helyen nulla nem állhat, a többi helyen marad 22 lehetőség. Ebből 16 nem tartal-
bármelyik számjegy szerepelhet. maz nullát: {12; 16; . . . ; 96}. De az első he-
lyen nulla sem állhat, és a hátul lerögzı́tett je-
gyek számjegy sem. A második helyen megint
hét lehetőség van, mert elvesztettük a leg-
elsőt is, de a nulla visszatért. Utána már csak
hat lehetőség van.
VAGY

8 7 6 | {z }
6
Nullát tartalmazó lehetséges végződés hat
van: {04; 08; 20; 40; 60; 80}. Ekkor elöl nyolc
lehetőség van, utána hét, utána hat.

A végeredmények: 9 · 10 · 10 · 25 = 22 500 illetve 7 · 7 · 6 · 16 + 8 · 7 · 6 · 6 = 6720. ♦

Ezekből a példákból jól lehet látni, hogy nincs szükség esetszétválasztásra akkor, amikor a
számjegyek ismétlődhetnek, és amikor nem, akkor is csak akkor van rá szükség, ha a nulla le-
hetséges végződés valahol hátul.

9.3. A szétbontásos feladatok #1


Adott n elem, és ezeket egy tulajdonság szerint kisebb csoportokra bontjuk. Ez a tulajdonság
a szövegből kinyerhető. A kisebb csoportokból választunk adott mennyiségű elemet. Az utolsó
csoport általában a ,,maradékokat” tartalmazza, ebből választjuk ki a ,,maradék” elemeket.
A feladat mindig arról szól, hogy ezekből a kisebb csoportokból választunk elemeket, és ezt
hányféleképp tehetjük meg.
Van egy jól átlátható rajz, ami segı́t az ilyen feladatok megoldásában. A kiválasztás mindig
ismétlés nélküli kombináció alapján történik, mert nem tesszük vissza a korábban kihúzott ele-
meket és nem számı́t a kiválasztás sorrendje sem.

Példa 1 :
Hányféleképpen húzhatunk a 32 lapos magyar kártyából hat lapot úgy, hogy pontosan két piros
legyen köztük?

Jelenleg a tulajdonság, ami szerint szétválasztjuk a 32 elemet az, hogy a lap piros, vagy nem.
Ebben az esetben nem érdekes, hogy a többi szı́n micsoda, elég két csoport.

165
9. fejezet

32 lap
↙ ↘
8 piros 24 többi
↓ ↓
2 4
Az ágrajz vizuálisan
  is segı́t,
 hogy a 8-as szám alatt a 2-es szám van, a 24-es alatt pedig a 4-es.
8 24
A végeredmény = 297 528 ♦
2 4

Példa 2 :
Egy 33 fős osztályban van 14 fiú és 19 lány. Hányféleképp lehet kiválasztani az osztályból hét
embert, hogy a kiválasztottak között pontosan két fiú legyen?

Most a tulajdonság az, hogy egy diák fiú vagy lány.

33 gyerek
↙ ↘
14 fiú 19 lány
↓ ↓
2 5
  
14 19
A végeredmény = 1 058 148 ♦
2 5

Példa 3 :
Egy 30 fős osztályban 13-an budapestiek, 12-en bejárósak és öten kollégisták. Hányféleképp le-
het választani közülük hét gyereket úgy, hogy pontosan két budapesti és pontosan egy kollégista
legyen?

Most a tulajdonság az, hogy honnan járnak be, ı́gy három csoport van.

30 gyerek
↙ ↓ ↘
13 Bp. 12 bej. 5 koll.
↓ ↓ ↓
2 4 1
   
13 12 5
A végeredmény = 193 050 ♦
2 4 1

Példa 4 :
A 12. évfolyamba jár 21 A osztályos, 24 B osztályos és 27 C osztályos tanuló. Kiválasztunk
közülük öt embert. Hányféleképp lehet köztük pont két A és három C osztályos?

Most a tulajdonság az, hogy melyik osztályba járnak.

72 gyerek
↙ ↓ ↘
21 A-s 24 B-s 27 C-s
↓ ↓ ↓
2 0 3

166
9. fejezet

   
21 24 27
A végeredmény = 614 250 ♦
2 0 3

Mint az a példákból látható, a szétbontogatás után felı́rjuk a binomiális összefüggéseket. Ha


mindent jól csinálunk, akkor a ,,felső” számok összege megegyezik az összes elem számával, az
,,alsó” számok összege pedig megegyezik a kiválasztott elemek számával. Érdemes azt is odaı́rni,
ha valamelyik csoportból nulla elemet veszünk ki, mert ı́gy látható marad ez a struktúra.

9.4. A szétbontásos feladatok #2


A 9.3 fejezetben azokkal az esetekkel foglalkoztunk, amikor a kiválasztás úgy történt, hogy
pontosan adott elemszámot kellett választani. Gyakrabban fordul elő viszont az az eset, amikor
a feladat úgy szól, hogy legalább vagy legfeljebb valamennyit kell választani.
Amikor ez a helyzet áll elő, végig kell menni az összes lehetőségen, kiszámolni ezek lehetséges
eseteinek a számát és az eredményeket összeadni.

Példa 1 :
Hányféleképpen húzhatunk a 32 lapos magyar kártyából hat lapot úgy, hogy legalább öt piros
legyen köztük?

A legalább öt piros azt jelenti, hogy pontosan öt vagy pontosan hat piros lehet közöttük.

32 lap 32 lap
↙ ↘ ↙ ↘
8 piros 24 többi 8 piros 24 többi
↓ ↓ ↓ ↓
5 1 6 0
     
8 24 8 24
A végeredmény + = 1372 ♦
5 1 6 0

Példa 2 :
20 termékből négy selejtes. Kiválasztunk hatot. Hányféleképp lehet köztük legfeljebb két selej-
tes?

A legfeljebb két selejtes azt jelenti, hogy a selejtesek száma 0, 1 vagy 2.

20 termék 20 termék 20 termék


↙ ↘ ↙ ↘ ↙ ↘
4 selejt 16 jó 4 selejt 16 jó 4 selejt 16 jó
↓ ↓ ↓ ↓ ↓ ↓
0 6 1 5 2 4
        
4 16 4 16 4 16
A végeredmény + + = 36 400 ♦
0 6 1 5 2 4

Ezekben az esetekben a ,,felső” számok változatlanok maradnak, csak az ,,alsó” számokat


változtatjuk. Az egyik csökken eggyel, a másik nő eggyel.

167
9. fejezet

9.5. Az ,,összes-rossz” módszer


Elképzelhető, hogy a feladatban szereplő esetekből nagyon sok van. Ezek összeszámolása bo-
nyolult, sokáig tart és nagy a hibalehetőség is. Ilyenkor gondoljunk arra, hogy helyette inkább
összeszámoljuk azokat az eseteket, amikor nem teljesül. Ezeket hı́vjuk rossz eseteknek. Ezek
összeszámolása könnyebb lehet. Ha viszont ezt számoljuk ki, akkor a rossz esetek számát ki kell
vonni az összes lehetséges esetből, hogy megkapjuk a jó esetek számát.
Minden feladatnak van egy fő része, és egy feltétele (ez általában a vessző vagy az ami, amely
szavak után jön). Az összes eset számánál csak a fő résszel kell foglalkozni, és kiszámolni a
lehetőségeket. A rossz esetek számánál pedig letagadjuk a feladat feltételét.
Mind kombinatorikában, mind valószı́nűségszámı́tásban, akkor javasolt ezt a taktikát alkalmaz-
ni, ha a rossz esetek számából jóval kevesebb van, mint a jó lehetőségek számából.

Példa 1 :
Hány olyan hatjegyű szám létezik a tı́zes számrendszerben, amelyben létezik egyes számjegy?

Gondoljuk végig, hány darab egyes számjegy lehet egy hatjegyű számban.
h i
0
|{z} 1| 2 3{z4 5 6}

A feladatban lévő létezik egyes számjegy azt jelenti, hogy az egyesek száma lehet: 1, 2, 3, 4,
5, 6. Ez túl sok eset, ezért inkább letagadjuk, mert a tagadásból származó esetek száma egy,
vagyis 0 darab egyes van benne.

Összes eset (ez a fő feladat): Rossz eset (feltétel tagadása):


Hány hatjegyű szám van 0 darab egyes van benne

9 10 10 10 10 10 8 9 9 9 9 9

A végeredmény 9 · 105 − 8 · 95 = 427 608.♦

Példa 2 :
Egy 25 fős osztályban 8 fiú és 17 lány van. Hányféleképp lehet kiválasztani egy négyfős csopor-
tot úgy, hogy legyen köztük fiú?

Hány fiú lehet a négyfős csoportban?


h i
0
|{z} 1| 2{z3 4}

A feladatban szereplő legyen fiú köztük azt jelenti, hogy a fiúk száma lehet: 1, 2, 3, 4. Ez négy
eset, ezért inkább letagadjuk, mert a tagadásból származó esetek száma egy, vagyis nincs fiú a
kiválasztott emberek között.

168
9. fejezet

Összes eset (ez a fő feladat): Rossz eset (feltétel tagadása):


25 főből választunk 4 főt nincs fiú köztük
25 gyerek
  ↙ ↘   
25 8 17
8 fiú 17 lány
4 0 4
↓ ↓
0 4
    
25 8 17
A végeredmény − = 10 270.♦
4 0 4

Példa 3 :
20 darab készüléknek 45%-a hibás. Hányféleképp lehet hét készüléket úgy kiválasztani, hogy a
kiválasztott készülékek között legalább két hibás legyen?

Hány hibás lehet a hét készülék között?


h i
0|{z}1 2| 3 4{z5 6 7}

A szövegben szereplő legalább két hibás azt jelenti, hogy a hibások száma lehet: 2, 3, 4, 5, 6, 7.
Ez hat eset, ezért megint letagadjuk, de most a tagadásból származó esetek száma kettő, vagyis
nincs hibás vagy egy hibás.

Összes eset (ez a fő feladat): Rossz eset (feltétel tagadása):


20 készülékből választunk 7-et nincs hibás vagy 1 hibás van
20 termék 20 termék
  ↙ ↘ ↙ ↘      
20 9 11 9 11
9 hibás 11 jó 9 hibás 11 jó +
7 0 7 1 6
↓ ↓ ↓ ↓
0 7 0 7
  "    #
20 9 11 9 11
A végeredmény − = 73 032.♦
7 0 7 1 6

Példa 4 :
Adott az A = {0; 1; 2; 3; 4; 5} halmaz. Az A halmaz elemeiből hány olyan hatjegyű szám ı́rható
fel, amely legalább egy egyest tartalmaz? A számjegyeket többször is felhasználhatjuk.

Megint gondoljuk végig hány darab egyes számjegy lehet egy hatjegyű számban.
h i
0
|{z} | 1 2 3 {z 4 5 6
}

A feladatban lévő legalább egy egyes számjegy megint azt jelenti, hogy az egyesek száma lehet:
1, 2, 3, 4, 5, 6. Ha viszont a maradék eseteket nézzük, akkor csak egy eset van, vagyis 0 darab
egyes van benne.

169
9. fejezet

Összes eset (ez a fő feladat): Rossz eset (feltétel tagadása):


Hány hatjegyű szám van 0 darab egyes van benne

5 6 6 6 6 6 4 5 5 5 5 5

A végeredmény 5 · 65 − 4 · 55 = 26 380.♦

9.6. Összegre vagy szorzatra bontós feladatok


Amikor az a feladat, hogy hányféleképp lehet felbontani egy számot több egész szám szorzatára
vagy összegére, akkor érdemes a legnagyobb lehetséges összetevőt felı́rni első helyre és azt
folyamatosan csökkenteni. Így nem hagyunk ki egy lehetőséget sem. A sorrendje permutálással
kapható meg. Ha minden összetevő különböző, akkor ismétlés nélküli, ha van közöttük azonos,
akkor ismétléses.

Példa 1 :
Hányféleképp lehet felbontani a hatot három pozitı́v egész szám összegére, ha a számok
sorrendjét megkülönböztetjük?

Az első helyre a legnagyobb számot vesszük, ami a 4. Kiszámoljuk hozzá az összes többi
lehetséges összeadandót, majd ezt a legnagyobb számot fokozatosan csökkentjük.
3!
6 = 4+1+1 → = 3
2!
6 = 3 + 2 + 1 → 3! = 6
3!
6 = 2+2+2 → = 1
3!
Összesen 10 féle felbontása létezik.♦

Példa 2 :
Hányféleképp lehet felbontani a kilencet öt pozitı́v egész szám összegére, ha a számok
sorrendjét megkülönböztetjük?

Az első helyre megint a legnagyobb számot vesszük. Ez most az 5. Ezt a legnagyobb számot
fokozatosan csökkentjük.
5!
9 = 5+1+1+1+1 → = 5
4!
5!
9 = 4+2+1+1+1 → = 20
3!
5!
9 = 3+3+1+1+1 → = 10
3! 2!
5!
9 = 3+2+2+1+1 → = 30
2! 2!
5!
9 = 2+2+2+2+1 → = 5
4!
Összesen 70 féle felbontása létezik.♦

170
9. fejezet

Példa 3 :
Hányféleképp lehet felbontani a 24-et négy pozitı́v egész szám szorzatára, ha a számok
sorrendjét megkülönböztetjük?

Az első helyre a 24 kerül, majd utána csökkenő sorrendben a 24 osztóit ı́rjuk.


4!
24 = 24 · 1 · 1 · 1 → = 4
3!
4!
24 = 12 · 2 · 1 · 1 → = 12
2!
4!
24 = 8·3·1·1 → = 12
2!
4!
24 = 6·4·1·1 → = 12
2!
4!
24 = 6·2·2·1 → = 12
2!
24 = 4·3·2·1 → 4! = 24
4!
24 = 3·2·2·2 → = 4
3!
Összesen 80 féle felbontása létezik.♦

Példa 4 :
Egy szabályos játékkocka két oldalára nullát, két oldalára kettest, két oldalára négyest ı́runk.
A dobókockát ötször egymás után feldobjuk, és a dobások eredményét rendre feljegyezzük.
Hányféle számötös esetében lehet a dobott pontok összege tı́z?

5!
10 = 4 + 4 + 2 + 0 + 0 → = 30
2! 2!
5!
10 = 4 + 2 + 2 + 2 + 0 → = 20
3!
5!
10 = 2 + 2 + 2 + 2 + 2 → = 1
5!
Összesen 51 féle számötös létezik.♦

171
10. fejezet

10. Valószı́nűségszámı́tás
10.1. A ,,csomagos” feladatok
A ,,csomagos” feladatok valószı́nűségszámı́tásban is előfordulhatnak. A kedvező esetek
számához ugyanazt alkalmazzuk, amit a 9.1. fejezetben is, a nevező pedig az összes lehetőség
száma.

Példa 1 :
Mekkora a valószı́nűsége, hogy ha a DZSUNGEL szó betűit összekeverjük, akkor a D, Z, S
betűk egymás mellé kerülnek? Mekkora a valószı́nűsége, ha azt is megköveteljük, hogy DZS
sorrendben?

A három betűt berakjuk egy csomagba, egy elemnek vesszük, ı́gy a csomagot és a maradék öt
betűt, azaz hat elemet kell permutálni. Ez 6! lehetőség. A csomagon belül is állhatnak a betűk
3! sorrendben.
6! · 3! 3
p= =
8! 28

Ha a három betű sorrendje rögzı́tett, akkor a csomagon belüli lehetőségek száma egy. Így ke-
vesebb a lehetőségek száma.
6! 1
p= = ♦
8! 56

Példa 2 :
Egy csomag magyar kártyát jól összekeverünk. Mennyi annak a valószı́nűsége, hogy a négy ász
egymás után helyezkedik el?

A négy ászt csomagba rakjuk. Permutáljuk a maradék 28 lapot és a csomagot, ez 29! lehetőség.
A csomagon belül is állhatnak a lapok 4! sorrendben.
29! · 4! 1
p= = ≈ 0,0008♦
32! 1240

10.2. A két kockadobásos feladatok


Azoknak a feladatoknak, amikor feldobunk két kockát egyszerre, és a dobott értékeket össze-
adjuk, kivonjuk, szorozzuk, négyzetüket kivonjuk, stb. van egy mindig működő, könnyű és
átlátható megoldása. Készı́tünk egy 6 × 6-os táblázatot, ı́gy látható, hogy 36 eset lesz összesen.
A táblázatot kitöltjük a feladat szövege szerint és megszámoljuk hányszor teljesül a feltétel a
36 esetből.

Példa 1 :
Két kockát dobálunk egyszerre, majd a kapott értékeket kivonjuk egymásból (a nagyobból a
kisebbet). Melyik értéknek a legnagyobb a valószı́nűsége? Mi a valószı́nűsége annak, hogy a
kapott érték legalább 1 de legfeljebb 3?

172
10. fejezet

A táblázat alapján látható, hogy a leggyakoribb különbség az 1, ı́gy ennek a legnagyobb a


valószı́nűsége.
Az 1 különbség tı́zszer, a 2 különbség nyolcszor, a 3 különbség pedig hatszor fordul elő, ı́gy
24
annak a valószı́nűsége, hogy a kapott érték 1 és 3 között van .♦
36
Példa 2 :
Két kockát feldobunk egyszerre, majd az értékeket összeadjuk. Mennyi a valószı́nűsége, hogy
az összeg prı́m? Mennyi annak a valószı́nűsége, hogy a dobott számok összege 7, feltéve, hogy
az összeg páratlan?

Prı́m a 2, 3, 5, 7, 11. Ezek összesen 15-ször szerepelnek a táblázatban, ı́gy a prı́mösszeg


15
valószı́nűsége .
36
Ha van egy feltétel, akkor ahhoz képest kell nézni, hányszor jön ki a 7, mint összeg. Páratlan
6
összeg összesen 18-szor jön ki, ebből hat esetben 7, ı́gy a valószı́nűség .♦
18
Példa 3 :
Két kockát feldobunk egyszerre, majd az értékeket összeszorozzuk. Mennyi a valószı́nűsége,
hogy a szorzat prı́m? Mennyi annak a valószı́nűsége, hogy a szorzat legalább 20, feltéve, hogy
a szorzat páros?

173
10. fejezet

6
Prı́m a 2, 3, 5. Ezek összesen hatszor szerepelnek, ı́gy a prı́mszorzat valószı́nűsége .
36
7
A szorzat 27 esetben páros, ebből hét esetben legalább 20, ı́gy a valószı́nűsége .♦
27
Példa 4 :
Egy kockával kétszer dobunk. Mennyi a valószı́nűsége, hogy az első dobás eredménye nagyobb,
mint a másodiké?

A táblázat üres, de attól még bejelöljük mely esetekben lesz az első dobás eredménye a nagyobb.
15
15 esetben következik ez be, ı́gy a valószı́nűség .♦
36

10.3. A hipergeometrikus eloszlás


A hipergeometrikus eloszlás a visszatevés nélküli mintavétel modellje. A 9.3. illetve 9.4.
fejezetben tárgyalt szétbontásos feladatokon múlik a gondolatmenet, kiegészı́tve azzal, hogy
el kell osztani az összes eset számával.

Példa 1 :
Húsz alma közül hét férges. Kiveszünk közülük válogatás nélkül ötöt. Mennyi a valószı́nűsége,
hogy két férges lesz?

  
13 7
3 2
p =   ≈ 0,3874♦
20
5

Példa 2 :
A szekrényben 13 szoknya van, három fehér, négy sötét és hat pöttyös. Utazáshoz négyet
véletlenszerűen kiválasztok. Mekkora a valószı́nűsége, hogy két vagy három fehér van köztük?

A két fehér, illetve a három fehér különböző eseteket jelent, vagyis ezek valószı́nűségeit össze-
adjuk.
     
3 10 3 10
2 2 3 1
p =   +   ≈ 0,2028♦
13 13
4 4

174
10. fejezet

Példa 3 :
100 darab villanykörtéből 13 hibás. Kiválasztunk egy 10 elemű mintát. Mekkora a valószı́nűsége,
hogy pont két hibás van köztük?

  
13 87
2 8
p =   ≈ 0,2633♦
100
10

Példa 4 :
Mekkora annak a valószı́nűsége, hogy a következő ötös lottóhúzáson csupa páros vagy csupa
páratlan számot húznak ki?

        
45 45 45 45 45 45
0 5 5 0 0 5
p=   +   =2·   ≈ 0,0556♦
90 90 90
5 5 5

A 9.4. fejezet utolsó gondolatmenetében szerepelt, hogy a szétbontogatós feladatoknál a ,,felső”


számok összege megegyezik az összes elem számával, az ,,alsó” számok összege pedig megegyezik
a kiválasztott elemek számával. Ez most is igaz, és még látványosabb. A nevező alapvetően az
összes eset számát jelenti minden valószı́nűségszámı́tás feladatban, ı́gy a nevezőben pont ez a
két szám látható ,,felül” és ,,alul”. Például:
  
13 87
2 8
 
100
10

10.4. A binomiális eloszlás


A binomiális eloszlás a visszatevéses mintavétel modellje. Egymástól függetlenül elvégezzük
ugyanazt a kı́sérletet n-szer és figyeljük A esemény bekövetkezik-e vagy sem.
A esemény valószı́nűsége p. (Ezt vagy tudjuk, mert benne van a feladatban, vagy a klasszikus
képlettel hamar kiszámolható.) k alkalommal bekövetkezik A esemény, n − k alkalommal nem
következik be A esemény, tehát A következik be, aminek a valószı́nűsége q = 1 − p.
Ekkor annak a valószı́nűsége, hogy A esemény n kı́sérletből pontosan k-szor következik be:
 
n k n−k
p ·q
k
.

Példa 1 :
Mennyi a valószı́nűsége, hogy tı́z kockadobásból pontosan kétszer dobunk hatost?

Kı́sérletek száma: n = 10
A esemény az, hogy hatost dobunk.
1 5
p= ⇒q=
6 6

175
10. fejezet

Bekövetkezések száma: k = 2
   2  8
10 1 5
· · ≈ 0,2907♦
2 6 6

Példa 2 :
Egy üzemben a napi nyersanyagellátás, egymástól függetlenül, 0,75 valószı́nűséggel zavartalan.
Mennyi a valószı́nűsége, hogy egy hét alatt (öt nap) pontosan háromszor zavartalan az ellátás?

Kı́sérletek száma: n = 5
A esemény az, hogy zavartalan a működés.
p = 0,75 ⇒ q = 0,25
Bekövetkezések száma: k = 3
 
5
· 0,753 · 0,252 ≈ 0,2637♦
3

Példa 3 :
A Sport szelet új játéka szerint minden negyedik csokoládéval nyerni lehet. Veszünk száz darab
Sport szeletet. Mennyi a valószı́nűsége, hogy tı́zzel nyerünk?

Kı́sérletek száma: n = 100


A esemény az, hogy nyerünk.
1 3
p= ⇒q=
4 4
Bekövetkezések száma: k = 10
   10  90
100 1 3
· · ≈ 0,000094♦
10 4 4

Példa 4 :
A lakosság 30%-a szenved valamilyen allergiás betegségben. Munkatársaink közül tetszőlegesen
kiválasztva 12 főt, mennyi annak a valószı́nűsége, hogy három vagy négy szenved allergiás be-
tegségben?

Kı́sérletek száma: n = 12
A esemény az, hogy allergiás.
p = 0,3 ⇒ q = 0,7
Bekövetkezések száma: k = 3; 4
   
12 3 9 12
· 0,3 · 0,7 + · 0,32 · 0,78 ≈ 0,4708♦
3 4

Példa 5 :
Dani sportlövészedzésre jár, ahol koronglövészetet tanul. Az első félév végén kiderült, hogy
még elég bizonytalanul céloz: húsz lövésből átlagosan ötször találja el a repülő agyagkorongot.
5
(Tekintsük ezt úgy, hogy minden lövésnél az esélye annak, hogy Dani találatot ér el. A
20
rendszeres edzéseknek köszönhetően Dani eredményessége javult. A második félév végén már

176
10. fejezet

0,72 volt annak a valószı́nűsége, hogy három egymás után leadott lövésből pontosan egy vagy
pontosan két találatot ér el. Számı́tsuk ki, hogy a második félév végén mekkora valószı́nűséggel
ér el találatot egy lövésből Dani.

Kı́sérletek száma: n = 3
A esemény az, hogy eltalálja a korongot.
p értékét nem tudjuk most
Bekövetkezések száma: k = 1; 2
   
3 1 2 3
· p · (1 − p) + · p2 · (1 − p)1 = 0,72
1 2
3p(1 − p)2 + 3p2 (1 − p) = 0,72
3p − 6p2 + 3p3 + 3p2 − 3p3 = 0,72
3p2 − 3p + 0,72 = 0
p1 = 0,4
p2 = 0,6
Mindkét megoldás jó, tehát vagy 0,4 vagy 0,6 valószı́nűséggel ér el egy találatot.♦

A feladat szövegéből nem könnyű eldönteni, mikor melyik eloszlás kell. Ehhez a következő
kérdést érdemes feltenni magunknak:

A korábban kihúzott elemeket visszatesszük?


↙ ↘
Igen Nem
⇓ ⇓
binomiális hipergeometrikus

De nem minden feladat szól arról, hogy valóban választunk elemeket és kihúzzuk őket (pl. kocka-
dobások). Ebben az esetben azt kell megkérdeznünk magunkban, hogy függetlenek-e egymástól
az események.
Ha elvégezzük az első elemmel a kı́sérletet,
az befolyásolja a valószı́nűségét annak,
hogy a következő kı́sérlet eredménye mi lesz?
↙ ↘
Nem Igen
⇓ ⇓
binomiális hipergeometrikus

Ha kockával egyest dobunk elsőre, akkor a második dobásnál is ugyanakkora a valószı́nűsége


az összes számnak. Ezt jelenti a függetlenség. Ha egy csoportból, amiben van tı́z lány és tı́z fiú,
kiválasztunk egy lányt, ezek után a lány kiválasztásának valószı́nűsége csökken, a fiúké nő. Itt
érezhető, hogy nincs függetlenség.

Az eldöntésben segı́thet még az is, hogy a hipergeometrikus eloszláshoz tudnunk kell hány
elemünk van, és hányat választunk ebből. A binomiális eloszláshoz nem kell tudnunk, hány
elemünk van összesen, ezért például amikor arról van szó, hogy a lakosság adott százaléka
valamilyen tulajdonságú, az binomiális.

177
10. fejezet

10.5. A teljes valószı́nűség tétele


Egy adott mennyiségű elemet kisebb csoportokra bontunk egy tulajdonság szerint, százalékok
vagy arányok alapján. (Általában nem ismerjük, hány elemünk van.) Az összes kisebb csopor-
tot újabb kisebb csoportokra bontjuk egy második tulajdonság szerint, megint csak százalékok
vagy arányok alapján. Választunk egy elemet. A teljes valószı́nűség tételénél a kérdés mindig
az, hogy mekkora a valószı́nűsége, hogy ez az elem rendelkezik a második tulajdonsággal.
Egy ágrajzzal egyszerűvé és látványossá lehet tenni a megoldást. A végeredményt úgy lehet
kiszámolni, hogy az egyes ágakhoz tartozó százalékokat átváltjuk tizedes törtekre, összeszoroz-
zuk, majd az ágakhoz tartozó számokat összeadjuk.

Példa 1 :
Egy üzemben három hétig azonos alkatrészeket gyártanak. Az egyes heteken rendre a termelés
30; 25 és 45%-át készı́tik el, a selejtszázalék pedig 7; 6 és 8%. Mi a valószı́nűsége annak, hogy
a teljes termelésből kiválasztott egyetlen termék selejt?

Az első tulajdonság, hogy melyik héten készült, a második tulajdonság, hogy selejt, vagy nem.

alkatrészek
↙ ↓ ↘
1. hét 2. hét 3. hét
30% 25% 45%
↙ ↘ ↙ ↘ ↙ ↘
selejt jó selejt jó selejt jó
7% 93% 6% 94% 8% 92%

p = 0,3 · 0,07 + 0,25 · 0,06 + 0,45 · 0,08 = 0,072♦

Figyelem! Az első tulajdonságnál szereplő százalékoknak kell a százaléka, vagyis nem a 30%-ot
vagy a 25%-ot kell még kisebb összeadandókra bontani, hanem ezt vesszük 100%-nak.

Példa 2 :
Egy egyetemi vizsgán az A szakos hallgatók 60%-a, a B szakos hallgatók 75%-a, a C szakos
hallgatók 85%-a vizsgázik sikeresen. Az A szakos hallgatók az évfolyam 40%-át, a B szakos hall-
gatók az évfolyam 35%-át teszik ki. Mennyi a valószı́nűsége annak, hogy egy véletlenszerűen
kiválasztott hallgatónak nem sikerült a vizsgája?

Az első tulajdonság, hogy milyen szakos, a második tulajdonság, hogy átmegy vagy nem.

hallgatók
↙ ↓ ↘
A szak B szak C szak
40% 35% 25%
↙ ↘ ↙ ↘ ↙ ↘
siker bukás siker bukás siker bukás
60% 40% 75% 25% 85% 15%

p = 0,4 · 0,4 + 0,35 · 0,25 + 0,25 · 0,15 = 0,285♦

178
10. fejezet

Példa 3 :
Három gép gyárt csavarokat. Az első gép 1%, a második 2%, a harmadik 3% selejtet produkál.
Az első gép az össztermék 50%-át, a második a 30%-át, a harmadik a 20%-át állı́tja elő. Az
össztermékből véletlenszerűen választva egyet, mennyi a valószı́nűsége, hogy az selejtes?

Az első tulajdonság, hogy melyik gép, a második tulajdonság, hogy selejt vagy nem.

csavarok
↙ ↓ ↘
1. gép 2. gép 3. gép
50% 30% 20%
↙ ↘ ↙ ↘ ↙ ↘
selejt jó selejt jó selejt jó
1% 99% 2% 98% 3% 97%

p = 0,5 · 0,01 + 0,3 · 0,02 + 0,2 · 0,03 = 0,017♦

10.6. A Bayes-tétel
A Bayes-tétel nagyon sokban hasonlı́t a teljes valószı́nűség tételére. Itt is egy adott mennyiségű
elemet csoportokra bontunk, majd mindegyiket újabb csoportokra. Választunk egy elemet. A
Bayes-tételnél mindig tudjuk, hogy a kiválasztott elem rendelkezik a második tulajdonsággal,
ı́gy a kérdés mindig az, hogy mekkora a valószı́nűsége, hogy az első tulajdonság szerinti egyik
csoportból való.
Az ágrajz elkészı́tése most is erősen javasolt. A végeredmény egy tört lesz. A nevezője mindig
a teljes valószı́nűség tétele szerint kiszámolt szám. A számláló pedig ennek az összegnek az az
összetevője, amelyikről a feladat szól.

Példa 1 :
Egy gyárban három gép gyártja a csavarokat. A termékek 25%-át az A gép, 35%-át a B gép,
40%-át a C gép gyártja. Az A gép 5%-ban, a B gép 4%-ban, a C gép pedig 2%-ban termel
selejtet. Ha egy találomra kiválasztott csavar selejtes, mennyi a valószı́nűsége, hogy azt az A
gép gyártotta?

Az első tulajdonság, hogy melyik gép, a második tulajdonság, hogy selejt vagy nem.

csavarok
↙ ↓ ↘
A gép B gép C gép
25% 35% 40%
↙ ↘ ↙ ↘ ↙ ↘
selejt jó selejt jó selejt jó
5% 95% 4% 96% 2% 98%

0,25 · 0,05 25
p= = ≈ 0,3623
0,25 · 0,05 + 0,35 · 0,04 + 0,4 · 0,02 69

A nevező a teljes valószı́nűség tételéhez hasonlóan jön ki. Selejtet választottunk, tehát ezt kell

179
10. fejezet

összeszámolni, pontosan ugyanúgy, mint a 10.5. fejezetben.


A számláló most 0,25 · 0,05, hiszen a választott termék az A gépről van, tehát az összegnek ez
az összetevője kell, mert ez vonatkozik az A gépre.♦

Példa 2 :
Egy műhelyben három műszakban gyártanak azonos terméket. Egy napon az összes gyártott
termékből az első műszakban 40%, a második és harmadik műszakban 30−30% készült. Az első
műszakban 2%, a másodikban 3%, a harmadikban 5% hibás áru készült. A három műszakban
elkészült teljes mennyiségből véletlenszerűen kiválasztunk egy darabot és megvizsgáljuk. A
termék hibás. Mennyi a valószı́nűsége, hogy a második műszakban gyártották ezt a terméket?

Az első tulajdonság, hogy melyik műszak, a második tulajdonság, hogy hibás vagy jó.

műszakok
↙ ↓ ↘
1. műszak 2. műszak 3. műszak
40% 30% 30%
↙ ↘ ↙ ↘ ↙ ↘
hibás jó hibás jó hibás jó
1% 98% 3% 97% 5% 95%

0,3 · 0,03
p= = 0,28125
0,4 · 0,02 + 0,3 · 0,03 + 0,3 · 0,05

A számláló most 0,3 · 0,03, hiszen a választott termék a második műszakban készült.♦

Példa 3 :
Labdarúgó edzésen jártunk. Tudjuk, hogy a résztvevő húsz játékos közül a csatárok (öt fő)
0,9 valószı́nűséggel, a középpályások (hét fő) 0,8, a védők (hat fő) 0,75, a kapusok (két fő)
0,7 valószı́nűséggel lövik be a büntetőt. Látunk egy játékost, aki kihagyja a büntetőjét. Mi a
valószı́nűsége, hogy ő csatár?

Az első tulajdonság, hogy milyen játékos, a második tulajdonság, hogy berúgja vagy nem.
Most nem százalékokat tudunk, hanem létszámokat, ı́gy ki tudjuk számolni a valószı́nűségeket.
A húsz játékosból például 0,25 valószı́nűséggel választunk csatárt.

játékosok
↙ ↓ ↓ ↘
csatár középp. védő kapus
0,25 0,35 0,3 0,1
↙ ↘ ↙ ↘ ↙ ↘ ↙ ↘
berúgja kihagyja berúgja kihagyja berúgja kihagyja berúgja kihagyja
0,9 0,1 0,8 0,2 0,75 0,25 0,7 0,3

0,25 · 0,1
p= = 0,125♦
0,25 · 0,1 + 0,35 · 0,2 + 0,3 · 0,25 + 0,1 · 0,3

Példa 4 :
Egy rendezvényre készülődve poharakat tesznek ki egy asztalra. A gyárban két gépsoron készül-
nek a poharak, amelyek külsőre mind egyformák. Az első gépsoron gyártott poharak 10%-a selej-
tes. A második gépsoron készült poharak 4%-a selejtes. Az összes pohár 60%-át az első gépsoron,

180
10. fejezet

40%-át a második gépsoron gyártják. Az elkészült poharakat összekeverik, majd véletlenszerűen


kiválasztanak egyet és azt tapasztalják, hogy selejtes. Mekkora annak a valószı́nűsége, hogy ez
a pohár az első gépsoron készült?

Az első tulajdonság, hogy melyik gépsor, a második tulajdonság, hogy selejt vagy jó.

poharak
↙ ↘
1. gépsor 2. gépsor
60% 40%
↙ ↘ ↙ ↘
selejt jó selejt jó
10% 90% 4% 96%

0,6 · 0,1 15
p= = ≈ 0,7895♦
0,6 · 0,1 + 0,4 · 0,04 19

Példa 5 :
Egy csomagolóüzembe két termelő szállı́t tojásokat: az összes tojás 60%-a származik az A,
40%-a a B termelőtől. Az A termelő árujának 60%-a első osztályú, 40%-a másodosztályú, a B
termelő árujának 30%-a első osztályú és 70%-a másodosztályú. Az összes beszállı́tott tojás közül
véletlenszerűen kiválasztunk egyet, és azt első osztályúnak találjuk. Mekkora a valószı́nűsége,
hogy az A termelő árujából való a kiválasztott tojás?

Az első tulajdonság, hogy melyik termelőtől érkezik, a második tulajdonság, hogy első- vagy
másodosztályú.

termelő
↙ ↘
A termelő B termelő
60% 40%
↙ ↘ ↙ ↘
I. osztályú II. osztályú I. osztályú II. osztályú
60% 40% 30% 70%

0,6 · 0,6
p= = 0,75♦
0,6 · 0,6 + 0,4 · 0,3

10.7. Az ,,összes-rossz” módszer


Az ,,összes-rossz” módszerről alaposan volt szó a 9.5. fejezetben. Valószı́nűségszámı́tásban is
alkalmazható azzal az egy különbséggel, hogy itt az összes eset valószı́nűségéből, ami 1, kell
kivonni a rossz lehetőségek valószı́nűségét.

P (A) = 1 − P A

Alkalmazása ugyanakkor javasolt, mint kombinatorikában.

181
10. fejezet

Példa 1 :
Egy pakli magyar kártyából húzunk öt lapot. Mennyi a valószı́nűsége, hogy lesz köztük makk?

Hány makk lehet?


h i
0
|{z} 1| 2 {z
3 4 5}

Jóval több eset van, amikor a feladatot direkt szeretnénk megoldani, ezért inkább letagadjuk,
mert a tagadásból származó esetek száma egy, vagyis nincs benne makk.
P (lesz makk) = 1 − P (nincs makk)
A feladat maga hipergeometrikus eloszlás.

32 lap
↙ ↘
8 makk 24 többi
↓ ↓
0 5
  
8 24
0 5
1 −   ≈ 0,7889♦
32
5

Példa 2 :
Egyszerre feldobunk tı́z dobókockát. Mekkora a valószı́nűsége, hogy lesz köztük hatos?

Hány hatos lehet?


h i
0
|{z} 1| 2 3 {z . . . 10}

Komplementerrel számolva: P (lesz hatos) = 1 − P (nincs hatos)


A feladat a klasszikus képlettel oldható meg.
Kedvező esetek száma: mind a tı́z dobás egytől ötig bármi lehet, 510 lehetőség.
Összes eset száma: mind a tı́z dobás egytől hatig bármi lehet, 610 lehetőség.

510
p=1− ≈ 0,8385♦
610

Példa 3 :
Egyszerre feldobunk tı́z pénzérmét. Mekkora a valószı́nűsége, hogy lesz köztük fej?

Hány fej lehet?


h i
0
|{z} 1| 2 3 {z . . . 10}

Komplementerrel számolva: P (lesz fej) = 1 − P (nincs fej)


A feladat a klasszikus képlettel oldható meg.
Kedvező esetek száma: mind a tı́z érme ı́rás, ez egy lehetőség.

182
10. fejezet

Összes eset száma: mind a tı́z érme lehet fej vagy ı́rás, 210 lehetőség.
1
p=1− ≈ 0,999♦
210

Példa 4 :
Anna 0,7 valószı́nűséggel ı́r matekból ötös dolgozatot. A tanév során 11 dolgozatot ı́rtak. Mennyi
a valószı́nűsége, hogy legalább két ötöst ı́r?

Hány ötöst ı́rhat?


h i
0|{z}1 2| 3 4 {z . . . 11}

P (legalább két ötös) = 1 − P (legfeljebb egy ötös)


A feladat maga binomiális eloszlásra vezet.
Kı́sérletek száma: n = 11
A esemény az, hogy ötöst ı́r.
p = 0,7 ⇒ q = 0,3
Bekövetkezések száma: k = 0; 1
    
11 0 11 11 1 10
1− · 0,7 · 0,3 + · 0,7 · 0,3 ≈ 0,999953♦
0 1

Példa 5 :
Az A osztályban 26-an vannak, a B osztályban 27-en. Alakı́tunk egy hétfős csapatot. Mennyi
a valószı́nűsége, hogy lesznek benne a B osztályból?

Hány B osztályos lehet?


h i
0
|{z} 1| 2 3{z . . . 7}

P (lesz B-s) = 1 − P (nincs B-s)


A feladat hipergeometrikus eloszlásra vezet.

53 diák
↙ ↘
26 A-s 27 B-s
↓ ↓
7 0
  
26 27
7 0
p=1−   ≈ 0,9957♦
53
7

183
10. fejezet

Példa 6 :
Egy közvélemény-kutató intézet felméréséből kiderült, hogy a felnőttek 4%-a szı́ntévesztő.
Véletlenszerűen kiválasztunk nyolc felnőttet abból a népességből, melyre ez a felmérés vonat-
kozott. Mekkora a valószı́nűsége, hogy közöttük legalább két személy szı́ntévesztő?

Hány szı́ntévesztő lehet?


h i
0|{z}1 2| 3 4{z . . . 8}

P (legalább két szı́ntévesztő) = 1 − P (legfeljebb egy szı́ntévesztő)


A feladat binomiális eloszlásra vezet.
Kı́sérletek száma: n = 8
A esemény az, hogy szı́ntévesztő.
p = 0,04 ⇒ q = 0,96
Bekövetkezések száma: k = 0; 1
    
8 0 8 8 1 7
1− · 0,04 · 0,96 + · 0,04 · 0,96 ≈ 0,0381♦
0 1

Példa 7 :
Egy gyártósoron nyolc gép dolgozik. A gépek mindegyike, egymástól függetlenül 0,05 valószı́nű-
séggel túlmelegszik a reggeli bekapcsoláskor. Ha a munkanap kezdetén három vagy több gép
túlmelegszik, akkor az egész gyártósor leáll. Igazoljuk, hogy (négy tizedes jegyre kerekı́tve)
0,0058 annak a valószı́nűsége, hogy a gépek túlmelegedése miatt a gyártósoron leáll a termelés
a munkanap kezdetekor.

Hány gép melegedhet túl?


h i
0| {z
1 2} 3| 4 {z. . . 8}

P (legalább három túlmelegszik) = 1 − P (legfeljebb kettő melegszik túl)


A feladat binomiális eloszlásra vezet.
Kı́sérletek száma: n = 8
A esemény az, hogy túlmelegszik.
p = 0,05 ⇒ q = 0,95
Bekövetkezések száma: k = 0; 1; 2
      
8 0 8 8 1 7 8 2 6
1− · 0,05 · 0,95 + · 0,05 · 0,95 + · 0,05 · 0,95 ≈ 0,0058♦
0 1 2

Példa 8 :
Egy rejtvénypályázaton három dı́jat sorsolnak ki a helyes megfejtést beküldők között (egy meg-
fejtő legfeljebb egy dı́jat kaphat). 74 jó megfejtés érkezett be összesen, ezek közül 22 Miskolcról.
Mi a valószı́nűsége, hogy lesz miskolci nyertes?

Hány miskolci lehet?


h i
0
|{z} 1| {z
2 3}

184
10. fejezet

P (lesz miskolci) = 1 − P (nincs miskolci)


A feladat hipergeometrikus eloszlásra vezet.

74 megfejtés
↙ ↘
22 miskolci 52 többi
↓ ↓
0 3
  
22 52
0 3
1−   ≈ 0,6591♦
74
3

10.8. A várható érték


A valószı́nűségi változó várható értéke úgy számolható ki, hogy az összes lehetséges értéket
megszorozzuk a saját valószı́nűségével, majd ezeket a szorzatokat összeadjuk.
Könnyű felismerni, ha ezt kell csinálni, mert általában vagy szó szerint ı́gy van benne a feladat-
ban, vagy valamilyen bevételre kı́váncsi a feladat. Gyakori még a sorsjegyes feladat bevételéből
származó összegre utaló kérdés is.

Példa 1 :
Mennyi a kockadobás várható értéke?
1
A kocka minden lapjára valószı́nűséggel esik, ezért minden lehetséges értéket ezzel kell meg-
6
szorozni.
1 1 1 1 1 1
E= · 1 + · 2 + · 3 + · 4 + · 5 + · 6 = 3,5♦
6 6 6 6 6 6

Példa 2 :
Egy sorsjátékon egy darab egymillió forintos, tı́z darab ötvenezer forintos és száz darab ötezer
forintos nyeremény van. A többivel nem lehet nyerni. A játékhoz negyvenezer darab sorsjegyet
adnak ki. Mennyi legyen a jegy ára, hogy a nyeremény várható értéke a jegy árának felével
egyezzen meg?
1
Az egymillió forintos sorsjegy kihúzásának valószı́nűsége:
40 000
10
Az ötvenezer forintos sorsjegy kihúzásának valószı́nűsége:
40 000
100
Az ötezer forintos sorsjegy kihúzásának valószı́nűsége:
40 000
1 10 100
E = 1 000 000 · + 50 000 · + 5000 · = 50
40 000 40 000 40 000

100 F t-ért kell árulni a jegyet.♦

185
10. fejezet

Példa 3 :
Egy iskolakirándulás során négy busz szállı́tja a diákokat. A négy buszban 40, 33, 25 illetve
50 diák utazik. Véletlenszerűen kiválasztunk egy diákot, legyen X az ő buszában utazó összes
tanuló száma. A négy buszsofőr közül szintén kiválasztunk egyet véletlenszerűen, legyen Y az
ő buszán utazó diákok száma. Számoljuk ki X és Y várható értékét.
40
Annak valószı́nűsége, hogy az első buszból választunk diákot:
148
33
Annak valószı́nűsége, hogy a második buszból választunk diákot:
148
25
Annak valószı́nűsége, hogy a harmadik buszból választunk diákot:
148
50
Annak valószı́nűsége, hogy a negyedik buszból választunk diákot:
148
40 33 25 50
E (X) = 40 · + 33 · + 25 · + 50 · ≈ 39,28
148 148 148 148
1
Mind a négy sofőrt valószı́nűséggel lehet választani.
4
1 1 1 1
E (Y ) = 40 · + 33 · + 25 · + 50 · = 37♦
4 4 4 4

Példa 4 :
Albert és Béla feldob egy-egy kockát. Albert annyi forintot kap Bélától, amennyi a két kockán
lévő pontok különbségének négyzete. Béla pedig annyi forintot kap Alberttől, amennyi a két
kockán lévő pontok összege. Melyiküknek kedvez a játék?

Legyen X a két kockán lévő pontok különbségének négyzete. 10.2. fejezet alapján egy táblázatba
foglaljuk, hogy milyen lehetőségek vannak a két kockán lévő pontok eltérésére.

Ekkor X lehetséges értékei a táblázatban szereplő számok négyzetei: {0; 1; 4; 9; 16; 25} Ezek a
következő valószı́nűségekkel rendelkeznek:

X 0
1 4 16 25 36
6
10 8 6 4 2
p
36
36 36 36 36 36
6 10 8 6 4 2 35
E (X) = 0 · +1· +4· +9· + 16 · + 25 · = ≈ 5,83
36 36 36 36 36 36 6

186
10. fejezet

Legyen Y a két kockán lévő pontok összege. Y lehetséges értékei: {2; 3; 4; 4; 6; 7; 8; 9; 10; 11; 12}.

Y 2 3 4 5 6 7 8 9 10 11 12
1 2 3 4 5 6 5 4 3 2 1
p
36 36 36 36 36 36 36 36 36 36 36
1 2 3 4 5 6 5 4 3 2 1
E (Y ) = 2 · +3· +4· +5· +6· +7· +8· +9· +10· +11· +12· =7
36 36 36 36 36 36 36 36 36 36 36

Béla jár hosszútávon jobban, mert nagyobb a várható értéke.♦

Példa 5 :
Egy iskola alapı́tványi bálján a korábban szokásos tombolahúzás helyett egy egyszerű lottó-
húzást szerveznek. A szelvényt vásárolóknak az első tı́z pozitı́v egész szám közül kell ötöt
megjelölniük. Húzáskor öt számot sorsolnak ki (az egyszer már kihúzott számokat nem te-
szik vissza). Egy lottószelvény 200 F t-ba kerül. Egy telitalálatos szelvénnyel 5000 F t értékű,
egy négytalálatos szelvénnyel 1000 F t értékű, az alapı́tvány által vásárolt könyvutalványt lehet
nyerni. Négynél kevesebb találatot elérő szelvénnyel nem lehet nyerni semmit. Ha a húzás előtt
240 szelvényt adtak el, akkor mekkora az alapı́tvány lottóhúzásból származó hasznának várható
értéke?

Az alapı́tvány bevétele 240 · 200 = 48 000 F t.

10 szám 10 szám
↙ ↘ ↙ ↘
5 kihúzott 5 többi 5 kihúzott 5 többi
↓ ↓ ↓ ↓
5 0 4 1
A telitalálatos
   szelvény valószı́nűsége: A négytalálatos
   szelvény valószı́nűsége:
5 5 5 5
5 0 4 1
  = 0,0039 ≈ 0,004   = 0,0992 ≈ 0,099
10 10
5 5
E(telitalálatos) = 240 · 0,004 = 0,96 E(négytalálatos)240 · 0,099 = 23,76

Az alapı́tvány kiadása 0,96 · 5000 + 23,76 · 1000 = 28 560 F t.


A várható haszon ı́gy 19 440 F t.♦

187
11. fejezet

11. Sorozatok
11.1. A páratlan tagú számtani sorozatok
Tegyük fel, hogy egy számtani sorozatban meg van adva páratlan sok tag összege. Előfordulhat,
hogy praktikusabb ekkor úgy felı́rni a tagokat, hogy kiindulunk a középső tagból, ezt elnevezzük
a-nak (index nélkül), és a többi, erre szimmetrikusan elhelyezkedő tagot d-k segı́tségével állı́tjuk
elő. Ha ezt a taktikát alkalmazzuk, akkor kijön a középső tag, mert összeadáskor kiesnek a
differenciák.

Példa 1 :
Egy számtani sorozat első hét elemének összege 210. A második és a harmadik tag összege 30.
Mekkora a sorozat hatodik tagja?

A középső tag legyen a. Ekkor a hét tagot ı́gy ı́rjuk fel:

a − 3d a − 2d a−d a a+d a + 2d a + 3d

a − 3d + a − 2d + a − d + a + a + d + a + 2d + a + 3d = 210
7a = 210
a = 30
Most már tudjuk a középső (negyedik) tagot, ı́gy a második és harmadik tag összegénél csak a
d az ismeretlen.

30 − 2d + 30 − d = 210
d = 10
A hatodik tag 50.♦

Példa 2 :
Egy számtani sorozatban az ötödik, a nyolcadik, a tizenegyedik, a tizennegyedik és a tizenhe-
tedik tag összege 360. A tizenegyedik és tizenhetedik tag különbsége −24. Mekkora a sorozat
századik tagja?

A középső (tizenegyedik) tag legyen a. Ekkor az öt tag:

a − 6d a − 3d a a + 3d a + 6d

a − 6d + a − 3d + a + a + 3d + a + 6d = 360
5a = 360
a = 72
Visszahelyettesı́tve a másik összefüggésbe:

72 − (72 + 6d) = −24


d = 4
a100 = a11 + 89d = 72 + 89 · 4 = 428
Nem szükséges tehát, hogy a sorozat tagjai közvetlenül szomszédosak legyenek. Az is elég, ha
a köztük lévő távolság ugyanakkora.♦

188
11. fejezet

Példa 3 :
Egy egész számokból álló számtani sorozat első öt tagjának összege 65, szorzata 129 168. Melyik
ez a sorozat?

A középső tag legyen a.

a − 2d a−d a a+d a + 2d

a − 2d + a − d + a + a + d + a + 2d = 65
5a = 65
a = 13
Behelyettesı́tjük a másik összefüggésbe:

(13 − 2d)(13 − d) · 13 · (13 + d)(13 + 2d) = 129168


(13 − 2d)(13 − d)(13 + d)(13 + 2d) = 9936
Ha párosı́tjuk a zárójeleket, akkor két nevezetes azonosságot kapunk.

(169 − d2 ) (169 − 4d2 ) = 9936


28 561 − 845d2 + 4d4 = 9936
4d4 − 845d2 + 18 625 = 0
Legyen e = d2

4e2 − 845e + 18 625 = 0


e1 = 186,25
e2 = 25
Visszahelyettesı́tve:

d2 = 186,25 ⇒ d ≈ ±13,65 nem egész szám


d2 = 25 ⇒ d = ±5
Ez a sorozat: 3; 8; 13; 18; 23; . . . vagy 23; 18; 13; 8; 3; . . .

11.2. A páratlan tagú mértani sorozatok


Most azt tegyük fel, hogy egy mértani sorozatban meg van adva páratlan sok tag szorzata.
Az előző gondolatot fogjuk most is alkalmazni. A középső tag a, a többi, erre szimmetrikusan
elhelyezkedő tagot q-k segı́tségével állı́tjuk elő. Most is kijön a középső tag, mert szorzáskor
kiesnek a hányadosok.

Példa 1 :
Egy mértani sorozat első három elemének szorzata 8000. A második és a harmadik tag összege
100. Mekkora a sorozat negyedik tagja?

A középső tag legyen a. Ekkor a három tagot ı́gy ı́rjuk fel:


a
a aq
q

189
11. fejezet

a
· a · aq = 8000
q
a3 = 8000
a = 20
Behelyettesı́tve a másik egyenletbe:

20 + 20q = 100
q = 4
A negyedik tag 320.♦

Példa 2 :
Egy mértani sorozat ötödik, nyolcadik, tizenegyedik, tizennegyedik és tizenhetedik tagjának
szorzata 7 962 624. Mekkora a sorozat hányadosa, ha az ötödik és tizenhetedik tag összege 102?

A középső tag ismét a. A többi tag:


a a
a aq 3 aq 6
q6 q3

a a
· · a · aq 3 · aq 6 = 7 962 624
q6 q3
a5 = 7 962 624
a = 24
Ezt beı́rva a másik egyenletbe:
24
+ 24q 6 = 102
q6
24 + 24q 12 = 102q 6
24q 12 − 102q 6 + 24 = 0
Ekkor legyen r = q 6

24r2 − 102r + 24 = 0
r1 = 4
1
r2 =
4
Visszahelyettesı́tve:

q6 = 4 ⇒ q = ± 6 4
r
1 1
q6 = ⇒q=±6
4 4
Az ellenőrzés során kijön, hogy mind a négy érték lehetséges.♦

11.3. Számtani sorozat → mértani sorozat


Amikor egy számtani sorozat tagjaiból csinálunk számok hozzáadásával mértani sorozatot, vagy
egy számtani sorozat bizonyos elemei egy mértani sorozatot alkotnak, akkor a számtani jelöléseit
kell használni (d), de a mértani definı́cióját kell felı́rni.

190
11. fejezet

Példa 1 :
Egy számtani sorozat első három tagjának összege 24. A három taghoz rendre hozzáadunk
1-et, 2-t és 35-öt, ı́gy egy mértani sorozat három szomszédos tagját kapjuk. Mekkora a mértani
sorozat hányadosa?

Számtani sorozat: a − d; a; a + d

a − d + a + a + d = 24
a = 8
Mértani sorozat: 8 − d + 1; 8 + 2; 8 + d + 35, vagyis 9 − d; 10; 43 + d
Mértani sorozat definı́ciója alapján:

102 = (9 − d)(43 + d)
100 = 387 + 9d − 43d − d2
d2 + 34d − 287 = 0
d1 = 7
d2 = −41
Ha d = 7, akkor a számtani sorozat: 1; 8; 15; . . . ⇒ a mértani: 2; 10; 50; . . ., vagyis q = 5.
Ha d = −41, akkor a számtani sorozat: 49; 8; −33; . . . ⇒ a mértani: 50; 10; 2; . . ., vagyis
1
q = .♦
5
Példa 2 :
Egy növekedő számtani sorozat első három tagjának összege 60. Az első tagot 64-gyel növelve, a
másik két tagot változatlanul hagyva, egy mértani sorozat első három tagjához jutunk. Mennyi
a két sorozat első három tagja?

Számtani sorozat: a − d; a; a + d

a − d + a + a + d = 60
a = 20
Mértani sorozat: 20 − d + 64; 20; 20 + d, vagyis 84 − d; 10; 20 + d
Mértani sorozat definı́ciója alapján:

202 = (84 − d)(20 + d)


400 = 1680 + 84d − 20d − d2
2
d − 64d − 1280 = 0
d1 = 80
d2 = −16
A d = −16 hamis megoldás, mert növekvő a sorozat.
Ha d = 80, akkor a számtani sorozat első három tagja: −60; 20; 100; . . . ⇒ a mértani sorozat
első három tagja: 4; 20; 100.♦

Példa 3 :
Egy számtani sorozat hetedik eleme 15. A sorozat első, negyedik és tizenharmadik eleme egy
mértani sorozat három egymást követő eleme. Adjuk meg a mértani sorozat hányadosát.

A számtani sorozatban ekkor a1 = 15 − 6d; a4 = 15 − 3d; a7 = 15 + 6d


Mértani sorozat definı́ciója alapján:

191
11. fejezet

(15 − 3d)2 = (15 − 6d)(15 + 6d)


225 − 90d + 9d2 = 225 − 36d2
45d2 − 90d = 0
d1 = 0
d2 = 2
Ha d = 0, akkor a számtani sorozat: a1 = a4 = a7 = 15 ⇒ q = 1.
Ha d = 2, akkor a számtani sorozat: a1 = 3; a4 = 9; a7 = 27 ⇒ q = 3. ♦

11.4. Mértani sorozat → számtani sorozat


Amikor egy mértani sorozat tagjaiból csinálunk számok hozzáadásával számtani sorozatot, vagy
egy mértani sorozat bizonyos elemei egy számtani sorozatot alkotnak, akkor a mértani jelöléseit
kell használni (q), de a számtani definı́cióját kell felı́rni.

Példa 1 :
Egy mértani sorozat első három tagjának szorzata 216. Ha a harmadik számot hárommal
csökkentjük, egy számtani sorozat első három tagját kapjuk. Határozzuk meg a számtani soro-
zatot.
a
Mértani sorozat: ; a; aq
q
a
· a · aq = 216
q
a = 6
6
Számtani sorozat: ; 6; 6q − 3
q
Számtani sorozat definı́ciója alapján:
6
2·6 = + 6q − 3
q
12q = 6 + 6q 2 − 3q
2
6q − 15q + 6 = 0
q1 = 2
1
q2 =
2
Ha q = 2, akkor a mértani sorozat első három tagja: 3; 6; 12 ⇒ a számtani sorozat első három
tagja: 3; 6; 9.
1
Ha q = , akkor a mértani sorozat első három tagja: 12; 6; 3 ⇒ a számtani sorozat első három
2
tagja: 12; 6; 0.♦

Példa 2 :
Egy pozitı́v tagokból álló mértani sorozat első három tagjának összege 26. Ha az első tag-
hoz egyet, a másodikhoz hatot, a harmadikhoz hármat adunk, akkor ebben a sorrendben egy
számtani sorozat első három tagját kapjuk. Adjuk meg ennek a számtani sorozatnak az első
három tagját.
a
Mértani sorozat: ; a; aq
q

192
11. fejezet

Most nem a tagok szorzata van megadva, hogy kiessenek a q-k, hanem az összegük, ı́gy tudjuk,
hogy:
a
+ a + aq = 26
q

De a hozzáadásokkal fel tudjuk ı́rni, hogy:


 
a
+ 1 + (a + 6) + (aq + 3) = 36
q

Mivel összesen 10-et adtunk hozzá a 26-hoz. Ezek egy számtani sorozat szomszédos tagjai.
 
a
+ 1 + (a + 6) + (aq + 3) = 36
q | {z } | {z }
| {z } b b+d
b−d

A korábbi feladatok tapasztalatai alapján:

b − d + b + b + d = 36
b = 12

Ebből visszakapjuk, hogy a = 6.


Az új adat ismeretében újraı́rjuk az egyenletünket:
6
+ 6 + 6q = 26
q
6 + 6q 2
= 20q
6q 2 − 20q + 6
= 0
q1
= 3
1
q2 =
3
Ha q = 3, akkor a mértani sorozat első három tagja: 2; 6; 18 ⇒ a számtani sorozat első három
tagja: 3; 12; 21.
1
Ha q = , akkor a mértani sorozat első három tagja: 18; 6; 2 ⇒ a számtani sorozat első három
3
tagja: 19; 12; 5.♦

11.5. Sorozatok egymásba fűzése


Ha egy olyan sorozat van, hogy a szomszédos tagjai váltakozva egyszer nőnek, egyszer csökken-
nek külön-külön ugyanannyi szorosára, akkor két mértani sorozat van egymásba fésülve, két
különböző q-val. A sorozat tagjait kettesével kell figyelembe venni, mert ı́gy a két különböző
hányadosból adódik egy közös. Ez általában akkor kell, ha an a kérdés. Ha az egész sorozatot
nézzük, akkor minden második tag alkot egy-egy mértani sorozatot. Ezekkel külön kell számolni
és az eredményeket összeadni. Ez általában akkor kell, ha Sn a kérdés.

Példa 1 :
Egy erdő faállománya 3500 m3 . A mindenkori faállomány évente 3%-kal gyarapszik, és kétéven-
ként a meglévő faállomány 2%-át kivágják. Mennyi fa lesz az erdőben húsz év múlva?

193
11. fejezet

Kezdeti érték: 3500 m3


·1,03
1 év után: 3500 · 1,03 ·1,032 · 0,98
·1,03 · 0,98
2 év után: 3500 · 1,032 · 0,98
·1,03
3 év után: 3500 · 1,033 · 0,98 ·1,032 · 0,98
·1,03 · 0,98
4 év után: 3500 · 1,034 · 0,982

Kicsit nehezebb átlátni a változást, ha minden évet egyesével nézünk. De azt vesszük észre, hogy
ha csak minden második évben nézzük a változást, akkor lesz egy közös hányados: 1,032 · 0,98.
10
Húsz év alatt tı́z ilyen ,,kétéves ciklus” megy le, ı́gy 3500 · (1,032 · 0,98) ≈ 5165 m3 fa lesz.♦

Példa 2 :
András edzőtáborban készül egy úszóversenyre, húsz napon át. Azt tervezte, hogy naponta
10 000 m-t úszik. De az első napon a tervezettnél 10%-kal többet, a második napon pedig az
előző napinál 10%-kal kevesebbet teljesı́tett. A harmadik napon ismét 10%-kal növelte az előző
napi adagját, a negyedik napon 10%-kal kevesebbet edzett, mint az előző napon, és ı́gy foly-
tatta, páratlan sorszámú napon 10%-kal többet, pároson 10%-kal kevesebbet teljesı́tett, mint a
megelőző napon. Hány métert úszott le a huszadik napon? Mennyit úszott összesen a húsz nap
alatt?

Tervezett: 10 000 m
·1,1
1. nap: 10 000 · 1,1 ·1,1 · 0,9 = 0,99
·0,9
2. nap: 10 000 · 1,1 · 0,9
·1,1
3. nap: 10 000 · 1,12 · 0,9 ·1,1 · 0,9 = 0,99
·0,9
4. nap: 10 000 · 1,12 · 0,92
·1,1
5. nap: 10 000 · 1,13 · 0,92

Újra egyszerűbb átlátni, ha minden második évben nézzük a változást. A közös hányados: 0,99.
A huszadik napig tı́z ilyen ,,kétnapos ciklus” történik, ı́gy a huszadik napon 10 000 · 0,9910 =
= 9043,82 métert úszik.
Ahhoz viszont, hogy megkapjuk a húsz nap alatt mennyit úszott le összesen, szét kell szednünk
a páratlan és a páros napokat, mert ezek külön-külön mértani sorozatot alkotnak, de egyben
nem, mert mindig más a q értéke. (A tervezett 10 000 m nem tagja egyik sorozatnak sem.)
Páratlan napok: Páros napok:
Első tag = 1. nap: 11 000 Első tag = 2. nap: 9900
q = 0,99 q = 0,99
0,9910 − 1 0,9910 − 1
S10 = 11 000 · ≈ 105 180 S10 = 9900 · ≈ 94 662
0,99 − 1 0,99 − 1
Összesen 199 842 métert úszott le.♦

194
11. fejezet

11.6. Határérték számolás leosztással


A módszer lényege, hogy meg kell keresni a törtben található legmagasabb fokú tagot, és min-
dent leosztani ezzel. Egy emeletes törtet fogunk ı́gy kapni, ahol a legtöbb tag nullához kon-
vergál. Alkalmazható polinomokból álló, gyökös kifejezéseket tartalmazó vagy exponenciális
kifejezéseket tartalmazó törteknél is. A módszer alapos ismerete nagyon fontos, mert összetet-
tebb feladatok részeként is előfordulhat.Azt az összefüggést használjuk a legtöbbször, hogy:
konstans konstans
lim = 0 és lim =∞
n→∞ ∞-be tartó n→∞ 0-ba tartó

Példa 1 :
6n − 3n3
Határozzuk meg lim határértéket.
n→∞ 2n + 7 + 9n4

6 3
6n − 3n3 3
− 0−0
lim = lim n n = = 0♦
n→∞ 2n + 7 + 9n 4 n→∞ 2 7 0+0+9
+ +9
n3 n4

Példa 2 :
3n3 − n2 + 8n + 23
Határozzuk meg lim határértéket.
n→∞ 4n3 + 7n − 15

1 8 23
3n3 − n2 + 8n + 23 3− + 2 + 3
lim = lim n n n = 3 − 0 + 0 + 0 = 3♦
n→∞ 3
4n + 7n − 15 n→∞ 7 15 4+0−0 4
4+ 2 − 3
n n

Példa 3 :
3n4 + 2
Határozzuk meg lim határértéket.
n→∞ 4n3 + 2n − 1

2
4
3n + 2 3+
3+0
lim = lim = n4
= ∞♦
n→∞ 4n3 + 2n − 1 n→∞ 4 2 1 0+0−0
+ 3− 4
n n n

Példa 4 :
6n − 3n4
Határozzuk meg lim határértéket.
n→∞ 2n + 7 + 9n4

6
6n − 3n4 −3 0−3 1
lim = lim n3 = = − ♦
n→∞ 2n + 7 + 9n4 n→∞ 2 7 0+0+9 3
+ +9
n3 n4

Példa 5 : √
6 + n2
Határozzuk meg lim = határértéket.
n→∞ 3n

195
11. fejezet

Itt tudni kell azt, hogy ha elosztunk egy négyzetgyökös kifejezést n-nel, akkor azt a négyzetgyök
alatt már n2 -tel kell elosztani, hiszen ha bármit beviszünk a négyzetgyök alá, akár egy osztást
is,
√ akkor azt négyzetre kell emelni. √
6 + n2 esetén csak a legnagyobb fokú tag számı́t, ezért n2 is elsőfokúnak minősül, akárcsak
3n, ı́gy n a legmagasabb fokú tag.
r
√ 6 √
+1
6 + n2 n2 0+1 1
lim = lim = = ♦
n→∞ 3n n→∞ 3 3 3

Példa 6 : √
3
n − n2
Határozzuk meg lim √ = határértéket.
n→∞ n + n2 + 1

3 2 √
n2 = n 3 , mı́g n2 = n1 , ı́gy n a legmagasabb fokú tag.
r
√ 1
3 2 1− 3
n− n n 1−0 1
lim √ = lim r = = ♦
n→∞ n + 2
n +1 n→∞ 1 1+1 2
1+ 1+ 2
n
Az exponenciális kifejezéseknél az a nagyobb, amelyiknek a hatványalapja nagyobb, ha az alap
nagyobb, mint egy. Ezekben a feladatokban azt a nevezetes határértéket használjuk, hogy:

lim q n = 0, ha |q| < 1


n→∞

Példa 7 :
4n
Határozzuk meg lim = határértéket.
n→∞ 3 · 4n + 2

4n 1 1 1
lim = lim  n = = ♦
n→∞ 3 · 4n + 2 n→∞ 1 3+0 3
3+2·
4

Példa 8 :
3n+2 · 2n−3 − 7 · 5n
Határozzuk meg lim = határértéket.
n→∞ 5 · 6n−1 + 9

1
3n+2 · 2n−3 − 7 · 5n 9 · 3n · · 2n − 7 · 5n
lim = lim 8 =
n→∞ 5·6 n−1 +9 n→∞ 1 n
5· ·6 +9
6  n
9 n 9 5n 9 5 9
· 6 − 7 · 5n −7· n −7· −7·0
8 6 27
= lim 8 = lim 8 6 = lim  n = 8 = ♦
n→∞ 5 n n→∞ 5 9 n→∞ 5 1 5 20
·6 +9 + n +9· +9·0
6 6 6 6 6 6

196
11. fejezet

11.7. Határérték számolás e-vel


Onnan lehet felismerni az e-vel megoldandó határérték
 n n számı́tási feladatokat, hogy a számláló-
ban, a nevezőben és a kitevőben is van n, azaz ,, ” kinézetű. A számlálót és a nevezőt is
n
egy közös, n-t tartalmazó tényezővel kell elosztani. Ezután a számláló és a nevező külön-külön
e egy hatványához fog konvergálni. A következő nevezetes határértékekre épülnek a feladatok:
 n  nα
k k k
lim 1+ =e és lim 1+ α = ek
n→∞ n n→∞ n

Példa 1 :  n
n+3
Határozzuk meg lim határértéket.
n→∞ n+4
 n  n
3
n
 3 3
n 1+ 1+ lim 1 +
e3

n+3 n n n→∞ n 1
lim = lim  = lim  n = n = 4 = ♦
 
4

n→∞ n+4 n→∞ n→∞ 4 4 e e
1+ 1+ lim 1 +
n n n→∞ n

Példa 2 :  n
n−4
Határozzuk meg lim határértéket.
n→∞ n+2
 n  n
4
n
 −4 −4
n 1− 1+ lim 1 +
e−4

n−4 n n n→∞ n
lim = lim  = lim = = = e−6 ♦
 
n n
2
   
n→∞ n+2 n→∞ n→∞ 2 2 e2
1+ 1+ lim 1 +
n n n→∞ n

Példa 3 :  n
3n + 7
Határozzuk meg lim határértéket.
n→∞ 3n + 2

Itt 3n-nel kell leosztani, utána a számlálókban egy törtet kell létrehozni, hogy látszódjon e
kitevője.
 n  7 n

7
n 7
 n 1+ 1+ lim 1 + 3 7
3n + 7 3n 3n n→∞ n e3 5
lim = lim   = lim  n = n = 2 = e 3 ♦
 
n→∞ 3n + 2 n→∞ 2 n→∞ 2
 2
e3
1+ 1+ lim 1 + 3
3n 3n n→∞ n

Példa 4 :  n+4
2n − 1
Határozzuk meg lim határértéket.
n→∞ 2n + 5

197
11. fejezet

 n+4 n 
 4
2n − 1 2n − 1 2n − 1
lim = lim · =
n→∞ 2n + 5 n→∞ 2n + 5 2n + 5
 n 

1 n
 
1 4
 1 1 4

1− 1− 1 − 1−
2n  2n  2n 2n 
= lim  · = lim  n ·  =
  
n→∞ 5  5 
n→∞ 5 5 
1+ 1+ 1+ 1 +
2n 2n 2n 2n
n
−1
 
1 4
 
lim 1 + 2 1− 1 4
e− 2

n→∞ n 2n 1−0
= · lim  = 5 · = e−3 ♦
 
5 n 5  1 + 0

n→∞ e2
lim 1 + 2 1+
n→∞ n 2n

Példa 5 : !n2
r
n2 + 1
Határozzuk meg lim határértéket.
n→∞ n2 − 1

  21
s n2
 !n2
1
1 n 2
n2 + 1  n2 + 1  1 + n2  
 2
lim  = lim = lim   =
n→∞ n2 − 1 n→∞ n2 − 1 n→∞ 
 −1  
1+ 2
n
 n2  12
1
 1 + n2   1  12
e
= lim   = = e♦
 
2
n  e−1

n→∞ 

−1
1+ 2
n

Példa 6 :  8n
1 1
Határozzuk meg lim 1− határértéket.
n→∞ 2 n

 8n  n !8
1 1 1 −1 1 1
lim 1− = lim 1+ = (e−1)8 = 8 ♦
n→∞ 2 n n→∞ 2 n 2 2e

Példa 7 :  −2n
n−1
Határozzuk meg lim határértéket.
n→∞ n+1

−1 n −2 
  

n−1
−2n 1+ e−1
−2
lim = lim 
 n  
= = e4 ♦
n→∞ n+1 n→∞ 1   e1
1+
n

198
11. fejezet

11.8. Határérték számolás rendőrelvvel


A rendőrelv alkalmazásakor az alapvető gondolatmenet az, hogy találni kell a sorozathoz egy
majoráns (,,nagyobb”) és egy minoráns (,,kisebb”) sorozatot, amik közé be lehet szorı́tani a mi
sorozatunkat. Ha a majoráns és a minoráns sorozat egyaránt ugyanahhoz a számhoz konvergál,
akkor a keresett sorozat is.

Ha an ≤ bn ≤ cn és lim an = lim cn = A, akkor lim bn = A


n→∞ n→∞ n→∞

A rendőrelv √ kétféle kinézetű sorozatokhoz alkalmas.


Az egyik a ,, n 4n + 3n ” tı́pusú sorozat. Minoránst úgy gyártunk, hogy a legnagyobb tagot meg-
hagyjuk, a többit kitöröljük. Majoránst pedig úgy készı́tünk, hogy a legnagyobb tagra átı́rjuk
az összes többit. Polinomoknál a legnagyobb mindig a legnagyobb fokú tagot jelenti, exponen-
ciális kifejezéseknél pedig a legnagyobb alapú tagot. Ha egyszerre szerepel benne hatvány is és
exponenciális kifejezés is, akkor mindig az exponenciális a nagyobb, ha az alap nagyobb, mint
egy, mert ha n tart végtelenbe, akkor előbb-utóbb ,,legyőzi” a hatványt. Ezeknél a feladatoknál
felhasználjuk a következő nevezetes határértékeket:

n

n
lim c = 1 és lim n=1
n→∞ n→∞

sin n
A másik a ,, ” tı́pusú sorozat. Egyes trigonometrikus kifejezések a korlátosságuk miatt egy
n
számmal mindig minorálhatók és majorálhatók.

Példa 1 : √
Határozzuk meg lim n n2 + 2n + 4 határértéket.
n→∞


n

n

n
n2 ≤ n2 + 2n + 4 ≤ n2 + n2 + n2


n √ 2
Minoráns: n2 = n n → 12 = 1
√ √
n
√ √ 2
Majoráns: n n2 + n2 + n2 = 3n2 = n 3 · n n → 1 · 12 = 1

n
lim n2 + 2n + 4 = 1♦
n→∞

Példa 2 : √
Határozzuk meg lim n 4n + 3n + n4 határértéket.
n→∞


n

n

4n ≤ 4n + 3n + n4 ≤ n 4n + 4n + 4n


n
Minoráns: 4n = 4 → 4
√ √ √ √
Majoráns: n 4n + 4n + 4n = n 3 · 4n = n 3 · n 4n → 1 · 4 = 4

n
lim 4n + 3n + n4 = 4♦
n→∞

199
11. fejezet

Példa 3 : √
Határozzuk meg lim n 32n + 3n + 1 határértéket.
n→∞


n

n

n
32n ≤ 32n + 3n + 1 ≤ 32n + 32n + 32n

n
Minoráns: 32n = 9 → 9
√ √
n
√ √ n
Majoráns: n 32n + 32n + 32n = 3 · 32n = n 3 · 32n → 1 · 9 = 9

n
lim 32n + 3n + 1 = 9♦
n→∞

Példa 4 :
sin n
Határozzuk meg lim határértéket.
n→∞ n

−1 sin n 1
≤ ≤
n n n
−1
Minoráns: →0
n
1
Majoráns: → 0
n
sin n
lim = 0♦
n→∞ n

Példa 5 :
cos n + 2
Határozzuk meg lim határértéket.
n→∞ 4n

1 cos n + 2 3
n
≤ n
≤ n
4 4 4
1
Minoráns: →0
4n
3
Majoráns: n → 0
4
cos n + 2
lim = 0♦
n→∞ 4n

Példa 6 :
cos2 n + cos n
Határozzuk meg lim határértéket.
n→∞ n2

−3 cos2 n + cos n 3
2
≤ 2
≤ 2
n n n
−3
Minoráns: →0
n2
3
Majoráns: 2 → 0
n
cos2 n + cos n
lim = 0♦
n→∞ n2

200
11. fejezet

11.9. Határérték számolás konjugálttal


√ √
A konjugált alkalmazása is könnyen felismerhető, mert általában a ,, n− m” kinézetű soroza-
tokhoz kell. Erre azért van szükség, mert két végtelenbe tartó sorozat összegéről tudjuk, hogy a
végtelenbe tart, de két végtelenbe tartó sorozat különbségét nem tudjuk azonnal megmondani.
Lehet, hogy az első tag jóval nagyobb, mint a második, ezért hiába vonunk ki belőle egy másik
végtelenbe tartó sorozatot, attól még a különbség a végtelenbe tart továbbra is. De az is lehet,
hogy a két végtelenbe tartó sorozat annyira közel halad egymáshoz, hogy a különbség egy valós
számhoz tart.
A konjugáltas módszer ezt a problémát kiküszöböli. A sorozatot megszorozzuk és el is osztjuk
a saját konjugáltjával, majd a leosztásos módszerrel kiszámoljuk a határértéket.(Az a + b alakú
kifejezés konjugáltja az a − b, az a − b-nek konjugáltja az a + b.)

Példa 1 : √ √
Határozzuk meg lim n + 1 − n − 1 határértéket.
n→∞

√ √  √ √ 
√ √ n+1− n−1 n+1+ n−1
lim n + 1 − n − 1 = lim √ √ =
n→∞ n→∞ n+1+ n−1
n + 1 − (n − 1) 2 2
= lim √ √ = lim √ √ = = 0♦
n→∞ n + 1 + n − 1 n→∞ n + 1 + n − 1 ∞+∞

Példa 2 : √ √
Határozzuk meg lim 7n − 10 − 7n + 10 határértéket.
n→∞

√ √  √ √ 
√ √ 7n − 10 −
7n + 10 7n − 10 + 7n + 10
lim 7n − 10 − 7n + 10 = lim √ √ =
n→∞ n→∞ 7n − 10 + 7n + 10
7n − 10 − (7n + 10) −20 −20
= lim √ √ = lim √ √ = = 0♦
n→∞ 7n − 10 + 7n + 10 n→∞ 7n − 10 + 7n + 10 ∞+∞

Példa 3 : √ √
Határozzuk meg lim 4n2 + 4n + 1 − 4n2 − 3n − 2 határértéket.
n→∞

√ √
lim 4n2 + 4n + 1 − 4n2 − 3n − 2 =
n→∞
√ √  √ √ 
4n2 + 4n + 1 − 4n2 − 3n − 2 4n2 + 4n + 1 + 4n2 − 3n − 2
= lim √ √ =
n→∞ 4n2 + 4n + 1 + 4n2 − 3n − 2
4n2 + 4n + 1 − (4n2 − 3n − 2) 7n + 3
= lim √ √ = lim √ √ =
n→∞ 4n + 4n + 1 + 4n − 3n − 2 n→∞ 4n + 4n + 1 + 4n2 − 3n − 2
2 2 2

3
7+ 7+0 7
= lim r n
r =√ √ = ♦
n→∞ 4 1 3 2 4+ 4 4
4+ + 2 + 4− − 2
n n n n

201
11. fejezet

Példa 4 : √ √
Határozzuk meg lim 5n3 − 5n3 − 9 határértéket.
n→∞

√√  √ √ 
√ √ 5n3 −
5n 3−9 5n 3+ 5n 3−9
lim 5n3 − 5n3 − 9 = lim √ √ =
n→∞ n→∞ 5n3 + 5n3 − 9
5n3 − (5n3 − 9) 9 9
= lim √ √ = lim √ √ = = 0♦
n→∞ 5n3 + 5n3 − 9 n→∞ 5n3 + 5n3 − 9 ∞+∞

11.10. A mértani sor


Adott egy számsorozat. A tagokat az elejétől fogva összeadjuk és definiálunk részösszegeket:

S1 = a1 ; S2 = a1 + a2 ; S3 = a1 + a2 + a3 ; . . .

A sor n-dik tagja a sorozat első n elemének összegével egyenlő.

S n = a1 + a2 + a3 + . . . + an

Ha a sorozat tagjait végtelenig adjuk össze, akkor végtelen (numerikus) sorról beszélünk.
Vagyis az ilyen részösszegekből álló számsorozat határértékét vizsgáljuk, ami megegyezik ennek
a végtelen hosszú összegnek az eredményével.

lim Sn = a1 + a2 + . . . + an + . . .
n→∞

Ezt rövidı́tjük a következő jelöléssel:



X
an
n=1

Számunkra a legfontosabb sorok azok, melyekben az an = q n . Ezek a végtelen mértani sorok.


1
Legyen például a1 = 1 és q = .
2
1 1 1
1 + + + + ...
|{z} 2 4 8
S
| 1 {z }
S2
| {z }
S3
| {z }
S4

3 7 15
Ebben az esetben S1 = 1; S2 = ; S3 = ; S4 = ; . . . és lim Sn = 2
2 4 8 n→∞
Azt mondjuk, hogy a sor konvergens, ha az eredmény egy valós szám. A mértani sorok konver-
genciájának az a feltétele, hogy |q| < 1 és ekkor a mértani sor összege:

X 1
q n = első tag ·
n=1
1−q

202
11. fejezet

Példa 1 :
∞  n
X 2
Mennyi ?
n=1
3

2
A sor konvergens, mert < 1.
3
∞  n
X 2 2 1
= · = 2♦
3 3 2
n=1 1−
3

Példa 2 :
∞  n
X 1
Mennyi − ?
n=3
5

1
A sor konvergens, mert − < 1.
5
∞  n  3
X 1 1 1 1
− = − ·   =− ♦
5 5 1 150
n=3 1− −
5

Példa 3 :
2
Egy mértani sorozat első tagja 6, hányadosa . Ha a sorozat tagjait a végtelenségig összeadjuk,
3
mekkora az ı́gy kapott összeg?
8 16 32
A sorozat tagjai: 6; 4; ; ; ; ...
3 9 27
2
A sor konvergens, mert < 1.
3
∞  n−1
X 2 1
6· =6· = 18♦
3 2
n=1 1−
3

Példa 4 :
1
Egy mértani sorozat első tagja 5, hányadosa . Ha a sorozat tagjait a végtelenségig összeadjuk,
2
mekkora az ı́gy kapott összeg?
5 5 5 5
A sorozat tagjai: 5; ; ; ; ; ...
2 4 8 16
1
A sor konvergens, mert < 1.
2
∞  n−1
X 1 1
5· =5· = 10♦
2 1
n=1 1−
2

203
11. fejezet

11.11. A mértani sor geometria feladatokban


A mértani sorok beazonosı́tása egy szöveges feladatban nem mindig egyszerű, ezért külön fi-
gyelmet fordı́tunk most erre.
Ha egy szöveges feladat úgy szól, hogy mekkora a valahányadik elem, akkor a sorozat an tagja
kell. Ha úgy szól, hogy mennyi az első valahány elem összege, akkor Sn kell. Ha pedig úgy szól,
hogy mennyi az összes elem összege, akkor a végtelen mértani sor összege kell. Ez az elem lehet
egy sı́kidom területe vagy akár a határoló vonalainak hossza. Emlékezni kell a geometriánál ta-
nult hasonlóságra is. Ha egydimenziós mértékről van szó (vonalhossz, kerület. . . ), akkor minden
a hasonlóság arányával változik, vagyis az elemek által alkotott mértani sor hányadosa egyenlő
a hasonlóság arányával. (λ = q) Ha kétdimenziós mértékről szól a feladat (terület), akkor min-
den a hasonlóság arányának négyzetével változik, vagyis az elemek által alkotott mértani sor
hányadosa egyenlő a hasonlóság arányának négyzetével. (λ2 = q)

Példa 1 :
Egy 10 cm oldalú négyzetbe érintőkört ı́runk, majd ebbe négyzetet, amibe ismét érintőkört. Ezt
ı́gy folytatva mekkora lesz a hatodik négyzet oldala? Ha az eljárást a végtelenségig folytatjuk,
mennyi lesz a körök területének összege?

A sorozat első tagja az első négyzet oldala: 10 cm. Meg kell tudnunk a második négyzet oldalát,
hogy lássuk mekkora a q.

Pitagorasz-tétel az ABC△-ben:

52 + 52 = BC

2

BC = 5 2

Innen tudjuk, hogy:



2
q=
2

A hatodik négyzet oldala:


√ !5
2
10 · ≈ 1,7678 cm
2

204
11. fejezet

A köröknél az első tag az első kör területe:

52 π = 25π

2
A hasonlóság aránya most is , de mivel területekről van szó, azok λ2 -szeresére változnak
2
mindig, ı́gy most
1
q = λ2 =
2

A körök területének összege:


1
25π · = 50π cm2 ♦
1
1−
2

Példa 2 :
Az A1 C0 C1 derékszögű háromszögben az A1 csúcsnál 30◦ -os szög van, az A1 C0 befogó hossza
1, az A1 C1 átfogó felezőpontja A2 . Az A2 C1 szakasz ,,fölé” az A1 C0 C1 háromszöghöz hasonló
A2 C1 C2 derékszögű háromszöget rajzoljuk az ábra szerint. Az A2 C2 átfogó felezőpontja A3 . Az
A3 C2 szakasz ,,fölé” az A2 C1 C2 háromszöghöz hasonló A3 C2 C3 derékszögű háromszöget rajzol-
juk. Ez az eljárás tovább folytatható. Számı́tsuk ki az ı́gy nyerhető végtelen sok derékszögű
háromszög területének összegét (az összeg első tagja az A1 C0 C1 háromszög területe). Igazoljuk,
hogy a C0 C1 C2 . . . Cn törött vonal hossza minden pozitı́v egész n-re kisebb, mint 1,4.

205
11. fejezet

A1 C0 C1 △-ben:
x1
tg 30◦ =
1

3
x1 =
3
A háromszögek területeiből álló sor első tagja:

3 √

TA1 C0 C1 △ = 3 = 3
2 6

Az összes háromszög hasonló, vagyis ha megtudjuk milyen hosszú A2 C1 , meglesz a λ.


A1 C0 C1 △-ben:
1
cos 30◦ =
A 1 C1
2
A1 C1 = √
3
1
A2 C1 = √
3
Ekkor:
A 2 C1 1
λ= =√
A 1 C0 3

A háromszögek területeit kell összeadni, ezért:


1
q = λ2 =
3

A háromszögek területeinek összege:


√ √
3 1 3
· =
6 1 4
1−
3
A törött vonal hosszánál a sor első tagja:

3
x1 =
3

Hányadosa pedig:
1
q=λ= √
3

A végtelen hosszú törött vonal hossza:



1 1 1 3 1
√ · =√ ·√ =√ ≈ 1,366 < 1,4♦
3 1 − √1 3 3−1 3−1
3

206
11. fejezet

Példa 3 :
Írjunk egy r sugarú körbe rendre egymáshoz csatlakozó húrokat a következő módon: a k-dik
húr hossza legyen egyenlő a körbe ı́rható 2k+1 -oldalú szabályos sokszög oldalhosszúságával, ahol
k = 1; 2; 3; . . .. Bizonyı́tsuk be, hogy a kapott végtelen törött vonal hossza kisebb, mint 4r.

Számoljuk ki az első két tagot.


Ha k = 1, akkor négyzetről van szó.
Ha k = 2, akkor nyolcszögről van szó.


Egy r sugarú körbe ı́rt négyzet oldala r 2 az ABO△-ben felı́rt Pitagorasz-tétel miatt.
A nyolcszög oldalának kiszámolásához felı́rjuk BCD△-ben a koszinusz-tételt:
√ 2
r 2 = b2 + b2√− 2 · b · b · cos 135◦
2r2 = 2b2 + 2b2
2r2
b2 = √
2+ 2
r
2
b = r √
2+ 2
Törött vonalak hosszának összegéről van szó, vagyis a hasonlóság aránya egyenlő a mértani sor
hányadosával.
r
2
√ √
2+ 2 2 1 1
q=λ= √ =p √ ·√ =p √ ≈ 0,5411
2 2+ 2 2 2+ 2
A vonalak hosszának összege:
√ 1
r 2· = 3,08r < 4r♦
1 − 0,5411

207
11. fejezet

Példa 4 :
Egy 1 m oldalú négyzetbe egy második négyzetet rajzoltunk úgy, hogy a belső négyzet min-
den csúcsa illeszkedjen a külső négyzet egy-egy oldalára. A belső és a külső négyzet oldalainak
aránya 5 : 7. A belső négyzetbe egy újabb, harmadik négyzetet rajzolunk úgy, hogy a harmadik
és a második négyzet oldalainak aránya is 5 : 7. Ezt az eljárást aztán gondolatban végtelen
sokszor megismételjük. Milyen arányban osztja két részre a belső négyzet csúcsa a külső négyzet
oldalát? Az arány pontos értékét adjuk meg. Mekkora lesz a kapott négyzetek kerületeinek az
összege, ha a kiindulási négyzet kerülete is tagja az összegnek?

5
Ha a külső négyzet oldala 1, akkor a belsőé .
7
Pitagorasz-tétel az ECH△-ben:
 2
2 2 5
(1 − x) + x =
7
2
49x − 49x + 12 = 0
4
x1 =
7
3
x2 =
7
Az osztópontok 3 : 4 arányban osztják ketté a négyzet oldalát.
Minden négyzet hasonló, és mivel megint kerületek összege kell, a kerületek által alkotott
mértani sor hányadosa egyenlő a hasonlóság arányával:
5
q=λ=
7

Az első négyzet kerülete 4.


A végtelen sok négyzet kerülete:
1
4· = 14 m♦
5
1−
7
208
11. fejezet

Példa 5 : √
Az ABCDEF szabályos hatszögben a rövidebb átló hossza 5 2. Számı́tsuk ki a hatszög
területének pontos értékét.
Az ABCDEF hatszög oldalfelező pontjai által meghatározott szabályos hatszög területét jelölje
t1 , a t1 területű hatszög oldalfelező pontjai által meghatározott szabályos hatszög területét t2 ,
és ı́gy tovább, képezve ezzel a {tn } sorozatot. Számı́tsuk ki a lim (t1 +t2 +. . .+tn ) határértéket.
n→∞
(Pontos értékekkel számoljunk.)


A szövegből tudjuk, hogy AC = 5 2. Az eredeti hatszög oldala legyen x.
ABC△-ben felı́rjuk a koszinusz-tételt:
√ 2
5 2 = x2 + x2 − 2 · x · x · cos 120◦
3x2
50 = r √ √
50 5 2 5 6
x = = √ =
3 3 3

√ !2
5 6 √
· 3
3 √
Thatszög = 6 · Tszabályos △ = 6 · = 25 3
4

Ki kell számolni a második hatszög oldalát, vagyis F1 F2 -t, hogy megtudjuk a hasonlóság
arányát. √
5 2
F1 F2 középvonal az ABC△-ben, ı́gy F1 F2 = .
2

5 2 √ √
2 = 5 2 3 3
λ= √ · √ =
5 6 2 5 6 2
3

209
11. fejezet

Mindegyik szabályos hatszög egymáshoz hasonló, és mivel területek összege kell, a területek
által alkotott mértani sor hányadosa egyenlő a hasonlóság arányának négyzetével.
3
q = λ2 =
4

A mértani sor első tagja:



√ 3 75 3
t1 = 25 3 · =
4 4

lim (t1 + t2 + . . . + tn ) jelenti a végtelen sok hatszög területének összegét, ami:


n→∞

75 3 1 √
· = 75 3♦
4 3
1−
4

Példa 6 :
Egy egységnyi oldalú szabályos háromszög oldalait osszuk három egyenlő részre. Az óramutató
járásával ellentétesen minden csúcstól számı́tott második osztópontot kössünk össze. Az ı́gy ka-
pott háromszögekkel ezt az eljárást folyamatosan megismételjük. Mekkora a hetedik háromszög
kerülete? Mekkora az ı́gy keletkezett háromszögek kerületeinek összege? Mekkora a kilencedik
háromszög területe? Mekkora az ı́gy keletkezett háromszögek területeinek összege?

Az első háromszög oldala 1, kerülete 3. Ez a sor első tagja.


Pitagorasz-tétel az ABD△-ben:
 2  2
1 2 2
+ AB =
3 3

1 3
AB = √ =
3 3

210
11. fejezet

Az összes háromszög szabályos és hasonló. A kerületek mindig λ-szorosára változnak, ı́gy a sor
hányadosa is ennyi.

3
q=λ=
3

A hetedik háromszög kerülete:


√ !6
3 1
3· =
3 9

A végtelen sok háromszög kerülete:


1 9
3· √ = √
3 3− 3
1−
3
Az első háromszög területe:
√ √
12 · 3 3
Tszabályos △ = =
4 4

A területek mindig λ2 -szeresére változnak, ı́gy most


1
q = λ2 =
3

A kilencedik háromszög területe:


√  8 √
3 1 3
· =
4 3 26244

A végtelen sok háromszög területe:


√ √
3 1 3 3
· = ♦
4 1 8
1−
3

11.12. Sorozatok monotonitása


Sorozatok monotonitásának meghatározásánál a következő a javaslat. Írjuk fel az első néhány
tagot, hogy lássuk, hogyan viselkedik a sorozat. Erre azért van szükség, mert előfordulhat,
hogy egy sorozat nem rögtön az első tagtól fogva monoton. Lehet, hogy az első néhány tag
,,rakoncátlankodik”. Ezek után összehasonlı́tjuk a sorozat n-dik tagját a sorozat (n + 1)-dik
tagjával, és ebből megállapı́tható a sorozat monotonitása.

211
11. fejezet

Példa 1 :
2n + 3
Milyen a monotonitása az an = sorozatnak?
n+4
7 9 11
A sorozat első néhány tagja: 1; ; ; ;...
6 7 8
Látjuk, hogy az elejétől növekszik, ezért felı́rjuk, hogy an ≤ an+1 és megmutatjuk, hogy ez igaz.

2n + 3 2(n + 1) + 3

n+4 (n + 1) + 4
2n + 3 2n + 5

n+4 n+5
(2n + 3)(n + 5) (n + 4)(2n + 5)

(n + 4)(n + 5) (n + 4)(n + 5)
(2n + 3)(n + 5) ≤ (n + 4)(2n + 5)
2n2 + 13n + 15 ≤ 2n2 + 13n + 20
15 ≤ 20
A sorozat valóban monoton nő.♦

Példa 2 :
3n − 2
Milyen a monotonitása az an = sorozatnak?
n+2
1 7 10
A sorozat első néhány tagja: ; 1; ; ;...
3 5 6
Erről is látjuk, hogy az első tagtól fogva növekszik.

3n − 2 3(n + 1) − 2

n+2 (n + 1) + 2
3n − 2 3n + 1

n+2 n+3
(3n − 2)(n + 3) (n + 2)(3n + 1)

(n + 2)(n + 3) (n + 2)(n + 3)
(3n − 2)(n + 3) ≤ (n + 2)(3n + 1)
3n2 + 7n − 6 ≤ 3n2 + 7n + 2
−6 ≤ 2
A sorozat monoton nő.♦

Példa 3 :
2n
Milyen a monotonitása az an = 2 sorozatnak?
n
8 32 64
Az első tagok: 2; 1; ; 1; ; ;...
9 25 36
Ez a sorozat is növekszik, de csak a harmadik tagtól fogva.

212
11. fejezet

2n 2n+1

n2 (n + 1)2
2n 2n · 2

n2 n2 + 2n + 1
1 2

n2 2
n + 2n + 1
n2 + 2n + 1 ≤ 2n2
0 ≤ n2 − 2n − 1
Ez egy másodfokú egyenlőtlenség, ı́gy az 5.3. fejezet alapján a megoldás ]−∞; −0,41]∪[2,41; ∞[.
Mivel n csak pozitı́v egész szám lehet, ezért ez a megoldás leszűkül n ≥ 3-ra. A levezetés tehát
kihozta, hogy a sorozat növekszik, de a harmadik tagtól, ahogyan azt az elején is láttuk.♦

Példa 4 :
Milyen a monotonitása az an = |n + 3| + |7 − n| sorozatnak?

Ehhez pontosan ugyanaz a gondolatmenet kell, mint a 2.6. fejezetben. Elkészı́tjük a táblázatot.
Ebben a feladatban viszont, mivel n pozitı́v egész szám, ezért az n ≤ −3 esettel nem kell fog-
lalkozni.

n ≤ −3 −3 < n < 7 7≤n


|n + 3| - n+3 n+3
|7 − n| - 7−n −(7 − n)
|n + 3| + |7 − n| - 10 2n − 4
A sorozat tagjai: 10; 10; 10; 10; 10; 10; 10; 12; 14; . . .
A sorozat monoton nő.♦

Példa 5 : !2
π  π 
Milyen a monotonitása az an = sin · n + cos ·n sorozatnak?
2 2

Senkinek sincs kedve ebbe ı́gy behelyettesı́teni számokat, ezért átalakı́tjuk addı́ciós tételt és
Pitagorasz-tételt alkalmazva.

!2
π  π 
an = sin · n + cos ·n=
2 2
π  π  π  π 
= sin2 · n + 2 sin · n cos · n + cos2 · n = 1 + sin(π · n)
2 2 2 2
Mivel a szinusz minden egész π-re nulla, ezért a sorozat a konstans 1.♦

Példa 6 :
Milyen a monotonitása az an = (−2)n + 2n sorozatnak?

Az első tagok: 0; 8; 0; 32; 0; 128; . . .


Páratlan n-re az érték 0, páros n-re pedig egy kettőhatvány.

213
11. fejezet

Mivel a sorozat tagonként egyszer nő, egyszer csökken a végtelenségig, nincs monotonitása.♦

Főleg faktorokat és exponenciális kifejezéseket tartalmazó sorozatoknál célszerű lehet, hogy az
an ≤ an+1 egyenlőtlenséget nem kivonással nullára, hanem osztással egyre rendezzük.

Példa 7 :
(n + 3)!
Milyen a monotonitása az an = sorozatnak?
3n
40 80 560 4480
Az első tagok: 8; ; ; ; ;...
3 3 9 27
an+1
A sorozat első néhány tagjából azt tapasztaljuk, hogy növekszik. Vizsgáljuk most meg 1 ≤
an
hányadost.

(n + 4)!
1 ≤ 3n+1
(n + 3)!
3n
(n + 4)(n + 3)! 3n
1 ≤ ·
3 · 3n (n + 3)!
n+4
1 ≤
3
−1 ≤ n

Végeredményünk alátámasztja kezdeti észrevételünket, vagyis a sorozat az első tagtól kezdve


folyamatosan monoton nő.

11.13. Sorozatok korlátossága


Érdemes a sorozat határértékét kiszámolni, mert ha konvergens, akkor biztosan korlátos is.
(A legkisebb felső korlát jelölése sup{an }, a legnagyobb alsó korlát jelölése inf{an }.)

Példa 1 :
n+3
Korlátos-e az an = sorozat? Ha igen, adjunk korlátokat.
n+2
lim an = 1, ezért biztosan korlátos.
n→∞
4 5 6 7 8
Az első tagok: ; ; ; ; ; . . .
3 4 5 6 7
Úgy tűnik, hogy a sorozat az elejétől fogva csökken.

214
11. fejezet

n+3 (n + 1) + 3

n+2 (n + 1) + 2
n+3 n+4

n+2 n+3
(n + 3)(n + 3) (n + 4)(n + 2)

(n + 2)(n + 3) (n + 3)(n + 2)
(n + 3)(n + 3) ≥ (n + 4)(n + 2)
n2 + 6n + 9 ≥ n2 + 6n + 8
9 ≥ 8

4
Ennek következtében az első tag a legnagyobb, ı́gy sup{an } = .
3
A legjobb alsó korlát pedig a határérték, mert az alá nem jut el a sorozat: inf{an } = 1.♦

Példa 2 :
n2 + 1
Korlátos-e az an = sorozat? Ha igen, adjunk korlátokat.
2n2 + 4
1
lim an = , ezért biztosan korlátos.
n→∞ 2
2 5 10 17 26
Az első tagok: ; ; ; ; ;...
6 12 22 36 54
Megint azt sejtjük, hogy a sorozat az elejétől fogva nő.

n2 + 1 (n + 1)2 + 1

2n2 + 4 2(n + 1)2 + 4
n2 + 1 n2 + 2n + 2

2n2 + 4 2n2 + 4n + 6
(n2 + 1)(2n2 + 4n + 6) (n2 + 2n + 2)(2n2 + 4)

(2n2 + 4)(2n2 + 4n + 6) (2n2 + 4n + 6)(2n2 + 4)
(n2 + 1)(2n2 + 4n + 6) ≤ (n2 + 2n + 2)(2n2 + 4)
2n4 + 4n3 + 8n2 + 4n + 6 ≤ 2n4 + 4n3 + 8n2 + 8n + 8
4n + 6 ≤ 8n + 8
1
− ≤ n
2

215
11. fejezet

1
Az első tag a legkisebb, ı́gy inf{an } = .
3
1
A legjobb felső korlát pedig a határérték: sup{an } = .♦
2
Példa 3 :  nπ 
Korlátos-e az an = tg sorozat? Ha igen, adjunk korlátokat.
3
√ √ √ √
Az első tagok: 3; − 3; 0; 3; − 3; 0; . . .
A tagok hármasával
√ ismétlődnek.

inf{an } = − 3; sup{an } = 3.♦

Példa 4 :
Korlátos-e az an = (−2)n sorozat? Ha igen, adjunk korlátokat.

Az első tagok: −2; 4; −8; 16; −32; 64; . . .


A sorozat nem korlátos, mert minden második tag egyre nő és minden második tag egyre
csökken .♦

Példa 5 :
n2 + 2
Korlátos-e az an = sorozat? Ha igen, adjunk korlátokat.
3n
lim an = ∞
n→∞
11 18 27 38
Az első tagok: 1; 1; ; ; ; ;...
9 12 15 18
n2 + 2 (n + 1)2 + 2

3n 3(n + 1)
n2 + 2 n2 + 2n + 3

3n 3n + 3
2
(n + 2)(3n + 3) 3n(n2 + 2n + 3)

3n(3n + 3) 3n(3n + 3)
(n2 + 2)(3n + 3) ≤ 3n(n2 + 2n + 3)
3n3 + 3n2 + 6n + 6 ≤ 3n3 + 6n2 + 9n
0 ≤ 3n2 + 3n − 6

A fenti másodfokú egyenlőtlenség megoldása a ]−∞; −2] ∪ [1; ∞[. Mivel n csak pozitı́v egész
szám lehet, ezért ez a megoldás leszűkül n ≥ 1-re. Vagyis a sorozat az első tagtól fogva monoton
nő, de ennek a sorozatnak csak alsó korlátja van: inf{an } = 1.♦

216
12. fejezet

12. Analı́zis
12.1. A lánc-szabály
A lánc-szabály az összetett függvények deriválására vonatkozó eljárás. A 2.7. fejezetben volt
már szó a függvények kompozı́ciójáról, de csak emlı́tés volt arról, hogy mi a belső és mi a külső
függvény.
Vegyünk egy összetett függvényt, pl. f (x) = sin (2x + 1). Tegyük fel, hogy be akarjuk az x
helyére helyettesı́teni a 10 számot. Ekkor előbb számoljuk ki, hogy mennyi 2 · 10 + 1 és utána
vesszük az eredmény szinuszát. Vagyis amit először csinálunk a számmal, az a belső függvény,
amit később, az a külső függvény.
Három függvényből álló összetett függvény esetén
√ definiálható a belső, a középső és a külső
függvény is. Legyen a függvényünk f (x) = log2 2x2 + 1. Most helyettesı́tsük be a 2-t.√Először
azt számoljuk ki mennyi 2 · 22 + 1, ez a belső függvény. Utána kiszámoljuk mennyi 9, ez a
középső függvény. Végül vesszük a 3 logaritmusát, ez a külső függvény.
Ha azt mondjuk, hogy a legkülső függvény mindig a legbaloldalibb, és ahogy haladunk jobb-
ra, mindig egyre beljebb haladunk a legbelső függvény felé, akkor majdnem igazunk van. A
hatványozás az egyetlen kivétel. Legyen most f (x) = sin x2 . Itt valóban előbb végezzük el
a négyzetre emelést, és utána vesszük az eredmény szinuszát. De f (x) = sin2 x-nél már más
a helyzet. Előbb számoljuk ki a behelyettesı́tett szám szinuszát, és utána emeljük négyzetre.
Szóval nagyon kell figyelni, mert a hatványozás megtévesztő lehet.
Erre azért van szükség, mert a lánc-szabálynál kı́vülről kell kezdeni a deriválást. A legkülső
függvényt deriváljuk úgy, hogy a belseje (akkor is, ha több függvényből áll a belseje) marad.
Ezután jön a következő külső függvény. Ezt is deriváljuk, és ezt is úgy, hogy a belseje meg-
marad. És ı́gy haladunk folyamatosan befelé, egészen addig, amı́g eljutunk a legbelső függvény
deriváltjáig. Az összes ilyen derivált tagot össze kell szorozni.

[f (g(x))]′ = f ′ (g(x)) · g ′ (x)

Példa 1 :
Deriváljuk az f (x) = sin (3x − 5) függvényt.

Belső: 3x − 5
Külső: sin ( )

f ′ (x) = cos(3x-5) · 3

külső deriváltja, belső megmarad belső deriváltja ♦

Példa 2 :
Deriváljuk az f (x) = tg (x2 ) függvényt.

Belső: x2
Külső: tg ( )

1
f ′ (x) = · 2x
cos2 (x2 )

külső deriváltja, belső megmarad belső deriváltja ♦

217
12. fejezet

Példa 3 :
Deriváljuk az f (x) = tg3 x függvényt.
Belső: tg ( )
Külső: ( )3

1
f ′ (x) = 3tg2 x ·
cos2 x

külső deriváltja, belső megmarad belső deriváltja ♦

Példa 4 :
Deriváljuk az f (x) = − ln cos (2x) függvényt.

Belső: 2x
Középső: cos ( )
Külső: − ln ( )

 
′ 1
f (x) = − · − sin(2x) · 2
cos(2x)

külső deriváltja, belseje megmarad középső deriváltja, belső marad belső deriváltja ♦

12.2. Függvények szélsőértéke


Ismeretes az a tény, hogy egy differenciálható f függvénynek szélsőértéke az x0 ∈ Df -ben
összefüggésben áll azzal, hogy f ′ (x0 ) = 0. Nem teljesen egyértelmű viszont, hogy a követ-
keztetés melyik irányba igaz. Vegyük példának az f (x) = x3 függvényt az x0 = 0 helyen.
A függvény deriváltjának helyettesı́tési értéke nulla, még sincsen szélsőértéke a függvénynek.
Ebből az egyszerű példából is látszik, hogy az implikáció melyik irányba igaz.

f függvénynek szélsőértéke van x0 helyen ⇒ f ′ (x0 ) = 0


Ahhoz, hogy ez az implikáció visszafelé is igaz legyen, kell további feltétel, másképp megfogal-
mazva, ellenőrizni kell a szélsőértékjelölteket. Erre kétféle megoldás létezik.
Ha a második derivált előjelét vizsgáljuk:

ˆ Nincs szélsőértéke, ha f ′′ (x0 ) = 0

ˆ Minimuma van, ha f ′′ (x0 ) > 0

ˆ Maximuma van, ha f ′′ (x0 ) < 0

Ha táblázatot készı́tünk:

ˆ Nincs szélsőértéke, ha f ′ az x0 helyen nem vált előjelet

ˆ Minimuma van, ha f ′ az x0 helyen negatı́vból pozitı́vba megy át

ˆ Maximuma van, ha f ′ az x0 helyen pozitı́vból negatı́vba megy át

218
12. fejezet

Ha a szélsőérték megállapı́tásán kı́vül más kérdés is van (pl. milyen intervallumon csökkenő
a függvény), akkor érdemes a táblázatos megoldást alkalmazni, mert ezzel a többi kérdésre
is válaszolhatunk. Ha csak a szélsőérték fajtájának megállapı́tása a kérdés, akkor gyorsabb a
második deriváltba helyettesı́tés.

Példa 1 :
Számoljuk ki az f (x) = x3 + 3x2 + 3x + 4 függvény szélsőértékeit. Milyen szélsőértékek ezek?

f ′ (x) = 3x2 + 6x + 3
3x2 + 6x + 3 = 0
x = −1

Ellenőrzés:

f ′′ (x) = 6x + 6
f ′′ (−1) = 0

f -nek nincs szélsőértéke itt.♦

Példa 2 :
1
Számoljuk ki az f (x) = x3 − 6x2 + 32x függvény szélsőértékeit. Milyen szélsőértékek ezek?
3

f ′ (x) = x2 − 12x + 32
x2 − 12x + 32 = 0
x1 = 4
x2 = 8

Ellenőrzés:

f ′′ (x) = 2x − 12
f ′′ (4) = −4 < 0
f ′′ (8) = 4 > 0

f -nek 4-ben maximuma, 8-ban pedig minimuma van.♦

Példa 3 :
Határozzuk meg, mely intervallumon monoton növekvő illetve monoton csökkenő az
f (x) = x4 − 4x3 + 4x2 + 1 függvény.

f ′ (x) = 4x3 − 12x2 + 8x


3 2
4x − 12x + 8x = 0
4x(x2 − 3x + 2) = 0
x1 = 0
x2 = 1
x3 = 2

Ebben a feladatban a táblázatot készı́tjük el, mert nem csupán a szélsőérték milyensége a kérdés.

219
12. fejezet

x<0 x=0 0<x<1 x=1 1<x<2 x=2 2<x


f (x) ↘ lok. min. ↗ lok. max. ↘ lok. min. ↗
f ′ (x) - 0 + 0 - 0 +

A függvény monoton csökken a ]−∞; 0[ ∪ ]1; 2[, monoton nő a ]0; 1[ ∪ ]2; ∞[ intervallumon.♦

Példa 4 :
ex
Határozzuk meg mely intervallumon monoton növekvő illetve monoton csökkenő az f (x) =
x
függvény.

ex · x − ex · 1
f ′ (x) =
x2
ex · x − ex · 1
= 0
x2
x
e (x − 1)
= 0
x2
x
e (x − 1) = 0
↙↘
ex = 0 x−1 = 0
nincs megoldás x = 1

x<0 0<x<1 x=1 1<x


f (x) ↘ ↘ lok. min. ↗
f ′ (x) - - 0 +

A függvény monoton csökken a ]−∞; 0[ ∪ ]0; 1[, illetve monoton nő az ]1; ∞[ intervallumon.♦

Amikor az első derivált sorában plusz- és mı́nuszjelekkel töltjük ki a táblázatot, akkor elegendő,
ha egyetlenegy számot kiválasztunk az adott intervallumból, behelyettesı́tjük az első deriváltba,
és kiszámoljuk milyen előjelű számot kapunk. Viszont, amikor az értelmezési tartomány nem
folytonos, mint a legutóbbi példában is, akkor nem szabad leegyszerűsı́teni, és azt mondani,
hogy csak egy intervallum létezik. Ketté kell szedni a szakadási helyeknél, és külön-külön be
kell helyettesı́teni egy-egy számot az intervallumokból. Ezért van dupla vonallal jelölve a x < 0
és 0 < x < 1 közötti határvonal.

12.3. Függvények görbülete


Hasonlóan az első derivált és a függvény szélsőértéke közötti kapcsolathoz, itt is ismeretes, hogy
egy kétszer differenciálható f függvénynek görbülete kapcsolatban áll a második deriváltjával.
Viszont itt is figyelni kell arra, hogy a következtetés melyik irányba helyes. Vegyünk most is
példának egy egyszerű függvényt, pl. az f (x) = x4 függvényt az x0 = 0 helyen. A függvény
második deriváltjának helyettesı́tési értéke nulla, még sincsen inflexiós pontja a függvénynek.
Látszik, hogy az implikáció csak az egyik irányba igaz.

f függvénynek inflexiós pontja van x0 helyen ⇒ f ′′ (x0 ) = 0

220
12. fejezet

Ahhoz, hogy ez az implikáció is igaz legyen visszafelé, kell további feltétel. Erre kétféle megoldás
létezik.
Ha a harmadik derivált előjelét vizsgáljuk:

ˆ Nincs inflexiós pont, ha f ′′′ (x0 ) = 0

ˆ Van inflexiós pont, ha f ′′′ (x0 ) ̸= 0

Ha táblázatot készı́tünk:

ˆ Nincs inflexiós pont, ha f ′′ az x0 helyen nem vált előjelet

ˆ Van inflexiós pont, ha f ′′ az x0 helyen negatı́vból pozitı́vba, vagy pozitı́vból negatı́vba


megy át

Hasonlóan a szélsőértékhez, ha csak az inflexiós pont bizonyı́tása a feladat, egyszerűbb a


harmadik deriváltat használni ellenőrzés céljából. Ha viszont más kérdés is van (pl. milyen
intervallumon konvex a függvény), akkor a táblázat a javasolt út.

Példa 1 :
Határozzuk meg az f (x) = x4 + 2x3 + 5x2 + 3 függvény inflexiós pontjait.

f ′ (x) = 4x3 + 6x2 + 10x


f ′′ (x) = 12x2 + 12x + 10
12x2 + 12x + 10 = 0

Ennek nincs megoldása, vagyis a függvénynek nincs inflexiós pontja.♦

Példa 2 :
Határozzuk meg az f (x) = xex függvény inflexiós pontjait.

f ′ (x)
= 1 · ex + xex = ex (x + 1)
f ′′ (x)
= ex (x + 1) + ex = ex (x + 2)
x
e (x + 2)
= 0
↙↘
ex = 0 nincs megoldás x = −2

Ellenőrzés:

f ′′′ (x) = ex (x + 2) + ex = ex (x + 3)
f (−2) = e−2 ̸= 0
′′′

Vagyis −2-ben inflexiós pontja van.♦

Példa 3 :
Mely intervallumon konvex az f (x) = −4x3 + 10x függvény?

f ′ (x) = −12x2 + 10
f ′′ (x) = −24x
−24x = 0
x = 0

221
12. fejezet

x<0 x=0 0<x


f (x) konvex infl. pont konkáv
f ′′ (x) + 0 -

A függvény konvex a ]−∞; 0[ intervallumon.♦

Példa 4 :
Mely intervallumon konvex illetve konkáv az f (x) = 2x4 + x3 − 12 függvény?

f ′ (x) = 8x3 + 3x2


f ′′ (x) = 24x2 + 6x
2
24x + 6x = 0
x1 = 0
1
x2 = −
4

1 1 1
x<− x=− − <x<0 x=0 0<x
4 4 4
f (x) konvex infl. pont konkáv infl. pont konvex

f ′′ (x) + 0 - 0 +
  
1 1
A függvény konvex a −∞; − ∪ ]0; ∞[ és konkáv a − ; 0 intervallumon.♦
4 4

12.4. A határozott integrál és a terület kapcsolata


A határozott integrál eredménye nem feltétlenül egyezik meg a függvénygörbe alatti területtel.
Három különböző eset állhat fent:

1. eset:
Ha az [a; b] intervallumon az f függvény teljes terjedelmével az x-tengely felett van, akkor a
terület valóban megegyezik az integrállal.

Zb
T = f (x) dx
a

222
12. fejezet

2. eset:
Ha az [a; b] intervallumon az f függvény teljes terjedelmével az x-tengely alatt van, akkor a
terület az integrál ellentettjével egyezik meg.

Zb
T =− f (x) dx
a

3. eset:
Ha az [a; b] intervallumon az f függvény az x-tengely felett és alatt is van, akkor fel kell darabol-
ni a zérushely(ek)nél, kiszámolni külön-külön az integrálokat, és összeadni. Ekkor természetesen
az x-tengely alatt lévő rész területe negatı́v, ı́gy annak vesszük az abszolút értékét az összeadás
előtt. A következő ábrán a második darab negatı́v és c ∈ [a; b].

Zc Zb
T = f (x) dx + f (x) dx
a c

A területkiszámolós feladatokat a függvény gyors felrajzolásával érdemes kezdeni. Ha nem megy


az ábrázolás, akkor a zérushelyeit kell meghatározni.
Ha van zérushelye, de az nem esik bele az [a; b] intervallumba, esetleg nincs is zérushelye, ak-
kor a függvény vagy az x-tengely felett, vagy az x-tengely alatt van teljesen. Hogy megtudjuk
melyik, az [a; b] intervallumból behelyettesı́tünk egy számot a függvénybe. Ha pozitı́v az eredmény,
felette van, ha negatı́v, akkor alatta.
Ha van zérushelye, és az bele is esik az [a; b] intervallumba, akkor minden részről kiválasztunk
egy-egy számot és azt is behelyettesı́tjük a függvénybe, hogy megkapjuk, hol helyezkedik el.Ezek

223
12. fejezet

után fogunk hozzá az integráláshoz.


Abban az esetben, ha úgy szól a kérdés, hogy mekkora területű sı́kidomot fog közre a függvény
és az x-tengely, akkor az integrálási határokat ki kell számolni, és ezek megegyeznek a függvény
zérushelyeivel.

Példa 1 :
Számı́tsuk ki az f (x) = x2 −4x+5 függvénygörbe és az x-tengely között fekvő sı́kidom területét
az [1; 4] intervallumon.

f (x) = x2 − 4x + 5 = (x − 2)2 + 1

A függvény hozzárendelési szabályából már tudjuk, hogy végig az x-tengely felett van.

Z4 4
x3

2 2
x − 4x + 5 dx = − 2x + 5x =
3 1
1
 3   3 
4 2 1 2 28 10
= −2·4 +5·4 − −2·1 +5·1 = − =6
3 3 3 3

A terület 6.♦

Példa 2 :
Számı́tsuk ki az f (x) = −x2 + 1 függvénygörbe és az x-tengely között fekvő sı́kidom területét
a [3; 4] intervallumon.

Erről a függvényről tudjuk, hogy végig az x-tengely alatt helyezkedik el a [3; 4] intervallumon.

Z4 4
x3
  3   3 
2 4 3 52 34
−x + 1 dx = − + x = − +4 − − +3 =− +6=−
3 3 3 3 3 3
3

34
A terület az integrál ellentettje .♦
3
Példa 3 :
Számı́tsuk ki az f (x) = −(x − 4)2 + 4 függvénygörbe és az x-tengely között fekvő sı́kidom
területét a [2; 7] intervallumon.

Ezt a függvényt sem nehéz ábrázolni, de most csináljuk a zérushellyel.

224
12. fejezet

−(x − 4)2 + 4 = 0
−x2 + 8x − 12 = 0
x1 = 2
x2 = 6

Választunk egy számot a 2 és a 6 között. Legyen a 3. Behelyettesı́tve a függvénybe, az eredmény


3, ami pozitı́v, vagyis biztos, hogy a [2; 6] intervallumon a függvény az x-tengely felett van.
Választunk egyet 6 és 7 között is. Legyen a 6,5. Ha ezt behelyettesı́tjük, az eredmény −2,25.
Ez azt jelenti, hogy a [6; 7] intervallumon a függvény az x-tengely alatt van.

−(x − 4)2 + 4 = −x2 + 8x − 12

Külön-külön kiszámoljuk az integrálokat.

Z6 6
x3

2 2
−x + 8x − 12 dx = − + 4x − 12x =
3 2
2
 3   3 
6 2 2 2 32
= − + 4 · 6 − 12 · 6 − − + 4 · 2 − 12 · 2 =
3 3 3

Z7  3 7
2 x 2
−x + 8x − 12 dx = − + 4x − 12x =
3 6
6
 3   3 
7 2 6 2 7
= − + 4 · 7 − 12 · 7 − − + 4 · 6 − 12 · 6 = −
3 3 3

Az integrálok előjele igazolja, hogy melyik részen hol helyezkedik el a függvény.


32 7
A terület + − = 13.♦
3 3

225
12. fejezet

Példa 4 :
Számı́tsuk ki az f (x) = x2 −5x+6 függvénygörbe és az x-tengely között fekvő sı́kidom területét
a [0; 4] intervallumon.

x2 − 5x + 6 = 0
x1 = 2
x2 = 3

A parabola alakjából tudjuk, hogy 2 előtt illetve 3 után az x-tengely felett van a görbe, 2 és 3
között pedig alatta.

Külön kiszámoljuk megint az integrálokat.

Z2 2
x3 5x2

2
x − 5x + 6 dx = − + 6x =
3 2 0
0
23 5 · 22
  3
5 · 02
 
0 14
= − +6·2 − − +6·0 =
3 2 3 2 3

Z3 3
x3 5x2

2
x − 5x + 6 dx = − + 6x =
3 2 2
2
 3
5 · 32
  3
5 · 22

3 2 9 14 1
= − +6·3 − − +6·2 = − =−
3 2 3 2 2 3 6

Z4 4
x3 5x2

2
x − 5x + 6 dx = − + 6x =
3 2 3
3
 3
5 · 42
  3
5 · 32

4 3 16 9 5
= − +6·4 − − +6·3 = − =
3 2 3 2 3 2 6

14 1 5 17
A terület + − + = .♦
3 6 6 3

226
12. fejezet

Példa 5 :
Adott a derékszögű koordináta-rendszerben az y = 3x2 − x3 egyenletű görbe. Számı́tsuk ki
annak a sı́kidomnak a területét, amelyet a görbe első sı́knegyedbe eső ı́ve és az x-tengely fog
közre.

Mivel most nincsenek megadva az integrálási határok, ezért ki kell őket számı́tani. Ezek a
zérushelyek.

3x2 − x3 = 0
x2 (3 − x) = 0
x1 = 0
x2 = 3

Behelyettesı́tjük az 1-et a függvénybe. 3 · 12 − 13 = 2, vagyis a [0; 3] intervallumon a függvény


nemnegatı́v.

Z3 3 
x4 34 04
   
2 3 3 3 3 27
3x − x dx = x − = 3 − = 0 − =
4 0 4 4 4
0

27
A terület .♦
4
Példa 6 : π  π π   π
Az ABCD négyzet csúcsai: A (0; 0) ; B ;0 ; C ; ; D 0; . Véletlenszerűen
2 2 2 2
kiválasztjuk a négyzet egy belső pontját.
h Mennyi a valószı́nűsége annak, hogy a kiválasztott
πi
pont a koordináta-tengelyek és az f : 0; → R, f (x) = cos x függvény grafikonja által
2
határolt tartomány egyik pontja?

227
12. fejezet

A függvény nemnegatı́v az intervallumon.


π
Z2 π π
cos x dx = [sin x]02 = sin − sin 0 = 1
2
0

Ez a terület is egyben.
π2
Tnégyzet =
4
1
p = 2 ≈ 0,4053 a keresett valószı́nűség.♦
π
4

12.5. Két függvény közötti terület


Nagyon gyakori az a feladattı́pus, amikor két függvény közötti terület kell. Ilyenkor nem számı́t,
hogy a két függvény az x-tengely alatt vagy felett van, mert akárhogy toljuk is el őket együtt,
a közöttük lévő távolság azonos marad.

Jól látszik, hogy a két terület megegyezik, annak ellenére, hogy mindkettő függvényt feljebb
toltuk.
Az sem számı́t, hogy melyik függvényből vonjuk ki a másikat, mert ha az alacsonyabban lévő
függvényből vonjuk ki a magasabbat, akkor a terület negatı́v lesz, de ezt egy előjelcserével el
lehet intézni. A két függvény közötti terület tehát minden esetben:

Zb
T = f (x) − g(x) dx
a

Példa 1 :
Mekkora területű sı́kidomot fog közre az f (x) = x+3 és g (x) = x2 +3x függvények grafikonja?

228
12. fejezet

Nincsenek megadva az integrálási határok, ezért kiszámoljuk őket. Ezek a metszéspontok lesz-
nek.

x+3 = x2 + 3x
2
x + 2x − 3 = 0
x1 = −3
x2 = 1

Z1 Z1 1
x3

2 2 2
(x + 3) − (x + 3x) dx = −x − 2x + 3 dx = − − x + 3x =
3 −3
−3 −3
 3
(−3)3
  
1 2 2 5 32
= − −1 +3·1 = − − (−3) + 3 · (−3) = − (−9) =
3 3 3 3
32
A terület .♦
3
Példa 2 :
Mekkora területű sı́kidomot fog közre az f (x) = 3x2 és g (x) = 4 − x2 függvények grafikonja?

229
12. fejezet

Ebben a feladatban nincsenek megadva az integrálási határok, ezért a 3x2 = 4 − x2 egyenletet


megoldva kapjuk meg őket.

3x2 = 4 − x2
x2 = 1
x1 = 1
x2 = −1

Z1 Z1 1
4x3

2 2 2
3x − (4 − x ) dx = 4x − 4 dx = − 4x =
3 −1
−1 −1
4 · 13 4 · (−1)3
   
8 8 16
= −4·1 − − 4 · (−1) = − − = −
3 3 3 3 3

16
A terület .♦
3
Példa 3 :
Legyen  f és g is a valós számok halmazán értelmezett függvény:
−1
 ha x ≤ −1
f (x) = 2x + 1 ha −1 < x < 0 és g(x) = x2 − 2

1 ha 0 ≤ x

Számı́tsuk ki a két függvény grafikonja által közrefogott zárt sı́kidom területét.

f (x) felrajzolása után látjuk, hogy az egyik metszéspontjuk a −1. A másikhoz egyenlet kell.
2
√ −2
1 = x
x = 3

(A − 3 nem eleme az intervallumnak.)
Mivel f más definı́ciót kap a 0 után, ezért most a két zárt sı́kidom területét külön kell kiszámolni.

230
12. fejezet

Z0 Z0 0
x3

2 2 2
(2x + 1) − (x − 2) dx = −x + 2x + 3 dx = − + x + 3x =
3 −1
−1 −1
 3
(−1)3
  
0 2 2 5
= − +0 +3·0 − − + (−1) + 3 · (−1) =
3 3 3

√ √
Z3 Z3  3 √ 3
x
1 − (x2 − 2) dx = −x2 + 3 dx = − + 3x =
3 0
0 0
√ 3 ! 
√ 03 √

3
= − +3 3 − − +3·0 =2 3
3 3

5 √
A terület + 2 3.♦
3
Példa 4 :
Mekkora a h (x) = 12 − x2 és az x 7→ −4(x ∈ R) függvények görbéi által közbezárt (korlátos)
terület?

12 − x2 = −4
16 = x2
x1 = −4
x2 = 4

Z4 Z4 4
x3

2 2
(12 − x ) − (−4) dx = 16 − x dx = 16x − =
3 −4
−4 −4
43 (−4)3
     
128 128 256
= 16 · 4 − − 16 · (−4) − = − − =
3 3 3 3 3

256
A terület .♦
3

231
12. fejezet

12.6. Határozott integrál feladatok


Ebben a részben megfordı́tjuk a korábbi feladatokat. Most tudjuk, hogy mekkora a függvény-
görbe alatt lévő terület, és az a kérdés, hogy meddig kell integrálni a függvényt. Ezek minden
esetben egyenletre vezetnek.

Példa 1 :
Adott az f : ]−1; 6[ → R, f (x) = −4x3 + 192x függvény. Jelölje c az f értelmezési
tartományának egy pozitı́v elemét. Határozzuk meg a c értékét úgy, hogy az x-tengely [0; c]
szakasza, az x-tengely és az x − c = 0 egyenletű egyenesek, valamint az f grafikonja által
közbezárt sı́kidom területe 704 területegység legyen.

−4x3 + 192x = 0
x1 = 0√
x2 = 48

x3 = − 48

Ezek egyike sincs benne a ]0; 6[ intervallumban, viszont bármely az intervallumból vett szám
helyettesı́tése esetén f pozitı́v értékű lesz. Ebből az következik, hogy a függvény teljes egészében
az x-tengely felett helyezkedik el.
Zc
−4x3 + 192x dx = 704
0
c
[−x4 + 96x2 ]0 = 704
(−c4 + 96c2 ) − 0 = 704
c4 − 96c2 + 704 = 0

Legyen d = c2

d2 − 962 + 704 = 0
d1 = 8
d2 = 88

Visszahelyettesı́tve:

c2 = 8 ⇒ c = √8
c2 = 88 ⇒ c = 88 ̸∈ [0; 6]

A negatı́v c-kel most nem kell foglalkozni, mert c pozitı́v szám.♦

Példa 2 :
Adott a valós számok halmazán értelmezett f (x) = x2 − 2 és g (x) = 10 + 10x − x2 függvény.
Határozzuk meg azt a t valós számot a [2; 8] intervallumban, amelyre teljesül, hogy az f függvény
görbéje alatti terület a [2; t] intervallumon megegyezik a g függvény görbéje alatti területtel a
[t; 8] intervallumon.
√ √
x2 − 2 = 0 megoldásai − 2 és 2, emiatt az f (x) = x2 − 2 a [2; t] intervallumban az x-tengely
felett van.
10 + 10x − x2 = 0 megoldásai pedig −0,92 és 10,92, vagyis g (x) = 10 + 10x − x2 is az x-tengely
felett van a [t; 8] intervallumon.

232
12. fejezet

Zt t
x3 t3
    3 
2 2 1 4
x − 2x dx = − 2x = − 2t − − 2 · 2 = t3 − 2t +
3 2 3 3 3 3
2

Z8 8
x3

2 2
10 + 10x − x dx = 10x + 5x − =
3 t
t
83 t3
   
2 2 688 1
= 10 · 8 + 5 · 8 − − 10t + 5t − = − 10t − 5t2 + t3
3 3 3 3

A két terület ugyanakkora, ı́gy ezek egyenlők egymással.


688 1 1 3 4
− 10t − 5t2 + t3 = t − 2t +
3 3 3 3
688 − 30t − 15t2 = −6t + 4
15t2 + 24t − 684 = 0
t1 = 6
t2 = −7,6 hamis megoldás
184
Az ellenőrzés során kijön, hogy mindkét keresett terület .♦
3
Példa 3 :
Válasszuk meg a c pozitı́v valós számot úgy, hogy az f (x) = c2 x − cx2 egyenletű görbe és az
x-tengely 10 egység területű sı́kidomot fogjon közre.

c2 x − cx2 = 0
cx(c − x) = 0
x1 = 0
x2 = c

Az integrálási határokat megkaptuk.


Zc
c2 x − cx2 dx = 10
0
c
c2 x2 cx3

− = 10
2 3 0
 4
c4

c
− −0 = 10
2 3
c4 = 60

c = 4 60

Negatı́v nem lehet c értéke. ♦

Példa 4 :
4x2
Adjuk meg a k 2 számot úgy, hogy az y = k 2 − 4x2 és az y = 1 − 2 egyenletű parabola közül
k
az egyik háromszor akkora területű sı́kidomot fogjon közre az x-tengellyel, mint a másik.

233
12. fejezet

k 2 − 4x2 = 0 4x2
1− = 0
k2 k2
= x2 k 2 = 4x2
4
k k
x = ± x = ±
2 2
Mindkét függvény esetén az integrálási határok ugyanazok.
k
Z2  k2
4x3

2 2 2
k − 4x dx = k x − =
3 − k2
− k2
  3    3 
k k
4·   4· −
 2 k 2  k 2 
 
  2
= k · −  − k · − − =

 2 3   2 3 

k3 k3 k3 k3 k3 2k 3
   
= − − − + = k3 − =
2 6 2 6 3 3

k
Z2 k
4x2 4x3 2

1 − 2 dx = x − 2 =
k 3k − k
2
− k2
  3    3 
k k
 k 4 · 2   k  4 · − 2 
= − − − − =
   
2 3k 2   2 3k 2 

   
k k k k k 2k
= − − − + =k− =
2 6 2 6 3 3

2k 3 2k
Legyen T1 = és T2 =
3 3

Ha T1 = 3T2 , akkor Ha 3T1 = T2 , akkor

2k 3 2k
= 2k 2k 3 =
3 3
3
2k − 6k = 0 6k 3 − 2k = 0
2k(k 2 − 3) = 0 2k(3k 2 − 1) = 0
↙↘ ↙↘
2
k =3 k = 0 hamis megoldás 1
k2 = k = 0 hamis megoldás
3
1
k 2 lehetséges értékei 3 és .♦
3

234
13. fejezet

13. Gondolkodási műveletek, halmazok


13.1. A skatulya-elv
A skatulya-elv azt az egyszerű gondolatmenetet jelenti, hogy ha van n darab dobozunk, és n + 1
darab elemünk, akkor biztosan lesz olyan doboz, amibe legalább két elem kerül.
A skatulya-elv feladatoknál mindig a legrosszabb esetet nézzük. Feltöltjük a maximális
lehetséges elemszámmal a skatulyákat úgy, hogy még ne teljesüljön a feladat feltétele. Ezután
egy újabb elem hozzáadásával már teljesülni fog.

Példa 1 :
Legalább mekkora létszámúnak kell lennie annak az osztálynak, hogy biztosan legyen köztük
három ember, aki ugyanabban a hónapban született?

2 2 2 2 2 2 2 2 2 2 2 2
Jan F eb M ar Apr M aj Jun Jul Aug Szep Okt N ov Dec

Minden hónap maximálisan van feltöltve, de ha hozzátesszük az új tanulót az osztályhoz, akkor
valamelyik hónapba már biztosan bele fog esni. Tehát 12·2+1 = 25 a minimális osztálylétszám.♦

Példa 2 :
Egy 5 × 5-ös táblázatot tetszőlegesen kitöltünk +1 és −1 számokkal. Ezután képezzük a sorok
és az oszlopok összegét. Bizonyı́tsuk be, hogy az ı́gy kapott számok között mindig lesz legalább
kettő azonos.

Nézzük végig, hogy milyen lehetőségek vannak.


Ha mind +1, az összeg 5.
Ha négy darab +1 és egy darab −1, az összeg 3.
Ha három darab +1 és két darab −1, az összeg 1.
Ha két darab +1 és három darab −1, az összeg −1.
Ha egy darab +1 és négy darab −1, az összeg −3.
Ha mind −1, az összeg −5.
Ez azt jelenti, hogy hat lehetőség van az összegre, de mivel tı́z oszlop és sor van összesen, ezért
biztos, hogy legalább kettő közös lesz közülük.♦

Példa 3 :
Bizonyı́tsuk be, hogy bármely hét négyzetszám között van legalább két olyan, amelyeknek a
különbsége osztható tı́zzel.

Nézzük végig a számjegyek négyzeteinek végződéseit.

02 = 0; 12 = 1; 22 = 4; 32 = 9; 42 = 16; 52 = 25; 62 = 36; 72 = 49; 82 = 64; 92 = 81

Látható, hogy egy négyzetszám sosem végződhet 2; 3; 7; 8 számjegyekre, tehát egy négyzetszám
végződése csak hatféle lehet: 0; 1; 4; 5; 6; 9.
Ha választunk hét négyzetszámot, akkor ezek között biztosan lesz kettő ugyanarra végződő
szám. Ha két ugyanolyan számjegyre végződő számot kivonunk egymásból, akkor ezek kiesésével
a különbség biztosan nullára végződik, vagyis osztható tı́zzel.♦

235
13. fejezet

Példa 4 :
Bizonyı́tsuk be, hogy létezik a háromnak két olyan hatványa, melyek különbsége osztható 2017-
tel.

2017-tel való oszthatósági maradék 2017 féle lehet: {0; 1; 2; . . . ; 2016}. Legrosszabb esetben
háromnak az első 2017 hatványa 2017-tel való osztási maradék szempontjából mind különböző.
Vagyis vesszük az első 2018 hatványát: 31 ; 32 ; 33 ; . . . ; 32018 . Ezek között biztosan lesz
legalább kettő, amelyek ugyanazt a maradékot adják 2017-tel osztva.
Az egyik szám legyen 2017k + i, a másik 2017l + i alakú, ahol i a közös maradékot jelenti. Ha
ezeket a számokat kivonjuk egymásból, akkor

2017k + i − (2017l + i) = 2017k − 2017l = 2017(k − l)

Tehát a különbség osztható 2017-tel.♦

A következő két feladatban visszafelé használjuk a skatulya-elvet. Először gondoljuk végig az


első példát újra. 12 skatulya van, és legalább három ember kellett, hogy biztosan beleessen
a harmadik tanuló. Tehát lecsı́pünk a 25-ből egyet, mert ı́gy még nem teljesül, és az összes
skatulyát feltöltjük kettő emberrel. A 25 úgy jött ki, hogy a skatulyák száma, megszorozva a
belerakható maximális elemszámmal, és a végén +1. Ezt használjuk most visszafelé.

Példa 5 :
Bizonyı́tsuk be, hogy akárhogy helyezünk el 51 pontot egy egységnyi területű négyzetben, min-
1
dig lesz három pont, amely lefedhető egy sugarú körrel.
7
Az 51 pont azt jelenti, hogy akkor már teljesül, hogy lesz három pont, szóval leveszünk belőle
egyet. Vagyis az 50 pont a maximális, hogy minden skatulyát kettő ponttal töltsünk fel, azaz
25 skatulya van. A 25-ről és arról, hogy egy négyzetet bontunk fel, beugrik az, hogy esetleg egy
5×5-ös hálót rakjunk a négyzetre. Minden kis négyzetben ı́gy két-két pont van. Ha hozzáveszünk
egy 51. pontot, akkor lesz olyan, amelyikben már három pont lesz.

1
Ezek után kiszámoljuk az oldalú négyzet körülı́rható körének sugarát. Erre azért van szükség,
5
mert a négyzet köré ı́rható kör az, ami lefedi egészében a négyzetet.

236
13. fejezet

Pitagorasz-tétel:
 2
1
= r2 + r2
5
1
r = √
50
1
Mivel ≈ 0,1414 < , ezért abban az esetben, hogy a három pont épp a négyzet csúcsaira esik,
7
1
az sugarú kör már biztosan tartalmazza mindhárom pontot.♦
7
Példa 6 :
Egy 10 m széles és 20 m hosszú kertben 51 darab gyümölcsfát ültettek. Bizonyı́tsuk be, hogy
van legalább két olyan gyümölcsfa, melyeket három méternél közelebb ültettek egymáshoz.

Most is az előző gondolatmenetet használjuk. 50 fát kell elhelyezni és mindegyik skatulyába egy
fa kerül. A 10 × 20 méteres kertet felosztjuk 50 darab 2 × 2 méteres négyzetre. Az 51. fával lesz
négyzet, amelyikben két fa van. Megint kiszámoljuk a 2 m oldalú négyzet körülı́rható körének
sugarát.

Pitagorasz-tétel:

22 = r√2 + r2
r = 2 ⇒ d ≈ 2,82 < 3

Egy körben az átmérő a leghosszabb húr, vagyis három méternél biztosan közelebb vannak
egymáshoz.♦

13.2. A teljes indukciós bizonyı́tás


A módszer elvét hűen reprezentálja a dominó-elv, amelynél az a célunk, hogy eldöntsünk min-
den dominót, amit függőlegesen szorosan egymás mellé felállı́tottunk. Ehhez csupán annyit kell
tennünk, hogy az első dominót meglökjük, mert ez láncreakciószerűen maga után vonja azt,
hogy az összes dominó eldől.
A teljes indukciós bizonyı́tásoknál a képletünk már megvan. A bizonyı́tás alapelve tehát az,
hogy megmutatjuk, hogy egy kiinduló elemre igaz az állı́tás, majd megmutatjuk az öröklődési
tulajdonságot.

237
13. fejezet

Példa 1 :
n(n + 1)
Bizonyı́tsuk be, hogy az első n pozitı́v egész szám összege: 1 + 2 + 3 + . . . + n =
2
1·2
Ha n = 1, akkor 1 = , ez igaz.
2
Most feltesszük, hogy az állı́tás igaz n = k esetén.
Írjuk fel n = k + 1-re is az állı́tást. Vagyis a képletbe n helyére mindenhova k + 1-et kell ı́rni.
Ügyeljünk a zárójelekre!

(k + 1) [(k + 1) + 1]
1 + 2 + 3 + . . . + k + (k + 1) =
2

Felhasználjuk az állı́tást, amiről feltettük, hogy igaz:

(k + 1)(k + 2)
|1 + 2 + 3{z+ . . . + k} +(k + 1) = 2
k(k+1)
2

k(k + 1) (k + 1)(k + 2)
+k+1 =
2 2
2 2
k + k + 2k + 2 = k + 3k + 2
k 2 + 3k + 2 = k 2 + 3k + 2

Igazság jött ki, ezzel beláttuk, hogy az állı́tás igaz.♦

Példa 2 :
n(n + 1)(2n + 1)
Bizonyı́tsuk be, hogy az első n pozitı́v négyzetszám összege: 12 +22 +. . .+n2 =
6
1·2·3
Ha n = 1, akkor 12 = , ez igaz.
6
Megint feltesszük, hogy az állı́tás igaz n = k esetén.
Felı́rjuk n = k + 1-re az állı́tást.

(k + 1)[(k + 1) + 1][(2(k + 1) + 1)]


12 + 22 + 32 + . . . + k 2 + (k + 1)2 =
6

Felhasználjuk az állı́tást, amiről feltettük, hogy igaz:

(k + 1)(k + 2)(2k + 3)
1| 2 + 22 + 3{z
2
+ . . . + k}2 +(k + 1)2 =
6
k(k+1)(2k+1)
6

k(k + 1)(2k + 1) (k + 1)(k + 2)(2k + 3)


+ (k + 1)2 =
6 6
3 2 2 2
2k + 3k + k + 6(k + 2k + 1) = (k + 1) (2k + 7k + 6)
2k 3 + 9k 2 + 13k + 6 = 2k 3 + 9k 2 + 13k + 6

Igazság jött ki, ezzel beláttuk, hogy az állı́tás igaz.♦

Példa 3 :
Bizonyı́tsuk be, hogy a pozitı́v páratlan számok összege mindig négyzetszám, vagyis:
1 + 3 + 5 + . . . + (2n − 1) = n2

Ha n = 1, akkor 1 = 12 , ez igaz.

238
13. fejezet

Megint feltesszük, hogy az állı́tás igaz n = k esetén.


Felı́rjuk n = k + 1-re az állı́tást.

1 + 3 + 5 + . . . + (2k − 1) + (2(k + 1) − 1) = (k + 1)2

Felhasználjuk az állı́tást, amiről feltettük, hogy igaz:

1 + 3 + 5 + . . . + (2k − 1) +(2k + 1) = (k + 1)2


| {z }
k2
k 2 + 2k + 1 = k 2 + 2k + 1

Igazság jött ki, ezzel beláttuk, hogy az állı́tás igaz.♦

Példa 4 :
Bizonyı́tsuk be, hogy 4n + 6n − 1 minden n pozitı́v egész szám esetén osztható kilenccel.

Ha n = 1, akkor 4 + 6 − 1 = 9, ez osztható kilenccel.


Megint feltesszük, hogy az állı́tás igaz n = k esetén.
Felı́rjuk n = k + 1-re az állı́tást.

4k+1 + 6(k + 1) − 1 = 4 · 4k + 6k + 5

Felhasználjuk az állı́tást, amiről feltettük, hogy igaz. Mivel 4 · 4k szerepel, ezért a kifejezés
négyszeresét állı́tjuk elő, amiről már tudjuk, hogy osztható kilenccel, és a ,,maradékokat” kom-
penzáljuk.

4 · 4k + 6k + 5 = 4 · 4k + 6k − 1 −18k + 9

| {z }
osztható 9-cel

−18k a 18 miatt, 9 pedig nyilvánvalóan osztható kilenccel, ezzel beláttuk, hogy az állı́tás igaz,
mert kilenccel osztható számok összege is osztható kilenccel.♦

13.3. Az indirekt bizonyı́tás


Egy tétel mindig vagy igaz, vagy hamis. Az indirekt bizonyı́tás lényege az, hogy feltesszük
az állı́tásról, hogy nem igaz, és megmutatjuk, hogy ez ellentmondásra vezet minden esetben.
Következésképp az eredeti tételnek igaznak kell lennie.

Példa 1 : √
Bizonyı́tsuk be, hogy 2 irracionális.
√ p
Tegyük fel, hogy alakban, ahol p, q ∈ N+ . Azt is feltehetjük,
2 racionális, vagyis felı́rható
q
hogy a tört tovább nem egyszerűsı́thető, azaz (p; q) = 1.
√ p
2 =
q
2q = p2
2

239
13. fejezet

p2 -nek párosnak kell lennie, de akkor 2 biztosan páros kitevővel szerepel p2 -ben. q 2 -ben akár
szerepel a 2-es prı́mtényező, akár nem, biztosan abban is páros kitevővel szerepel, de ha meg-
szorozzuk 2-vel, akkor már páratlan kitevővel fog szerepelni. Ezért nem lehet, hogy 2q 2 = p2 . ♦

Példa 2 :
Bizonyı́tsuk be, hogy végtelen sok prı́mszám létezik.

Tegyük fel, hogy véges sok létezik, vagyis felsorolhatók: p1 ; p2 ; p3 ; . . . ; pn .


n
Y
Vegyük ezek szorzatát. Legyen A = p1 · p2 · p3 · . . . , pn = pi .
i=1
Ekkor A osztható az összes prı́mmel.
Vegyük most A + 1-et. Ez a +1 elrontotta az összes oszthatóságot, mert A + 1 most minden
prı́mmel osztva egyet ad maradékul. Ez azt jelenti, hogy egyik ismert prı́mmel sem osztható,
vagyis A + 1 egy új prı́m. De ez ellenkezik az eredeti feltevéssel, hogy csak n darab van. Vagyis
végtelen sok prı́m van.♦

13.4. Bizonyı́tás elégséges feltételek láncolatával


Egy klasszikus bizonyı́tás általában A ⇒ L1 ⇒ L2 ⇒ . . . ⇒ Ln ⇒ B szerkezetű. Vagyis kiin-
dulunk a feltételből, abból következik az első lépés, abból a második lépés és ı́gy tovább. Az
utolsó lépésből pedig adódik az állı́tás következménye.
Az elégséges feltételekkel bizonyı́tás B ⇐ L1 ⇐ L2 ⇐ . . . ⇐ Ln ⇐ A szerkezetű. Kiindulunk
a következményből, ehhez keresünk egy elégséges feltételt, aztán ahhoz is egyet és ı́gy tovább.
Az utolsóból pedig már következni fog valami nyilvánvaló dolog. Ha fordı́tva csinálnánk, akkor
nem feltétlenül jutna eszünkbe, hogy pont ebből a nyilvánvaló dologból induljunk ki.

Példa 1 :
Bizonyı́tsuk be a számtani és mértani közép közti egyenlőtlenséget két tagra, ahol a, b ≥ 0.

Kiindulunk abból, hogy


a+b √
≥ ab
2

Ehhez keresünk folyamatosan elégséges feltételeket:

⇑ √
a+b ≥ 2 ab

2 2
a + 2ab + b ≥ 4ab

2 2
a − 2ab + b ≥ 0

2
(a − b) ≥ 0

Ezt pedig már tudjuk. Egyenlőség akkor áll fenn, ha a = b.♦

240
13. fejezet

Példa 2 :
1
Bizonyı́tsuk be, hogy ha x > 0, akkor x + ≥ 2.
x
Kiindulunk abból, hogy
1
x+ ≥2
x

Ehhez keresünk újra elégséges feltételeket:


2
x + 1 ≥ 2x

2
x − 2x + 1 ≥ 0

(x − 1)2 ≥ 0

Ez már mindig igaz. Egyenlőség x = 1 esetén áll fenn. ♦

Példa 3 :  2
a+b
Bizonyı́tsuk be, hogy ha a, b ≥ 0, akkor ab ≤ .
2

Kiindulunk abból, hogy


 2
a+b
ab ≤
2

A következő elégséges feltételeket találjuk:


a2 + 2ab + b2
ab ≤
4

4ab ≤ a2 + 2ab + b2

0 ≤ a2 − 2ab + b2

0 ≤ (a − b)2

Most is egyenlőség a = b esetén áll fenn.♦

Mindhárom feladatban a kiindulási adat, vagyis a levezetés utolsó sora olyan, hogy valószı́nűleg
senkinek nem jutna eszébe pont ezzel elindulni. Hozzátenném, hogy természetesen az elégséges
feltételek közül több ekvivalencia is egyben, de a bizonyı́tás folyamatában ez az információ nem
lényeges.

241
13. fejezet

13.5. Bizonyı́tás nevezetes közepekkel


A nevezetes közepeket is fel lehet használni egyenlőtlenségek bizonyı́tásához.
n tagra a következő nevezetes közepek léteznek:
n
Harmonikus közép:
1 1 1
+ + ... +
a1 a2 an

Mértani közép: n
a1 · a2 · . . . · an
a1 + a2 + . . . + an
Számtani közép:
r n
a1 2 + a2 2 + . . . + an 2
Négyzetes közép:
n
A nevezetes közepeknek a sorrendje pedig:

harmonikus közép ≤ mértani közép ≤ számtani közép ≤ négyzetes közép

Példa 1 :
a b c
Bizonyı́tsuk be, hogy + + ≥ 3, ha a; b; c > 0.
b c a

Írjuk fel a három törtre a számtani és mértani közép közti egyenlőtlenséget:
a b c
+ + r
b c a ≥ 3 a·b·c
3 b c a
a b c
+ +
b c a ≥√ 3
1
3
a b c
+ + ≥ 3♦
b c a

Példa 2 :
n+1 √
Bizonyı́tsuk be, hogy ≥ n n!, ha n ∈ N.
2
n! jelentése segı́t abban a gondolatban, hogy ı́rjuk fel 1-től n-ig a természetes számokra a
számtani és mértani közép közti egyenlőtlenséget.
1 + 2 + 3 + ... + n √
≥ n 1 · 2 · 3 · ... · n
n
n(n + 1)
2 √
≥ n n!
n
n+1 √
≥ n n!♦
2

Példa 3 :
Bizonyı́tsuk be, hogy a3 + b3 + c3 ≥ 3abc, ha a; b; c > 0.

Felı́rjuk a három összetevőre a számtani és mértani közép közti egyenlőtlenséget:

242
13. fejezet

a3 + b 3 + c 3 √3
≥ a3 · b 3 · c 3
3
a3 + b 3 + c 3
≥ abc
3
a3 + b3 + c3 ≥ 3abc♦

Példa 4 :
1 1 1 9
Bizonyı́tsuk be, hogy + + ≥ , ha a; b; c > 0.
a b c a+b+c
Most a harmonikus és számtani közepeket ı́rjuk fel:
3 a+b+c

1 1 1 3
+ +
a b c
9
≤ a+b+c
1 1 1
+ +
a b c  
1 1 1
9 ≤ (a + b + c) + +
a b c
9 1 1 1
≤ + + ♦
a+b+c a b c

13.6. Folytonos halmazok megadása


Egy halmaz folytonos, ha elemei nem sorolhatók fel. Ezek általában ponthalmazokat,
geometriai alakzatokat alkotnak és koordináta-rendszerben ábrázolhatók.

Példa 1 :
Adjuk meg a következő halmaz elemeit: {P (x; y)|(x − 4)(y − 4) ≥ 0}

(x − 4)(y − 4) ≥ 0
↙ ↘
x − 4 ≥ 0 és y − 4 ≥ 0 x − 4 ≤ 0 és y − 4 ≤ 0
x ≥ 4 és y ≥ 4 x ≤ 4 és y ≤ 4

A bal oldali ág a (4; 4) ponttól ,,jobbra felfelé” lévő részt, mı́g a jobb oldali ág a (4; 4) ponttól
,,balra lefelé” lévő részt jelenti.

243
13. fejezet

Szöveggel megfogalmazva: a végeredmény két zárt negyedsı́kban lévő pontok uniója.♦

Példa 2 :
Határozzuk meg a sı́k azon P (x; y) pontjait, amelyek koordinátáira teljesül az |y − x| ≤ 2
egyenlőtlenség.

|y − x| ≤ 2
−2 ≤ y−x ≤ 2
x−2 ≤ y ≤ x+2

A megoldás a két egyenes közötti zárt sávba eső pontok.♦

Példa 3 :
Határozzuk meg a sı́k azon P (x; y) pontjait, amelyek koordinátáira teljesül az x2 + y 2 ≤ 16
egyenlőtlenség.

x2 + y 2 = 16 egy origó középpontú 4 egység sugarú kör egyenlete. De mivel most x2 + y 2 ≤ 16,
ezért a kör belseje is kell.

244
13. fejezet

Szöveggel megfogalmazva: a végeredmény egy zárt körlemezbe eső pontok halmaza.♦

Példa 4 :
Határozzuk meg a sı́k azon P (x; y) pontjait, amelyek koordinátáira egyszerre teljesül a
4|x−y| ≤ 8 és a 8|x−y| ≥ 4 egyenlőtlenség.

4|x−y| ≤8 8|x−y| ≥4
3|x−y|
22|x−y|
≤2 3 2 ≥ 22
mivel az exp. fv. szig. mon nő... mivel az exp. fv. szig. mon nő...
2|x − y| ≤ 3 3|x − y| ≥ 2
3 2
|x − y| ≤ |x − y| ≥
2 3
3 3 ↙↘
− ≤ x−y ≤
2 2
3 3 2 2
− −x≤ −y ≤ −x x−y ≥ x−y ≤ −
2 2 3 3
3 3 2 2
x+ ≥ y ≥x− x− ≥ y x+ ≤ y
2 2 3 3
3 3
A bal oldali ág az y = x + és y = x − egyenesek közötti zárt sáv, mı́g a jobb oldali ág az
2 2
2 2
y = x + és y = x − egyeneseken kı́vül eső terület.
3 3

Mivel a két egyenlőtlenség által meghatározott halmazoknak egyszerre kell teljesülniük, ezért
a két ,,szı́nes rész” metszete kell. A végeredmény a rajzon látható.♦

245
13. fejezet

246

You might also like